Download UPSC CMS Paper II 2020 Question Paper with Answer Key

Download UPSC Combined Medical Services (CMS) CMS Paper II 2020 Question Paper with Answer Key

Combined Medical Services Examination-2020
Paper-II

1.
Indications for fasciotomy in compartment syndrome include all EXCEPT:

(a) Distal sensory disturbance
(b) Compartment pressure > 30 mm Hg
(c) Pain on passive movement of affected muscles
(d) Palpable distal pulses
2.
Which one of the following statements is NOT correct regarding Necrotising Soft Tissue
infections?

(a) Crepitus, skin blistering and focal skin gangrene are typical presenting features
(b) They are monomicrobial in nature
(c) Treatment consists of wide local excision and appropriate antibiotics
(d) Tissue biopsy is required for culture and diagnosis
3.
Which type of surgery is laparoscopic cholecystectomy classified as?

(a) Clean
(b) Clean contaminated
(c) Contaminated
(d) Dirty
4.
A 22-year female has presented with a history of malaise, cough, alternating constipation and
diarrhoea with intermittent abdominal pain for last 6 months. She also complains of
abdominal distension for last 2 days. On examination her abdomen has a doughy feel along
with an ill defined mass over the right lower quadrant. She is most likely suffering from:

(a) Appendicular lump
(b) Ileocaecal tuberculosis
(c) Carcinoma caecum
(d) Ovarian mass
5.
Consider the following statements regarding needle stick injuries:

1. Injured part should be washed under running water
2. Dominant index finger is the commonest site for needle stick injury
3. All needle stick injuries should be reported
4. Hepatitis/HIV testing should be done after needle stick injury

Which of the statements given above are correct?

(a) 1, 2 and 4
(b) 1, 2 and 3
(c) 1, 3 and 4
(d) 2, 3 and 4

FirstRanker.com - FirstRanker's Choice
Combined Medical Services Examination-2020
Paper-II

1.
Indications for fasciotomy in compartment syndrome include all EXCEPT:

(a) Distal sensory disturbance
(b) Compartment pressure > 30 mm Hg
(c) Pain on passive movement of affected muscles
(d) Palpable distal pulses
2.
Which one of the following statements is NOT correct regarding Necrotising Soft Tissue
infections?

(a) Crepitus, skin blistering and focal skin gangrene are typical presenting features
(b) They are monomicrobial in nature
(c) Treatment consists of wide local excision and appropriate antibiotics
(d) Tissue biopsy is required for culture and diagnosis
3.
Which type of surgery is laparoscopic cholecystectomy classified as?

(a) Clean
(b) Clean contaminated
(c) Contaminated
(d) Dirty
4.
A 22-year female has presented with a history of malaise, cough, alternating constipation and
diarrhoea with intermittent abdominal pain for last 6 months. She also complains of
abdominal distension for last 2 days. On examination her abdomen has a doughy feel along
with an ill defined mass over the right lower quadrant. She is most likely suffering from:

(a) Appendicular lump
(b) Ileocaecal tuberculosis
(c) Carcinoma caecum
(d) Ovarian mass
5.
Consider the following statements regarding needle stick injuries:

1. Injured part should be washed under running water
2. Dominant index finger is the commonest site for needle stick injury
3. All needle stick injuries should be reported
4. Hepatitis/HIV testing should be done after needle stick injury

Which of the statements given above are correct?

(a) 1, 2 and 4
(b) 1, 2 and 3
(c) 1, 3 and 4
(d) 2, 3 and 4




6.
Consider the following statements regarding claudication:

1. It is a marker for silent coronary disease
2. Structured exercise program ( 2 hours per week for 3 months) leads to
improvement in symptoms
3. Diabetes mellitus increases the risk and severity of claudication
4. Beta blockers may exacerbate claudication

Which of the above statements are correct?

(a) 1 and 2 only
(b) 1, 3 and 4 only
(c) 2, 3 and 4 only
(d) 1, 2, 3 and 4
7.
A 50-year old lady underwent uneventful bariatric surgery for morbid obesity. On the third
post operative day, she develops breathless and pulmonary embolism is suspected. The next
investigation to confirm the diagnosis will be:

(a) Echocardiography
(b) Duplex venography
(c) CT pulmonary angiography
(d) MR angiography
8.
Medical management of thyrotoxic crisis includes all of the following EXCEPT:

(a) IV fluids
(b) IV propanolol
(c) IV hydrocortisone
(d) IV antibiotics
9.
A patient operated for a parotid gland tumour developed symptoms of sweating and
erytherma (flushing) over the region of surgical excision while eating. The probable
diagnosis is:

(a) Parotid gland fistula
(b) Sialadenitis
(c) Chronic wound infection
(d) Frey?s syndrome
10.
First line hormone therapy for post-menopausal woman with metastatic carcinoma breast is:

(a) Tamoxifen
(b) Ovarian suppression by surgery
(c) Antiprogestins
(d) Anastrazole
FirstRanker.com - FirstRanker's Choice
Combined Medical Services Examination-2020
Paper-II

1.
Indications for fasciotomy in compartment syndrome include all EXCEPT:

(a) Distal sensory disturbance
(b) Compartment pressure > 30 mm Hg
(c) Pain on passive movement of affected muscles
(d) Palpable distal pulses
2.
Which one of the following statements is NOT correct regarding Necrotising Soft Tissue
infections?

(a) Crepitus, skin blistering and focal skin gangrene are typical presenting features
(b) They are monomicrobial in nature
(c) Treatment consists of wide local excision and appropriate antibiotics
(d) Tissue biopsy is required for culture and diagnosis
3.
Which type of surgery is laparoscopic cholecystectomy classified as?

(a) Clean
(b) Clean contaminated
(c) Contaminated
(d) Dirty
4.
A 22-year female has presented with a history of malaise, cough, alternating constipation and
diarrhoea with intermittent abdominal pain for last 6 months. She also complains of
abdominal distension for last 2 days. On examination her abdomen has a doughy feel along
with an ill defined mass over the right lower quadrant. She is most likely suffering from:

(a) Appendicular lump
(b) Ileocaecal tuberculosis
(c) Carcinoma caecum
(d) Ovarian mass
5.
Consider the following statements regarding needle stick injuries:

1. Injured part should be washed under running water
2. Dominant index finger is the commonest site for needle stick injury
3. All needle stick injuries should be reported
4. Hepatitis/HIV testing should be done after needle stick injury

Which of the statements given above are correct?

(a) 1, 2 and 4
(b) 1, 2 and 3
(c) 1, 3 and 4
(d) 2, 3 and 4




6.
Consider the following statements regarding claudication:

1. It is a marker for silent coronary disease
2. Structured exercise program ( 2 hours per week for 3 months) leads to
improvement in symptoms
3. Diabetes mellitus increases the risk and severity of claudication
4. Beta blockers may exacerbate claudication

Which of the above statements are correct?

(a) 1 and 2 only
(b) 1, 3 and 4 only
(c) 2, 3 and 4 only
(d) 1, 2, 3 and 4
7.
A 50-year old lady underwent uneventful bariatric surgery for morbid obesity. On the third
post operative day, she develops breathless and pulmonary embolism is suspected. The next
investigation to confirm the diagnosis will be:

(a) Echocardiography
(b) Duplex venography
(c) CT pulmonary angiography
(d) MR angiography
8.
Medical management of thyrotoxic crisis includes all of the following EXCEPT:

(a) IV fluids
(b) IV propanolol
(c) IV hydrocortisone
(d) IV antibiotics
9.
A patient operated for a parotid gland tumour developed symptoms of sweating and
erytherma (flushing) over the region of surgical excision while eating. The probable
diagnosis is:

(a) Parotid gland fistula
(b) Sialadenitis
(c) Chronic wound infection
(d) Frey?s syndrome
10.
First line hormone therapy for post-menopausal woman with metastatic carcinoma breast is:

(a) Tamoxifen
(b) Ovarian suppression by surgery
(c) Antiprogestins
(d) Anastrazole



11.
All of the following are major subtypes of breast cancer based on Gene array analysis
EXCEPT:

(a) Luminal A and Luminal B
(b) Triple negative
(c) Her-2 receptor positive
(d) Oestrogen receptor positive
12.
All of the following are sequelae of peptic ulcer surgery EXCEPT:

(a) Bilious vomiting
(b) Dumping syndrome
(c) Diarrhoea
(d) Increased appetite
13.
The Child-Turcotte-Pugh (CTP) score for quantifying the severity of chronic liver disease
includes all variables EXCEPT:

(a) Serum bilirubin
(b) Serum albumin
(c) Serum creatinine
(d) INR (International Normalised Ratio)
14.
?Chain of Lakes? appearance due to sacculation with intervening short strictures of pancreatic
duct is seen on:

(a) ERCP
(b) CECT abdomen
(c) Plain X-ray abdomen
(d) Ultrasonography
15.
Which one of the following statements is NOT correct regarding Pyogenic Liver Abscess?

(a) Anorexia, fever, malaise and right upper quadrant abdominal discomfort are the
most common presenting features
(b) It is more common in elderly, diabetics and immunocompromised patients
(c) Treatment is with oral antibiotics alone
(d) Streptococcus milleri and escherichia coli are the most common causative
organisms
16.
?Swiss cheese defects? are seen during laparoscopic repair of:

(a) Ventral hernia
(b) Inguinal hernia
(c) Obturator hernia
(d) Femoral hernia
FirstRanker.com - FirstRanker's Choice
Combined Medical Services Examination-2020
Paper-II

1.
Indications for fasciotomy in compartment syndrome include all EXCEPT:

(a) Distal sensory disturbance
(b) Compartment pressure > 30 mm Hg
(c) Pain on passive movement of affected muscles
(d) Palpable distal pulses
2.
Which one of the following statements is NOT correct regarding Necrotising Soft Tissue
infections?

(a) Crepitus, skin blistering and focal skin gangrene are typical presenting features
(b) They are monomicrobial in nature
(c) Treatment consists of wide local excision and appropriate antibiotics
(d) Tissue biopsy is required for culture and diagnosis
3.
Which type of surgery is laparoscopic cholecystectomy classified as?

(a) Clean
(b) Clean contaminated
(c) Contaminated
(d) Dirty
4.
A 22-year female has presented with a history of malaise, cough, alternating constipation and
diarrhoea with intermittent abdominal pain for last 6 months. She also complains of
abdominal distension for last 2 days. On examination her abdomen has a doughy feel along
with an ill defined mass over the right lower quadrant. She is most likely suffering from:

(a) Appendicular lump
(b) Ileocaecal tuberculosis
(c) Carcinoma caecum
(d) Ovarian mass
5.
Consider the following statements regarding needle stick injuries:

1. Injured part should be washed under running water
2. Dominant index finger is the commonest site for needle stick injury
3. All needle stick injuries should be reported
4. Hepatitis/HIV testing should be done after needle stick injury

Which of the statements given above are correct?

(a) 1, 2 and 4
(b) 1, 2 and 3
(c) 1, 3 and 4
(d) 2, 3 and 4




6.
Consider the following statements regarding claudication:

1. It is a marker for silent coronary disease
2. Structured exercise program ( 2 hours per week for 3 months) leads to
improvement in symptoms
3. Diabetes mellitus increases the risk and severity of claudication
4. Beta blockers may exacerbate claudication

Which of the above statements are correct?

(a) 1 and 2 only
(b) 1, 3 and 4 only
(c) 2, 3 and 4 only
(d) 1, 2, 3 and 4
7.
A 50-year old lady underwent uneventful bariatric surgery for morbid obesity. On the third
post operative day, she develops breathless and pulmonary embolism is suspected. The next
investigation to confirm the diagnosis will be:

(a) Echocardiography
(b) Duplex venography
(c) CT pulmonary angiography
(d) MR angiography
8.
Medical management of thyrotoxic crisis includes all of the following EXCEPT:

(a) IV fluids
(b) IV propanolol
(c) IV hydrocortisone
(d) IV antibiotics
9.
A patient operated for a parotid gland tumour developed symptoms of sweating and
erytherma (flushing) over the region of surgical excision while eating. The probable
diagnosis is:

(a) Parotid gland fistula
(b) Sialadenitis
(c) Chronic wound infection
(d) Frey?s syndrome
10.
First line hormone therapy for post-menopausal woman with metastatic carcinoma breast is:

(a) Tamoxifen
(b) Ovarian suppression by surgery
(c) Antiprogestins
(d) Anastrazole



11.
All of the following are major subtypes of breast cancer based on Gene array analysis
EXCEPT:

(a) Luminal A and Luminal B
(b) Triple negative
(c) Her-2 receptor positive
(d) Oestrogen receptor positive
12.
All of the following are sequelae of peptic ulcer surgery EXCEPT:

(a) Bilious vomiting
(b) Dumping syndrome
(c) Diarrhoea
(d) Increased appetite
13.
The Child-Turcotte-Pugh (CTP) score for quantifying the severity of chronic liver disease
includes all variables EXCEPT:

(a) Serum bilirubin
(b) Serum albumin
(c) Serum creatinine
(d) INR (International Normalised Ratio)
14.
?Chain of Lakes? appearance due to sacculation with intervening short strictures of pancreatic
duct is seen on:

(a) ERCP
(b) CECT abdomen
(c) Plain X-ray abdomen
(d) Ultrasonography
15.
Which one of the following statements is NOT correct regarding Pyogenic Liver Abscess?

(a) Anorexia, fever, malaise and right upper quadrant abdominal discomfort are the
most common presenting features
(b) It is more common in elderly, diabetics and immunocompromised patients
(c) Treatment is with oral antibiotics alone
(d) Streptococcus milleri and escherichia coli are the most common causative
organisms
16.
?Swiss cheese defects? are seen during laparoscopic repair of:

(a) Ventral hernia
(b) Inguinal hernia
(c) Obturator hernia
(d) Femoral hernia



17.
Which of the following are correct regarding splenic artery aneurysm?

1. Main arterial trunk is the common site
2. Palpable thrill can be felt
3. It is symptomless unless it ruptures

Select the correct answer using the code given below:

(a) 1 and 2 only
(b) 2 and 3 only
(c) 1 and 3 only
(d) 1, 2 and 3
18.
Valentino?s syndrome is:

(a) Pain on per-vaginal examination in pelvic abscess
(b) Pain over left shoulder in left hypochondriac collection
(c) Pain over left groin in perirenal collection
(d) Pain in right iliac fossa in perforated peptic ulcer
19.
Spontaneous bacterial peritonitis occurs due to:

(a) duodenal stump blowout
(b) peptic ulcer perforation
(c) acute bacterial infection of ascites
(d) infection via fallopian tubes
20.
Structure not forming boundaries of the ?Triangle of doom? seen during laparoscopic
inguinal hernia surgery dissection is:

(a) Vas deferens
(b) Inferior epigastric artery
(c) Spermatic cord vessels
(d) Peritoneum
21.
The term mid-line shift is associated with:

(a) Head injury
(b) Chest injury
(c) Abdominal injury
(d) Limb injury
22.
Which one of the following cranial nerves does NOT supply to the external ear?

(a) Cranial nerve V
(b) Cranial nerve VI
FirstRanker.com - FirstRanker's Choice
Combined Medical Services Examination-2020
Paper-II

1.
Indications for fasciotomy in compartment syndrome include all EXCEPT:

(a) Distal sensory disturbance
(b) Compartment pressure > 30 mm Hg
(c) Pain on passive movement of affected muscles
(d) Palpable distal pulses
2.
Which one of the following statements is NOT correct regarding Necrotising Soft Tissue
infections?

(a) Crepitus, skin blistering and focal skin gangrene are typical presenting features
(b) They are monomicrobial in nature
(c) Treatment consists of wide local excision and appropriate antibiotics
(d) Tissue biopsy is required for culture and diagnosis
3.
Which type of surgery is laparoscopic cholecystectomy classified as?

(a) Clean
(b) Clean contaminated
(c) Contaminated
(d) Dirty
4.
A 22-year female has presented with a history of malaise, cough, alternating constipation and
diarrhoea with intermittent abdominal pain for last 6 months. She also complains of
abdominal distension for last 2 days. On examination her abdomen has a doughy feel along
with an ill defined mass over the right lower quadrant. She is most likely suffering from:

(a) Appendicular lump
(b) Ileocaecal tuberculosis
(c) Carcinoma caecum
(d) Ovarian mass
5.
Consider the following statements regarding needle stick injuries:

1. Injured part should be washed under running water
2. Dominant index finger is the commonest site for needle stick injury
3. All needle stick injuries should be reported
4. Hepatitis/HIV testing should be done after needle stick injury

Which of the statements given above are correct?

(a) 1, 2 and 4
(b) 1, 2 and 3
(c) 1, 3 and 4
(d) 2, 3 and 4




6.
Consider the following statements regarding claudication:

1. It is a marker for silent coronary disease
2. Structured exercise program ( 2 hours per week for 3 months) leads to
improvement in symptoms
3. Diabetes mellitus increases the risk and severity of claudication
4. Beta blockers may exacerbate claudication

Which of the above statements are correct?

(a) 1 and 2 only
(b) 1, 3 and 4 only
(c) 2, 3 and 4 only
(d) 1, 2, 3 and 4
7.
A 50-year old lady underwent uneventful bariatric surgery for morbid obesity. On the third
post operative day, she develops breathless and pulmonary embolism is suspected. The next
investigation to confirm the diagnosis will be:

(a) Echocardiography
(b) Duplex venography
(c) CT pulmonary angiography
(d) MR angiography
8.
Medical management of thyrotoxic crisis includes all of the following EXCEPT:

(a) IV fluids
(b) IV propanolol
(c) IV hydrocortisone
(d) IV antibiotics
9.
A patient operated for a parotid gland tumour developed symptoms of sweating and
erytherma (flushing) over the region of surgical excision while eating. The probable
diagnosis is:

(a) Parotid gland fistula
(b) Sialadenitis
(c) Chronic wound infection
(d) Frey?s syndrome
10.
First line hormone therapy for post-menopausal woman with metastatic carcinoma breast is:

(a) Tamoxifen
(b) Ovarian suppression by surgery
(c) Antiprogestins
(d) Anastrazole



11.
All of the following are major subtypes of breast cancer based on Gene array analysis
EXCEPT:

(a) Luminal A and Luminal B
(b) Triple negative
(c) Her-2 receptor positive
(d) Oestrogen receptor positive
12.
All of the following are sequelae of peptic ulcer surgery EXCEPT:

(a) Bilious vomiting
(b) Dumping syndrome
(c) Diarrhoea
(d) Increased appetite
13.
The Child-Turcotte-Pugh (CTP) score for quantifying the severity of chronic liver disease
includes all variables EXCEPT:

(a) Serum bilirubin
(b) Serum albumin
(c) Serum creatinine
(d) INR (International Normalised Ratio)
14.
?Chain of Lakes? appearance due to sacculation with intervening short strictures of pancreatic
duct is seen on:

(a) ERCP
(b) CECT abdomen
(c) Plain X-ray abdomen
(d) Ultrasonography
15.
Which one of the following statements is NOT correct regarding Pyogenic Liver Abscess?

(a) Anorexia, fever, malaise and right upper quadrant abdominal discomfort are the
most common presenting features
(b) It is more common in elderly, diabetics and immunocompromised patients
(c) Treatment is with oral antibiotics alone
(d) Streptococcus milleri and escherichia coli are the most common causative
organisms
16.
?Swiss cheese defects? are seen during laparoscopic repair of:

(a) Ventral hernia
(b) Inguinal hernia
(c) Obturator hernia
(d) Femoral hernia



17.
Which of the following are correct regarding splenic artery aneurysm?

1. Main arterial trunk is the common site
2. Palpable thrill can be felt
3. It is symptomless unless it ruptures

Select the correct answer using the code given below:

(a) 1 and 2 only
(b) 2 and 3 only
(c) 1 and 3 only
(d) 1, 2 and 3
18.
Valentino?s syndrome is:

(a) Pain on per-vaginal examination in pelvic abscess
(b) Pain over left shoulder in left hypochondriac collection
(c) Pain over left groin in perirenal collection
(d) Pain in right iliac fossa in perforated peptic ulcer
19.
Spontaneous bacterial peritonitis occurs due to:

(a) duodenal stump blowout
(b) peptic ulcer perforation
(c) acute bacterial infection of ascites
(d) infection via fallopian tubes
20.
Structure not forming boundaries of the ?Triangle of doom? seen during laparoscopic
inguinal hernia surgery dissection is:

(a) Vas deferens
(b) Inferior epigastric artery
(c) Spermatic cord vessels
(d) Peritoneum
21.
The term mid-line shift is associated with:

(a) Head injury
(b) Chest injury
(c) Abdominal injury
(d) Limb injury
22.
Which one of the following cranial nerves does NOT supply to the external ear?

(a) Cranial nerve V
(b) Cranial nerve VI
(c) Cranial nerve VII
(d) Cranial nerve IX

23.
Left Internal Mammary Artery (LIMA) has become the conduit of choice for Left Anterior
Descending (LAD) artery during coronary artery bypass grafting because:

(a) Long term patency rates are more than 98%
(b) It is close to LAD
(c) Atherosclerosis is never seen in this vessel
(d) It is very easy to harvest
24.
A 50-year old male with significant smoking history presented in the surgical emergency
with sudden severe breathlessness. Chest X-ray shows right sided Pneumothorax. The
appropriate management requires:

(a) Aspiration of air with 16-18 G cannula
(b) Right chest drain of size 8-14 Fr
(c) Oxygen by face mask
(d) Mechanical ventilation
25.
Which one of the following statements regarding Felon is NOT correct?

(a) There is infection of the finger tip between specialised fibrous septa
(b) It is a painless condition
(c) Incision and drainage is the treatment of choice
(d) It is common in diabetics
26.
The most common site for osteosarcoma is:

(a) Proximal femur
(b) Distal femur
(c) Proximal humerus
(d) Distal humerus
27.
Rapid Sequence Induction is indicated in:

(a) Emergency surgery for intestinal obstruction
(b) Elective open hernia surgery
(c) Cardiopulmonary bypass surgery
(d) Elective laparoscopic surgery

28.
Which one of the following statements about Compartment Syndrome is NOT correct?

(a) It is commonest in a closed fracture
(b) Pain is on active movement but not on passive movement of muscles
(c) Fasciotomy is the treatment of choice
(d) Volkmann?s Ischaemic contractive is a late complication

FirstRanker.com - FirstRanker's Choice
Combined Medical Services Examination-2020
Paper-II

1.
Indications for fasciotomy in compartment syndrome include all EXCEPT:

(a) Distal sensory disturbance
(b) Compartment pressure > 30 mm Hg
(c) Pain on passive movement of affected muscles
(d) Palpable distal pulses
2.
Which one of the following statements is NOT correct regarding Necrotising Soft Tissue
infections?

(a) Crepitus, skin blistering and focal skin gangrene are typical presenting features
(b) They are monomicrobial in nature
(c) Treatment consists of wide local excision and appropriate antibiotics
(d) Tissue biopsy is required for culture and diagnosis
3.
Which type of surgery is laparoscopic cholecystectomy classified as?

(a) Clean
(b) Clean contaminated
(c) Contaminated
(d) Dirty
4.
A 22-year female has presented with a history of malaise, cough, alternating constipation and
diarrhoea with intermittent abdominal pain for last 6 months. She also complains of
abdominal distension for last 2 days. On examination her abdomen has a doughy feel along
with an ill defined mass over the right lower quadrant. She is most likely suffering from:

(a) Appendicular lump
(b) Ileocaecal tuberculosis
(c) Carcinoma caecum
(d) Ovarian mass
5.
Consider the following statements regarding needle stick injuries:

1. Injured part should be washed under running water
2. Dominant index finger is the commonest site for needle stick injury
3. All needle stick injuries should be reported
4. Hepatitis/HIV testing should be done after needle stick injury

Which of the statements given above are correct?

(a) 1, 2 and 4
(b) 1, 2 and 3
(c) 1, 3 and 4
(d) 2, 3 and 4




6.
Consider the following statements regarding claudication:

1. It is a marker for silent coronary disease
2. Structured exercise program ( 2 hours per week for 3 months) leads to
improvement in symptoms
3. Diabetes mellitus increases the risk and severity of claudication
4. Beta blockers may exacerbate claudication

Which of the above statements are correct?

(a) 1 and 2 only
(b) 1, 3 and 4 only
(c) 2, 3 and 4 only
(d) 1, 2, 3 and 4
7.
A 50-year old lady underwent uneventful bariatric surgery for morbid obesity. On the third
post operative day, she develops breathless and pulmonary embolism is suspected. The next
investigation to confirm the diagnosis will be:

(a) Echocardiography
(b) Duplex venography
(c) CT pulmonary angiography
(d) MR angiography
8.
Medical management of thyrotoxic crisis includes all of the following EXCEPT:

(a) IV fluids
(b) IV propanolol
(c) IV hydrocortisone
(d) IV antibiotics
9.
A patient operated for a parotid gland tumour developed symptoms of sweating and
erytherma (flushing) over the region of surgical excision while eating. The probable
diagnosis is:

(a) Parotid gland fistula
(b) Sialadenitis
(c) Chronic wound infection
(d) Frey?s syndrome
10.
First line hormone therapy for post-menopausal woman with metastatic carcinoma breast is:

(a) Tamoxifen
(b) Ovarian suppression by surgery
(c) Antiprogestins
(d) Anastrazole



11.
All of the following are major subtypes of breast cancer based on Gene array analysis
EXCEPT:

(a) Luminal A and Luminal B
(b) Triple negative
(c) Her-2 receptor positive
(d) Oestrogen receptor positive
12.
All of the following are sequelae of peptic ulcer surgery EXCEPT:

(a) Bilious vomiting
(b) Dumping syndrome
(c) Diarrhoea
(d) Increased appetite
13.
The Child-Turcotte-Pugh (CTP) score for quantifying the severity of chronic liver disease
includes all variables EXCEPT:

(a) Serum bilirubin
(b) Serum albumin
(c) Serum creatinine
(d) INR (International Normalised Ratio)
14.
?Chain of Lakes? appearance due to sacculation with intervening short strictures of pancreatic
duct is seen on:

(a) ERCP
(b) CECT abdomen
(c) Plain X-ray abdomen
(d) Ultrasonography
15.
Which one of the following statements is NOT correct regarding Pyogenic Liver Abscess?

(a) Anorexia, fever, malaise and right upper quadrant abdominal discomfort are the
most common presenting features
(b) It is more common in elderly, diabetics and immunocompromised patients
(c) Treatment is with oral antibiotics alone
(d) Streptococcus milleri and escherichia coli are the most common causative
organisms
16.
?Swiss cheese defects? are seen during laparoscopic repair of:

(a) Ventral hernia
(b) Inguinal hernia
(c) Obturator hernia
(d) Femoral hernia



17.
Which of the following are correct regarding splenic artery aneurysm?

1. Main arterial trunk is the common site
2. Palpable thrill can be felt
3. It is symptomless unless it ruptures

Select the correct answer using the code given below:

(a) 1 and 2 only
(b) 2 and 3 only
(c) 1 and 3 only
(d) 1, 2 and 3
18.
Valentino?s syndrome is:

(a) Pain on per-vaginal examination in pelvic abscess
(b) Pain over left shoulder in left hypochondriac collection
(c) Pain over left groin in perirenal collection
(d) Pain in right iliac fossa in perforated peptic ulcer
19.
Spontaneous bacterial peritonitis occurs due to:

(a) duodenal stump blowout
(b) peptic ulcer perforation
(c) acute bacterial infection of ascites
(d) infection via fallopian tubes
20.
Structure not forming boundaries of the ?Triangle of doom? seen during laparoscopic
inguinal hernia surgery dissection is:

(a) Vas deferens
(b) Inferior epigastric artery
(c) Spermatic cord vessels
(d) Peritoneum
21.
The term mid-line shift is associated with:

(a) Head injury
(b) Chest injury
(c) Abdominal injury
(d) Limb injury
22.
Which one of the following cranial nerves does NOT supply to the external ear?

(a) Cranial nerve V
(b) Cranial nerve VI
(c) Cranial nerve VII
(d) Cranial nerve IX

23.
Left Internal Mammary Artery (LIMA) has become the conduit of choice for Left Anterior
Descending (LAD) artery during coronary artery bypass grafting because:

(a) Long term patency rates are more than 98%
(b) It is close to LAD
(c) Atherosclerosis is never seen in this vessel
(d) It is very easy to harvest
24.
A 50-year old male with significant smoking history presented in the surgical emergency
with sudden severe breathlessness. Chest X-ray shows right sided Pneumothorax. The
appropriate management requires:

(a) Aspiration of air with 16-18 G cannula
(b) Right chest drain of size 8-14 Fr
(c) Oxygen by face mask
(d) Mechanical ventilation
25.
Which one of the following statements regarding Felon is NOT correct?

(a) There is infection of the finger tip between specialised fibrous septa
(b) It is a painless condition
(c) Incision and drainage is the treatment of choice
(d) It is common in diabetics
26.
The most common site for osteosarcoma is:

(a) Proximal femur
(b) Distal femur
(c) Proximal humerus
(d) Distal humerus
27.
Rapid Sequence Induction is indicated in:

(a) Emergency surgery for intestinal obstruction
(b) Elective open hernia surgery
(c) Cardiopulmonary bypass surgery
(d) Elective laparoscopic surgery

28.
Which one of the following statements about Compartment Syndrome is NOT correct?

(a) It is commonest in a closed fracture
(b) Pain is on active movement but not on passive movement of muscles
(c) Fasciotomy is the treatment of choice
(d) Volkmann?s Ischaemic contractive is a late complication




29.
Which one of the following is NOT the strength of ultrasound as a diagnostic modality?

(a) No radiation
(b) Short learning curve
(c) Inexpensive
(d) Allows dynamic studies to be done
30.
Which of the following statements regarding lymphoedema following breast cancer treatment
are correct?

1. Incidence has decreased due to rarely combined therapy of axillary LN dissection and
radiotherapy
2. Precipitating cause like LN metastasis is a major determinant
3. The condition is often painful
4. Oedematous limb is susceptible to bacterial infection

Select the correct answer using the code given below:

(a) 1, 2 and 3
(b) 2, 3 and 4
(c) 1, 3 and 4
(d) 1, 2 and 4
31.
Which one of the following is NOT a risk factor for development of venous thrombosis in
surgical patients?

(a) Age > 60 years
(b) Pregnancy
(c) Obesity (BMI > 30 kg/m
2
)
(d) Diabetes ( HbA1c > 7.5%)
32.
Which one of the following is NOT a complication of massive blood transfusion?

(a) Coagulopathy
(b) Hypocalcaemia
(c) Hyperthermia
(d) Hyperkalemia
33.
Stage III ?Pressure sore? is full thickness skin loss extending:

(a) into subcutaneous tissue but not through fascia
(b) through subcutaneous tissue into fascia
(c) through subcutaneous tissue into fascia and muscles
(d) through subcutaneous tissue into fascia, muscles and bone


FirstRanker.com - FirstRanker's Choice
Combined Medical Services Examination-2020
Paper-II

1.
Indications for fasciotomy in compartment syndrome include all EXCEPT:

(a) Distal sensory disturbance
(b) Compartment pressure > 30 mm Hg
(c) Pain on passive movement of affected muscles
(d) Palpable distal pulses
2.
Which one of the following statements is NOT correct regarding Necrotising Soft Tissue
infections?

(a) Crepitus, skin blistering and focal skin gangrene are typical presenting features
(b) They are monomicrobial in nature
(c) Treatment consists of wide local excision and appropriate antibiotics
(d) Tissue biopsy is required for culture and diagnosis
3.
Which type of surgery is laparoscopic cholecystectomy classified as?

(a) Clean
(b) Clean contaminated
(c) Contaminated
(d) Dirty
4.
A 22-year female has presented with a history of malaise, cough, alternating constipation and
diarrhoea with intermittent abdominal pain for last 6 months. She also complains of
abdominal distension for last 2 days. On examination her abdomen has a doughy feel along
with an ill defined mass over the right lower quadrant. She is most likely suffering from:

(a) Appendicular lump
(b) Ileocaecal tuberculosis
(c) Carcinoma caecum
(d) Ovarian mass
5.
Consider the following statements regarding needle stick injuries:

1. Injured part should be washed under running water
2. Dominant index finger is the commonest site for needle stick injury
3. All needle stick injuries should be reported
4. Hepatitis/HIV testing should be done after needle stick injury

Which of the statements given above are correct?

(a) 1, 2 and 4
(b) 1, 2 and 3
(c) 1, 3 and 4
(d) 2, 3 and 4




6.
Consider the following statements regarding claudication:

1. It is a marker for silent coronary disease
2. Structured exercise program ( 2 hours per week for 3 months) leads to
improvement in symptoms
3. Diabetes mellitus increases the risk and severity of claudication
4. Beta blockers may exacerbate claudication

Which of the above statements are correct?

(a) 1 and 2 only
(b) 1, 3 and 4 only
(c) 2, 3 and 4 only
(d) 1, 2, 3 and 4
7.
A 50-year old lady underwent uneventful bariatric surgery for morbid obesity. On the third
post operative day, she develops breathless and pulmonary embolism is suspected. The next
investigation to confirm the diagnosis will be:

(a) Echocardiography
(b) Duplex venography
(c) CT pulmonary angiography
(d) MR angiography
8.
Medical management of thyrotoxic crisis includes all of the following EXCEPT:

(a) IV fluids
(b) IV propanolol
(c) IV hydrocortisone
(d) IV antibiotics
9.
A patient operated for a parotid gland tumour developed symptoms of sweating and
erytherma (flushing) over the region of surgical excision while eating. The probable
diagnosis is:

(a) Parotid gland fistula
(b) Sialadenitis
(c) Chronic wound infection
(d) Frey?s syndrome
10.
First line hormone therapy for post-menopausal woman with metastatic carcinoma breast is:

(a) Tamoxifen
(b) Ovarian suppression by surgery
(c) Antiprogestins
(d) Anastrazole



11.
All of the following are major subtypes of breast cancer based on Gene array analysis
EXCEPT:

(a) Luminal A and Luminal B
(b) Triple negative
(c) Her-2 receptor positive
(d) Oestrogen receptor positive
12.
All of the following are sequelae of peptic ulcer surgery EXCEPT:

(a) Bilious vomiting
(b) Dumping syndrome
(c) Diarrhoea
(d) Increased appetite
13.
The Child-Turcotte-Pugh (CTP) score for quantifying the severity of chronic liver disease
includes all variables EXCEPT:

(a) Serum bilirubin
(b) Serum albumin
(c) Serum creatinine
(d) INR (International Normalised Ratio)
14.
?Chain of Lakes? appearance due to sacculation with intervening short strictures of pancreatic
duct is seen on:

(a) ERCP
(b) CECT abdomen
(c) Plain X-ray abdomen
(d) Ultrasonography
15.
Which one of the following statements is NOT correct regarding Pyogenic Liver Abscess?

(a) Anorexia, fever, malaise and right upper quadrant abdominal discomfort are the
most common presenting features
(b) It is more common in elderly, diabetics and immunocompromised patients
(c) Treatment is with oral antibiotics alone
(d) Streptococcus milleri and escherichia coli are the most common causative
organisms
16.
?Swiss cheese defects? are seen during laparoscopic repair of:

(a) Ventral hernia
(b) Inguinal hernia
(c) Obturator hernia
(d) Femoral hernia



17.
Which of the following are correct regarding splenic artery aneurysm?

1. Main arterial trunk is the common site
2. Palpable thrill can be felt
3. It is symptomless unless it ruptures

Select the correct answer using the code given below:

(a) 1 and 2 only
(b) 2 and 3 only
(c) 1 and 3 only
(d) 1, 2 and 3
18.
Valentino?s syndrome is:

(a) Pain on per-vaginal examination in pelvic abscess
(b) Pain over left shoulder in left hypochondriac collection
(c) Pain over left groin in perirenal collection
(d) Pain in right iliac fossa in perforated peptic ulcer
19.
Spontaneous bacterial peritonitis occurs due to:

(a) duodenal stump blowout
(b) peptic ulcer perforation
(c) acute bacterial infection of ascites
(d) infection via fallopian tubes
20.
Structure not forming boundaries of the ?Triangle of doom? seen during laparoscopic
inguinal hernia surgery dissection is:

(a) Vas deferens
(b) Inferior epigastric artery
(c) Spermatic cord vessels
(d) Peritoneum
21.
The term mid-line shift is associated with:

(a) Head injury
(b) Chest injury
(c) Abdominal injury
(d) Limb injury
22.
Which one of the following cranial nerves does NOT supply to the external ear?

(a) Cranial nerve V
(b) Cranial nerve VI
(c) Cranial nerve VII
(d) Cranial nerve IX

23.
Left Internal Mammary Artery (LIMA) has become the conduit of choice for Left Anterior
Descending (LAD) artery during coronary artery bypass grafting because:

(a) Long term patency rates are more than 98%
(b) It is close to LAD
(c) Atherosclerosis is never seen in this vessel
(d) It is very easy to harvest
24.
A 50-year old male with significant smoking history presented in the surgical emergency
with sudden severe breathlessness. Chest X-ray shows right sided Pneumothorax. The
appropriate management requires:

(a) Aspiration of air with 16-18 G cannula
(b) Right chest drain of size 8-14 Fr
(c) Oxygen by face mask
(d) Mechanical ventilation
25.
Which one of the following statements regarding Felon is NOT correct?

(a) There is infection of the finger tip between specialised fibrous septa
(b) It is a painless condition
(c) Incision and drainage is the treatment of choice
(d) It is common in diabetics
26.
The most common site for osteosarcoma is:

(a) Proximal femur
(b) Distal femur
(c) Proximal humerus
(d) Distal humerus
27.
Rapid Sequence Induction is indicated in:

(a) Emergency surgery for intestinal obstruction
(b) Elective open hernia surgery
(c) Cardiopulmonary bypass surgery
(d) Elective laparoscopic surgery

28.
Which one of the following statements about Compartment Syndrome is NOT correct?

(a) It is commonest in a closed fracture
(b) Pain is on active movement but not on passive movement of muscles
(c) Fasciotomy is the treatment of choice
(d) Volkmann?s Ischaemic contractive is a late complication




29.
Which one of the following is NOT the strength of ultrasound as a diagnostic modality?

(a) No radiation
(b) Short learning curve
(c) Inexpensive
(d) Allows dynamic studies to be done
30.
Which of the following statements regarding lymphoedema following breast cancer treatment
are correct?

1. Incidence has decreased due to rarely combined therapy of axillary LN dissection and
radiotherapy
2. Precipitating cause like LN metastasis is a major determinant
3. The condition is often painful
4. Oedematous limb is susceptible to bacterial infection

Select the correct answer using the code given below:

(a) 1, 2 and 3
(b) 2, 3 and 4
(c) 1, 3 and 4
(d) 1, 2 and 4
31.
Which one of the following is NOT a risk factor for development of venous thrombosis in
surgical patients?

(a) Age > 60 years
(b) Pregnancy
(c) Obesity (BMI > 30 kg/m
2
)
(d) Diabetes ( HbA1c > 7.5%)
32.
Which one of the following is NOT a complication of massive blood transfusion?

(a) Coagulopathy
(b) Hypocalcaemia
(c) Hyperthermia
(d) Hyperkalemia
33.
Stage III ?Pressure sore? is full thickness skin loss extending:

(a) into subcutaneous tissue but not through fascia
(b) through subcutaneous tissue into fascia
(c) through subcutaneous tissue into fascia and muscles
(d) through subcutaneous tissue into fascia, muscles and bone



34.
During subclavian vein puncture in a surgical ward suddenly a patient developed severe
breathlessness. On auscultation breath sound was absent and the ipsilateral chest was
tympanitic on percussion. The probable diagnosis is:

(a) Iatrogenic pneumothorax
(b) Introgenic hemothorax
(c) Spontaneous pneumothorax
(d) Tension pneumothorax
35.
Kohler?s disease is avascular necrosis of :

(a) Lunate
(b) Capitellum of humerus
(c) First metatarsal head
(d) Navicular
36.
Which one of the following is NOT true of Pyoderma gangrenosum?

(a) It is characterized by cutaneous ulceration with purple undermined edges
(b) It is often secondary to heightened immunological reactivity from another disease
process
(c) Cultures often show Gram positive Staphylococci
(d) Lesions generally respond to steroids
37.
Which of the following is NOT a tissue repair surgery for inguinal hernia repair?

(a) Bassini?s repair
(b) Shouldice repair
(c) Stoppa?s repair
(d) Desarda repair
38.
Which one of the following type of meshes is recommended for intraperitoneal use in
abdominal wall hernia?
(a) Light weight, porous meshes
(b) Heavy weight, porous meshes
(c) Absorbable meshes
(d) Tissue separating meshes
39.
Which one of the following is NOT correct regarding MEN-1 syndrome?

(a) It involves parathyroid glands
(b) It involves pancreas
(c) It involves pituitary gland
(d) It involves pineal gland

FirstRanker.com - FirstRanker's Choice
Combined Medical Services Examination-2020
Paper-II

1.
Indications for fasciotomy in compartment syndrome include all EXCEPT:

(a) Distal sensory disturbance
(b) Compartment pressure > 30 mm Hg
(c) Pain on passive movement of affected muscles
(d) Palpable distal pulses
2.
Which one of the following statements is NOT correct regarding Necrotising Soft Tissue
infections?

(a) Crepitus, skin blistering and focal skin gangrene are typical presenting features
(b) They are monomicrobial in nature
(c) Treatment consists of wide local excision and appropriate antibiotics
(d) Tissue biopsy is required for culture and diagnosis
3.
Which type of surgery is laparoscopic cholecystectomy classified as?

(a) Clean
(b) Clean contaminated
(c) Contaminated
(d) Dirty
4.
A 22-year female has presented with a history of malaise, cough, alternating constipation and
diarrhoea with intermittent abdominal pain for last 6 months. She also complains of
abdominal distension for last 2 days. On examination her abdomen has a doughy feel along
with an ill defined mass over the right lower quadrant. She is most likely suffering from:

(a) Appendicular lump
(b) Ileocaecal tuberculosis
(c) Carcinoma caecum
(d) Ovarian mass
5.
Consider the following statements regarding needle stick injuries:

1. Injured part should be washed under running water
2. Dominant index finger is the commonest site for needle stick injury
3. All needle stick injuries should be reported
4. Hepatitis/HIV testing should be done after needle stick injury

Which of the statements given above are correct?

(a) 1, 2 and 4
(b) 1, 2 and 3
(c) 1, 3 and 4
(d) 2, 3 and 4




6.
Consider the following statements regarding claudication:

1. It is a marker for silent coronary disease
2. Structured exercise program ( 2 hours per week for 3 months) leads to
improvement in symptoms
3. Diabetes mellitus increases the risk and severity of claudication
4. Beta blockers may exacerbate claudication

Which of the above statements are correct?

(a) 1 and 2 only
(b) 1, 3 and 4 only
(c) 2, 3 and 4 only
(d) 1, 2, 3 and 4
7.
A 50-year old lady underwent uneventful bariatric surgery for morbid obesity. On the third
post operative day, she develops breathless and pulmonary embolism is suspected. The next
investigation to confirm the diagnosis will be:

(a) Echocardiography
(b) Duplex venography
(c) CT pulmonary angiography
(d) MR angiography
8.
Medical management of thyrotoxic crisis includes all of the following EXCEPT:

(a) IV fluids
(b) IV propanolol
(c) IV hydrocortisone
(d) IV antibiotics
9.
A patient operated for a parotid gland tumour developed symptoms of sweating and
erytherma (flushing) over the region of surgical excision while eating. The probable
diagnosis is:

(a) Parotid gland fistula
(b) Sialadenitis
(c) Chronic wound infection
(d) Frey?s syndrome
10.
First line hormone therapy for post-menopausal woman with metastatic carcinoma breast is:

(a) Tamoxifen
(b) Ovarian suppression by surgery
(c) Antiprogestins
(d) Anastrazole



11.
All of the following are major subtypes of breast cancer based on Gene array analysis
EXCEPT:

(a) Luminal A and Luminal B
(b) Triple negative
(c) Her-2 receptor positive
(d) Oestrogen receptor positive
12.
All of the following are sequelae of peptic ulcer surgery EXCEPT:

(a) Bilious vomiting
(b) Dumping syndrome
(c) Diarrhoea
(d) Increased appetite
13.
The Child-Turcotte-Pugh (CTP) score for quantifying the severity of chronic liver disease
includes all variables EXCEPT:

(a) Serum bilirubin
(b) Serum albumin
(c) Serum creatinine
(d) INR (International Normalised Ratio)
14.
?Chain of Lakes? appearance due to sacculation with intervening short strictures of pancreatic
duct is seen on:

(a) ERCP
(b) CECT abdomen
(c) Plain X-ray abdomen
(d) Ultrasonography
15.
Which one of the following statements is NOT correct regarding Pyogenic Liver Abscess?

(a) Anorexia, fever, malaise and right upper quadrant abdominal discomfort are the
most common presenting features
(b) It is more common in elderly, diabetics and immunocompromised patients
(c) Treatment is with oral antibiotics alone
(d) Streptococcus milleri and escherichia coli are the most common causative
organisms
16.
?Swiss cheese defects? are seen during laparoscopic repair of:

(a) Ventral hernia
(b) Inguinal hernia
(c) Obturator hernia
(d) Femoral hernia



17.
Which of the following are correct regarding splenic artery aneurysm?

1. Main arterial trunk is the common site
2. Palpable thrill can be felt
3. It is symptomless unless it ruptures

Select the correct answer using the code given below:

(a) 1 and 2 only
(b) 2 and 3 only
(c) 1 and 3 only
(d) 1, 2 and 3
18.
Valentino?s syndrome is:

(a) Pain on per-vaginal examination in pelvic abscess
(b) Pain over left shoulder in left hypochondriac collection
(c) Pain over left groin in perirenal collection
(d) Pain in right iliac fossa in perforated peptic ulcer
19.
Spontaneous bacterial peritonitis occurs due to:

(a) duodenal stump blowout
(b) peptic ulcer perforation
(c) acute bacterial infection of ascites
(d) infection via fallopian tubes
20.
Structure not forming boundaries of the ?Triangle of doom? seen during laparoscopic
inguinal hernia surgery dissection is:

(a) Vas deferens
(b) Inferior epigastric artery
(c) Spermatic cord vessels
(d) Peritoneum
21.
The term mid-line shift is associated with:

(a) Head injury
(b) Chest injury
(c) Abdominal injury
(d) Limb injury
22.
Which one of the following cranial nerves does NOT supply to the external ear?

(a) Cranial nerve V
(b) Cranial nerve VI
(c) Cranial nerve VII
(d) Cranial nerve IX

23.
Left Internal Mammary Artery (LIMA) has become the conduit of choice for Left Anterior
Descending (LAD) artery during coronary artery bypass grafting because:

(a) Long term patency rates are more than 98%
(b) It is close to LAD
(c) Atherosclerosis is never seen in this vessel
(d) It is very easy to harvest
24.
A 50-year old male with significant smoking history presented in the surgical emergency
with sudden severe breathlessness. Chest X-ray shows right sided Pneumothorax. The
appropriate management requires:

(a) Aspiration of air with 16-18 G cannula
(b) Right chest drain of size 8-14 Fr
(c) Oxygen by face mask
(d) Mechanical ventilation
25.
Which one of the following statements regarding Felon is NOT correct?

(a) There is infection of the finger tip between specialised fibrous septa
(b) It is a painless condition
(c) Incision and drainage is the treatment of choice
(d) It is common in diabetics
26.
The most common site for osteosarcoma is:

(a) Proximal femur
(b) Distal femur
(c) Proximal humerus
(d) Distal humerus
27.
Rapid Sequence Induction is indicated in:

(a) Emergency surgery for intestinal obstruction
(b) Elective open hernia surgery
(c) Cardiopulmonary bypass surgery
(d) Elective laparoscopic surgery

28.
Which one of the following statements about Compartment Syndrome is NOT correct?

(a) It is commonest in a closed fracture
(b) Pain is on active movement but not on passive movement of muscles
(c) Fasciotomy is the treatment of choice
(d) Volkmann?s Ischaemic contractive is a late complication




29.
Which one of the following is NOT the strength of ultrasound as a diagnostic modality?

(a) No radiation
(b) Short learning curve
(c) Inexpensive
(d) Allows dynamic studies to be done
30.
Which of the following statements regarding lymphoedema following breast cancer treatment
are correct?

1. Incidence has decreased due to rarely combined therapy of axillary LN dissection and
radiotherapy
2. Precipitating cause like LN metastasis is a major determinant
3. The condition is often painful
4. Oedematous limb is susceptible to bacterial infection

Select the correct answer using the code given below:

(a) 1, 2 and 3
(b) 2, 3 and 4
(c) 1, 3 and 4
(d) 1, 2 and 4
31.
Which one of the following is NOT a risk factor for development of venous thrombosis in
surgical patients?

(a) Age > 60 years
(b) Pregnancy
(c) Obesity (BMI > 30 kg/m
2
)
(d) Diabetes ( HbA1c > 7.5%)
32.
Which one of the following is NOT a complication of massive blood transfusion?

(a) Coagulopathy
(b) Hypocalcaemia
(c) Hyperthermia
(d) Hyperkalemia
33.
Stage III ?Pressure sore? is full thickness skin loss extending:

(a) into subcutaneous tissue but not through fascia
(b) through subcutaneous tissue into fascia
(c) through subcutaneous tissue into fascia and muscles
(d) through subcutaneous tissue into fascia, muscles and bone



34.
During subclavian vein puncture in a surgical ward suddenly a patient developed severe
breathlessness. On auscultation breath sound was absent and the ipsilateral chest was
tympanitic on percussion. The probable diagnosis is:

(a) Iatrogenic pneumothorax
(b) Introgenic hemothorax
(c) Spontaneous pneumothorax
(d) Tension pneumothorax
35.
Kohler?s disease is avascular necrosis of :

(a) Lunate
(b) Capitellum of humerus
(c) First metatarsal head
(d) Navicular
36.
Which one of the following is NOT true of Pyoderma gangrenosum?

(a) It is characterized by cutaneous ulceration with purple undermined edges
(b) It is often secondary to heightened immunological reactivity from another disease
process
(c) Cultures often show Gram positive Staphylococci
(d) Lesions generally respond to steroids
37.
Which of the following is NOT a tissue repair surgery for inguinal hernia repair?

(a) Bassini?s repair
(b) Shouldice repair
(c) Stoppa?s repair
(d) Desarda repair
38.
Which one of the following type of meshes is recommended for intraperitoneal use in
abdominal wall hernia?
(a) Light weight, porous meshes
(b) Heavy weight, porous meshes
(c) Absorbable meshes
(d) Tissue separating meshes
39.
Which one of the following is NOT correct regarding MEN-1 syndrome?

(a) It involves parathyroid glands
(b) It involves pancreas
(c) It involves pituitary gland
(d) It involves pineal gland

40.
Which one of the following is NOT an electronic information site in surgery?

(a) Pubmed
(b) Embase
(c) Cochrane library
(d) National medical library
41.
Which one of the following statements regarding pre-conceptional counseling is NOT
correct?

(a) It is needed only in selected complicated pregnancies
(b) It helps in early detection of risk factors
(c) It helps in reducing maternal morbidity and mortality
(d) It is a part of preventive medicine
42.
Consider the following statements regarding Non Stress Test (NST):

1. Reactive NST indicates a healthy fetus
2. NST is an observed association of fetal breathing with fetal movements
3. NST has a low false negative rate (< 1%) but high false positive rate (>50%)
4. Testing should be started at 20 weeks

Which of the statement(s) given above is/are correct?

(a) 1 and 3
(b) 2 only
(c) 3 only
(d) 1 and 4
43.
Which one of the following is a protective factor for endometrial hyperplasia?

(a) Diabetes
(b) Tamoxifen therapy
(c) Multiparity
(d) Delayed menopause
44.
A woman who is not breast feeding her newborn child is advised to use a contraceptive
method by:

(a) 3
rd
postpartum week
(b) 6
th
postpartum week
(c) 3
rd
postpartum month
(d) 6
th
postpartum month





FirstRanker.com - FirstRanker's Choice
Combined Medical Services Examination-2020
Paper-II

1.
Indications for fasciotomy in compartment syndrome include all EXCEPT:

(a) Distal sensory disturbance
(b) Compartment pressure > 30 mm Hg
(c) Pain on passive movement of affected muscles
(d) Palpable distal pulses
2.
Which one of the following statements is NOT correct regarding Necrotising Soft Tissue
infections?

(a) Crepitus, skin blistering and focal skin gangrene are typical presenting features
(b) They are monomicrobial in nature
(c) Treatment consists of wide local excision and appropriate antibiotics
(d) Tissue biopsy is required for culture and diagnosis
3.
Which type of surgery is laparoscopic cholecystectomy classified as?

(a) Clean
(b) Clean contaminated
(c) Contaminated
(d) Dirty
4.
A 22-year female has presented with a history of malaise, cough, alternating constipation and
diarrhoea with intermittent abdominal pain for last 6 months. She also complains of
abdominal distension for last 2 days. On examination her abdomen has a doughy feel along
with an ill defined mass over the right lower quadrant. She is most likely suffering from:

(a) Appendicular lump
(b) Ileocaecal tuberculosis
(c) Carcinoma caecum
(d) Ovarian mass
5.
Consider the following statements regarding needle stick injuries:

1. Injured part should be washed under running water
2. Dominant index finger is the commonest site for needle stick injury
3. All needle stick injuries should be reported
4. Hepatitis/HIV testing should be done after needle stick injury

Which of the statements given above are correct?

(a) 1, 2 and 4
(b) 1, 2 and 3
(c) 1, 3 and 4
(d) 2, 3 and 4




6.
Consider the following statements regarding claudication:

1. It is a marker for silent coronary disease
2. Structured exercise program ( 2 hours per week for 3 months) leads to
improvement in symptoms
3. Diabetes mellitus increases the risk and severity of claudication
4. Beta blockers may exacerbate claudication

Which of the above statements are correct?

(a) 1 and 2 only
(b) 1, 3 and 4 only
(c) 2, 3 and 4 only
(d) 1, 2, 3 and 4
7.
A 50-year old lady underwent uneventful bariatric surgery for morbid obesity. On the third
post operative day, she develops breathless and pulmonary embolism is suspected. The next
investigation to confirm the diagnosis will be:

(a) Echocardiography
(b) Duplex venography
(c) CT pulmonary angiography
(d) MR angiography
8.
Medical management of thyrotoxic crisis includes all of the following EXCEPT:

(a) IV fluids
(b) IV propanolol
(c) IV hydrocortisone
(d) IV antibiotics
9.
A patient operated for a parotid gland tumour developed symptoms of sweating and
erytherma (flushing) over the region of surgical excision while eating. The probable
diagnosis is:

(a) Parotid gland fistula
(b) Sialadenitis
(c) Chronic wound infection
(d) Frey?s syndrome
10.
First line hormone therapy for post-menopausal woman with metastatic carcinoma breast is:

(a) Tamoxifen
(b) Ovarian suppression by surgery
(c) Antiprogestins
(d) Anastrazole



11.
All of the following are major subtypes of breast cancer based on Gene array analysis
EXCEPT:

(a) Luminal A and Luminal B
(b) Triple negative
(c) Her-2 receptor positive
(d) Oestrogen receptor positive
12.
All of the following are sequelae of peptic ulcer surgery EXCEPT:

(a) Bilious vomiting
(b) Dumping syndrome
(c) Diarrhoea
(d) Increased appetite
13.
The Child-Turcotte-Pugh (CTP) score for quantifying the severity of chronic liver disease
includes all variables EXCEPT:

(a) Serum bilirubin
(b) Serum albumin
(c) Serum creatinine
(d) INR (International Normalised Ratio)
14.
?Chain of Lakes? appearance due to sacculation with intervening short strictures of pancreatic
duct is seen on:

(a) ERCP
(b) CECT abdomen
(c) Plain X-ray abdomen
(d) Ultrasonography
15.
Which one of the following statements is NOT correct regarding Pyogenic Liver Abscess?

(a) Anorexia, fever, malaise and right upper quadrant abdominal discomfort are the
most common presenting features
(b) It is more common in elderly, diabetics and immunocompromised patients
(c) Treatment is with oral antibiotics alone
(d) Streptococcus milleri and escherichia coli are the most common causative
organisms
16.
?Swiss cheese defects? are seen during laparoscopic repair of:

(a) Ventral hernia
(b) Inguinal hernia
(c) Obturator hernia
(d) Femoral hernia



17.
Which of the following are correct regarding splenic artery aneurysm?

1. Main arterial trunk is the common site
2. Palpable thrill can be felt
3. It is symptomless unless it ruptures

Select the correct answer using the code given below:

(a) 1 and 2 only
(b) 2 and 3 only
(c) 1 and 3 only
(d) 1, 2 and 3
18.
Valentino?s syndrome is:

(a) Pain on per-vaginal examination in pelvic abscess
(b) Pain over left shoulder in left hypochondriac collection
(c) Pain over left groin in perirenal collection
(d) Pain in right iliac fossa in perforated peptic ulcer
19.
Spontaneous bacterial peritonitis occurs due to:

(a) duodenal stump blowout
(b) peptic ulcer perforation
(c) acute bacterial infection of ascites
(d) infection via fallopian tubes
20.
Structure not forming boundaries of the ?Triangle of doom? seen during laparoscopic
inguinal hernia surgery dissection is:

(a) Vas deferens
(b) Inferior epigastric artery
(c) Spermatic cord vessels
(d) Peritoneum
21.
The term mid-line shift is associated with:

(a) Head injury
(b) Chest injury
(c) Abdominal injury
(d) Limb injury
22.
Which one of the following cranial nerves does NOT supply to the external ear?

(a) Cranial nerve V
(b) Cranial nerve VI
(c) Cranial nerve VII
(d) Cranial nerve IX

23.
Left Internal Mammary Artery (LIMA) has become the conduit of choice for Left Anterior
Descending (LAD) artery during coronary artery bypass grafting because:

(a) Long term patency rates are more than 98%
(b) It is close to LAD
(c) Atherosclerosis is never seen in this vessel
(d) It is very easy to harvest
24.
A 50-year old male with significant smoking history presented in the surgical emergency
with sudden severe breathlessness. Chest X-ray shows right sided Pneumothorax. The
appropriate management requires:

(a) Aspiration of air with 16-18 G cannula
(b) Right chest drain of size 8-14 Fr
(c) Oxygen by face mask
(d) Mechanical ventilation
25.
Which one of the following statements regarding Felon is NOT correct?

(a) There is infection of the finger tip between specialised fibrous septa
(b) It is a painless condition
(c) Incision and drainage is the treatment of choice
(d) It is common in diabetics
26.
The most common site for osteosarcoma is:

(a) Proximal femur
(b) Distal femur
(c) Proximal humerus
(d) Distal humerus
27.
Rapid Sequence Induction is indicated in:

(a) Emergency surgery for intestinal obstruction
(b) Elective open hernia surgery
(c) Cardiopulmonary bypass surgery
(d) Elective laparoscopic surgery

28.
Which one of the following statements about Compartment Syndrome is NOT correct?

(a) It is commonest in a closed fracture
(b) Pain is on active movement but not on passive movement of muscles
(c) Fasciotomy is the treatment of choice
(d) Volkmann?s Ischaemic contractive is a late complication




29.
Which one of the following is NOT the strength of ultrasound as a diagnostic modality?

(a) No radiation
(b) Short learning curve
(c) Inexpensive
(d) Allows dynamic studies to be done
30.
Which of the following statements regarding lymphoedema following breast cancer treatment
are correct?

1. Incidence has decreased due to rarely combined therapy of axillary LN dissection and
radiotherapy
2. Precipitating cause like LN metastasis is a major determinant
3. The condition is often painful
4. Oedematous limb is susceptible to bacterial infection

Select the correct answer using the code given below:

(a) 1, 2 and 3
(b) 2, 3 and 4
(c) 1, 3 and 4
(d) 1, 2 and 4
31.
Which one of the following is NOT a risk factor for development of venous thrombosis in
surgical patients?

(a) Age > 60 years
(b) Pregnancy
(c) Obesity (BMI > 30 kg/m
2
)
(d) Diabetes ( HbA1c > 7.5%)
32.
Which one of the following is NOT a complication of massive blood transfusion?

(a) Coagulopathy
(b) Hypocalcaemia
(c) Hyperthermia
(d) Hyperkalemia
33.
Stage III ?Pressure sore? is full thickness skin loss extending:

(a) into subcutaneous tissue but not through fascia
(b) through subcutaneous tissue into fascia
(c) through subcutaneous tissue into fascia and muscles
(d) through subcutaneous tissue into fascia, muscles and bone



34.
During subclavian vein puncture in a surgical ward suddenly a patient developed severe
breathlessness. On auscultation breath sound was absent and the ipsilateral chest was
tympanitic on percussion. The probable diagnosis is:

(a) Iatrogenic pneumothorax
(b) Introgenic hemothorax
(c) Spontaneous pneumothorax
(d) Tension pneumothorax
35.
Kohler?s disease is avascular necrosis of :

(a) Lunate
(b) Capitellum of humerus
(c) First metatarsal head
(d) Navicular
36.
Which one of the following is NOT true of Pyoderma gangrenosum?

(a) It is characterized by cutaneous ulceration with purple undermined edges
(b) It is often secondary to heightened immunological reactivity from another disease
process
(c) Cultures often show Gram positive Staphylococci
(d) Lesions generally respond to steroids
37.
Which of the following is NOT a tissue repair surgery for inguinal hernia repair?

(a) Bassini?s repair
(b) Shouldice repair
(c) Stoppa?s repair
(d) Desarda repair
38.
Which one of the following type of meshes is recommended for intraperitoneal use in
abdominal wall hernia?
(a) Light weight, porous meshes
(b) Heavy weight, porous meshes
(c) Absorbable meshes
(d) Tissue separating meshes
39.
Which one of the following is NOT correct regarding MEN-1 syndrome?

(a) It involves parathyroid glands
(b) It involves pancreas
(c) It involves pituitary gland
(d) It involves pineal gland

40.
Which one of the following is NOT an electronic information site in surgery?

(a) Pubmed
(b) Embase
(c) Cochrane library
(d) National medical library
41.
Which one of the following statements regarding pre-conceptional counseling is NOT
correct?

(a) It is needed only in selected complicated pregnancies
(b) It helps in early detection of risk factors
(c) It helps in reducing maternal morbidity and mortality
(d) It is a part of preventive medicine
42.
Consider the following statements regarding Non Stress Test (NST):

1. Reactive NST indicates a healthy fetus
2. NST is an observed association of fetal breathing with fetal movements
3. NST has a low false negative rate (< 1%) but high false positive rate (>50%)
4. Testing should be started at 20 weeks

Which of the statement(s) given above is/are correct?

(a) 1 and 3
(b) 2 only
(c) 3 only
(d) 1 and 4
43.
Which one of the following is a protective factor for endometrial hyperplasia?

(a) Diabetes
(b) Tamoxifen therapy
(c) Multiparity
(d) Delayed menopause
44.
A woman who is not breast feeding her newborn child is advised to use a contraceptive
method by:

(a) 3
rd
postpartum week
(b) 6
th
postpartum week
(c) 3
rd
postpartum month
(d) 6
th
postpartum month





45.
Pearl index for contraceptive effectiveness is calculated in terms of which of the following?

1. Pregnancy rate
2. Abortion rate
3. Hundred woman years
4. Thousand woman years

Select the correct answer using the code given below:

(a) 1 only
(b) 2 and 3
(c) 1, 2 and 4
(d) 1 and 3
46.
Indications for removal of IUDs are all EXCEPT:

(a) Perforation of uterus
(b) Cyclical menstrual bleeding
(c) Flaring up of salpingitis
(d) Pregnancy with IUD
47.
Contraindications for insertion of IUDs are all EXCEPT:

(a) Suspected pregnancy
(b) Trophoblastic disease
(c) Severe dysmenorrhea
(d) During cesarean section
48.
Which one of the following is NOT a contraindication for use of Mini pill?

(a) Pregnancy
(b) Breast feeding
(c) Thromboembolic disease
(d) History of breast cancer
49.
Which one of the following is the most commonly used surgical method/technique of female
sterilization as recommended by Government of India?

(a) Uchida technique
(b) Irving method
(c) Pomeroy?s method
(d) Madlener technique







FirstRanker.com - FirstRanker's Choice
Combined Medical Services Examination-2020
Paper-II

1.
Indications for fasciotomy in compartment syndrome include all EXCEPT:

(a) Distal sensory disturbance
(b) Compartment pressure > 30 mm Hg
(c) Pain on passive movement of affected muscles
(d) Palpable distal pulses
2.
Which one of the following statements is NOT correct regarding Necrotising Soft Tissue
infections?

(a) Crepitus, skin blistering and focal skin gangrene are typical presenting features
(b) They are monomicrobial in nature
(c) Treatment consists of wide local excision and appropriate antibiotics
(d) Tissue biopsy is required for culture and diagnosis
3.
Which type of surgery is laparoscopic cholecystectomy classified as?

(a) Clean
(b) Clean contaminated
(c) Contaminated
(d) Dirty
4.
A 22-year female has presented with a history of malaise, cough, alternating constipation and
diarrhoea with intermittent abdominal pain for last 6 months. She also complains of
abdominal distension for last 2 days. On examination her abdomen has a doughy feel along
with an ill defined mass over the right lower quadrant. She is most likely suffering from:

(a) Appendicular lump
(b) Ileocaecal tuberculosis
(c) Carcinoma caecum
(d) Ovarian mass
5.
Consider the following statements regarding needle stick injuries:

1. Injured part should be washed under running water
2. Dominant index finger is the commonest site for needle stick injury
3. All needle stick injuries should be reported
4. Hepatitis/HIV testing should be done after needle stick injury

Which of the statements given above are correct?

(a) 1, 2 and 4
(b) 1, 2 and 3
(c) 1, 3 and 4
(d) 2, 3 and 4




6.
Consider the following statements regarding claudication:

1. It is a marker for silent coronary disease
2. Structured exercise program ( 2 hours per week for 3 months) leads to
improvement in symptoms
3. Diabetes mellitus increases the risk and severity of claudication
4. Beta blockers may exacerbate claudication

Which of the above statements are correct?

(a) 1 and 2 only
(b) 1, 3 and 4 only
(c) 2, 3 and 4 only
(d) 1, 2, 3 and 4
7.
A 50-year old lady underwent uneventful bariatric surgery for morbid obesity. On the third
post operative day, she develops breathless and pulmonary embolism is suspected. The next
investigation to confirm the diagnosis will be:

(a) Echocardiography
(b) Duplex venography
(c) CT pulmonary angiography
(d) MR angiography
8.
Medical management of thyrotoxic crisis includes all of the following EXCEPT:

(a) IV fluids
(b) IV propanolol
(c) IV hydrocortisone
(d) IV antibiotics
9.
A patient operated for a parotid gland tumour developed symptoms of sweating and
erytherma (flushing) over the region of surgical excision while eating. The probable
diagnosis is:

(a) Parotid gland fistula
(b) Sialadenitis
(c) Chronic wound infection
(d) Frey?s syndrome
10.
First line hormone therapy for post-menopausal woman with metastatic carcinoma breast is:

(a) Tamoxifen
(b) Ovarian suppression by surgery
(c) Antiprogestins
(d) Anastrazole



11.
All of the following are major subtypes of breast cancer based on Gene array analysis
EXCEPT:

(a) Luminal A and Luminal B
(b) Triple negative
(c) Her-2 receptor positive
(d) Oestrogen receptor positive
12.
All of the following are sequelae of peptic ulcer surgery EXCEPT:

(a) Bilious vomiting
(b) Dumping syndrome
(c) Diarrhoea
(d) Increased appetite
13.
The Child-Turcotte-Pugh (CTP) score for quantifying the severity of chronic liver disease
includes all variables EXCEPT:

(a) Serum bilirubin
(b) Serum albumin
(c) Serum creatinine
(d) INR (International Normalised Ratio)
14.
?Chain of Lakes? appearance due to sacculation with intervening short strictures of pancreatic
duct is seen on:

(a) ERCP
(b) CECT abdomen
(c) Plain X-ray abdomen
(d) Ultrasonography
15.
Which one of the following statements is NOT correct regarding Pyogenic Liver Abscess?

(a) Anorexia, fever, malaise and right upper quadrant abdominal discomfort are the
most common presenting features
(b) It is more common in elderly, diabetics and immunocompromised patients
(c) Treatment is with oral antibiotics alone
(d) Streptococcus milleri and escherichia coli are the most common causative
organisms
16.
?Swiss cheese defects? are seen during laparoscopic repair of:

(a) Ventral hernia
(b) Inguinal hernia
(c) Obturator hernia
(d) Femoral hernia



17.
Which of the following are correct regarding splenic artery aneurysm?

1. Main arterial trunk is the common site
2. Palpable thrill can be felt
3. It is symptomless unless it ruptures

Select the correct answer using the code given below:

(a) 1 and 2 only
(b) 2 and 3 only
(c) 1 and 3 only
(d) 1, 2 and 3
18.
Valentino?s syndrome is:

(a) Pain on per-vaginal examination in pelvic abscess
(b) Pain over left shoulder in left hypochondriac collection
(c) Pain over left groin in perirenal collection
(d) Pain in right iliac fossa in perforated peptic ulcer
19.
Spontaneous bacterial peritonitis occurs due to:

(a) duodenal stump blowout
(b) peptic ulcer perforation
(c) acute bacterial infection of ascites
(d) infection via fallopian tubes
20.
Structure not forming boundaries of the ?Triangle of doom? seen during laparoscopic
inguinal hernia surgery dissection is:

(a) Vas deferens
(b) Inferior epigastric artery
(c) Spermatic cord vessels
(d) Peritoneum
21.
The term mid-line shift is associated with:

(a) Head injury
(b) Chest injury
(c) Abdominal injury
(d) Limb injury
22.
Which one of the following cranial nerves does NOT supply to the external ear?

(a) Cranial nerve V
(b) Cranial nerve VI
(c) Cranial nerve VII
(d) Cranial nerve IX

23.
Left Internal Mammary Artery (LIMA) has become the conduit of choice for Left Anterior
Descending (LAD) artery during coronary artery bypass grafting because:

(a) Long term patency rates are more than 98%
(b) It is close to LAD
(c) Atherosclerosis is never seen in this vessel
(d) It is very easy to harvest
24.
A 50-year old male with significant smoking history presented in the surgical emergency
with sudden severe breathlessness. Chest X-ray shows right sided Pneumothorax. The
appropriate management requires:

(a) Aspiration of air with 16-18 G cannula
(b) Right chest drain of size 8-14 Fr
(c) Oxygen by face mask
(d) Mechanical ventilation
25.
Which one of the following statements regarding Felon is NOT correct?

(a) There is infection of the finger tip between specialised fibrous septa
(b) It is a painless condition
(c) Incision and drainage is the treatment of choice
(d) It is common in diabetics
26.
The most common site for osteosarcoma is:

(a) Proximal femur
(b) Distal femur
(c) Proximal humerus
(d) Distal humerus
27.
Rapid Sequence Induction is indicated in:

(a) Emergency surgery for intestinal obstruction
(b) Elective open hernia surgery
(c) Cardiopulmonary bypass surgery
(d) Elective laparoscopic surgery

28.
Which one of the following statements about Compartment Syndrome is NOT correct?

(a) It is commonest in a closed fracture
(b) Pain is on active movement but not on passive movement of muscles
(c) Fasciotomy is the treatment of choice
(d) Volkmann?s Ischaemic contractive is a late complication




29.
Which one of the following is NOT the strength of ultrasound as a diagnostic modality?

(a) No radiation
(b) Short learning curve
(c) Inexpensive
(d) Allows dynamic studies to be done
30.
Which of the following statements regarding lymphoedema following breast cancer treatment
are correct?

1. Incidence has decreased due to rarely combined therapy of axillary LN dissection and
radiotherapy
2. Precipitating cause like LN metastasis is a major determinant
3. The condition is often painful
4. Oedematous limb is susceptible to bacterial infection

Select the correct answer using the code given below:

(a) 1, 2 and 3
(b) 2, 3 and 4
(c) 1, 3 and 4
(d) 1, 2 and 4
31.
Which one of the following is NOT a risk factor for development of venous thrombosis in
surgical patients?

(a) Age > 60 years
(b) Pregnancy
(c) Obesity (BMI > 30 kg/m
2
)
(d) Diabetes ( HbA1c > 7.5%)
32.
Which one of the following is NOT a complication of massive blood transfusion?

(a) Coagulopathy
(b) Hypocalcaemia
(c) Hyperthermia
(d) Hyperkalemia
33.
Stage III ?Pressure sore? is full thickness skin loss extending:

(a) into subcutaneous tissue but not through fascia
(b) through subcutaneous tissue into fascia
(c) through subcutaneous tissue into fascia and muscles
(d) through subcutaneous tissue into fascia, muscles and bone



34.
During subclavian vein puncture in a surgical ward suddenly a patient developed severe
breathlessness. On auscultation breath sound was absent and the ipsilateral chest was
tympanitic on percussion. The probable diagnosis is:

(a) Iatrogenic pneumothorax
(b) Introgenic hemothorax
(c) Spontaneous pneumothorax
(d) Tension pneumothorax
35.
Kohler?s disease is avascular necrosis of :

(a) Lunate
(b) Capitellum of humerus
(c) First metatarsal head
(d) Navicular
36.
Which one of the following is NOT true of Pyoderma gangrenosum?

(a) It is characterized by cutaneous ulceration with purple undermined edges
(b) It is often secondary to heightened immunological reactivity from another disease
process
(c) Cultures often show Gram positive Staphylococci
(d) Lesions generally respond to steroids
37.
Which of the following is NOT a tissue repair surgery for inguinal hernia repair?

(a) Bassini?s repair
(b) Shouldice repair
(c) Stoppa?s repair
(d) Desarda repair
38.
Which one of the following type of meshes is recommended for intraperitoneal use in
abdominal wall hernia?
(a) Light weight, porous meshes
(b) Heavy weight, porous meshes
(c) Absorbable meshes
(d) Tissue separating meshes
39.
Which one of the following is NOT correct regarding MEN-1 syndrome?

(a) It involves parathyroid glands
(b) It involves pancreas
(c) It involves pituitary gland
(d) It involves pineal gland

40.
Which one of the following is NOT an electronic information site in surgery?

(a) Pubmed
(b) Embase
(c) Cochrane library
(d) National medical library
41.
Which one of the following statements regarding pre-conceptional counseling is NOT
correct?

(a) It is needed only in selected complicated pregnancies
(b) It helps in early detection of risk factors
(c) It helps in reducing maternal morbidity and mortality
(d) It is a part of preventive medicine
42.
Consider the following statements regarding Non Stress Test (NST):

1. Reactive NST indicates a healthy fetus
2. NST is an observed association of fetal breathing with fetal movements
3. NST has a low false negative rate (< 1%) but high false positive rate (>50%)
4. Testing should be started at 20 weeks

Which of the statement(s) given above is/are correct?

(a) 1 and 3
(b) 2 only
(c) 3 only
(d) 1 and 4
43.
Which one of the following is a protective factor for endometrial hyperplasia?

(a) Diabetes
(b) Tamoxifen therapy
(c) Multiparity
(d) Delayed menopause
44.
A woman who is not breast feeding her newborn child is advised to use a contraceptive
method by:

(a) 3
rd
postpartum week
(b) 6
th
postpartum week
(c) 3
rd
postpartum month
(d) 6
th
postpartum month





45.
Pearl index for contraceptive effectiveness is calculated in terms of which of the following?

1. Pregnancy rate
2. Abortion rate
3. Hundred woman years
4. Thousand woman years

Select the correct answer using the code given below:

(a) 1 only
(b) 2 and 3
(c) 1, 2 and 4
(d) 1 and 3
46.
Indications for removal of IUDs are all EXCEPT:

(a) Perforation of uterus
(b) Cyclical menstrual bleeding
(c) Flaring up of salpingitis
(d) Pregnancy with IUD
47.
Contraindications for insertion of IUDs are all EXCEPT:

(a) Suspected pregnancy
(b) Trophoblastic disease
(c) Severe dysmenorrhea
(d) During cesarean section
48.
Which one of the following is NOT a contraindication for use of Mini pill?

(a) Pregnancy
(b) Breast feeding
(c) Thromboembolic disease
(d) History of breast cancer
49.
Which one of the following is the most commonly used surgical method/technique of female
sterilization as recommended by Government of India?

(a) Uchida technique
(b) Irving method
(c) Pomeroy?s method
(d) Madlener technique







50.
Which of the following is/are required for a registered medical practitioner to qualify for
performing Medical Termination of Pregnancy (MTP), as per revised rules of MTP Act?

1. Certified for assisting at least 15 MTP in an authorized centre
2. Diploma or degree in Obstetrics and Gynaecology
3. House surgeon training for 3 months in Obstetrics and Gynaecology
4. Certified training for 6 months in laparoscopic surgeries

Select the correct answer using the code given below:
(a) 1 only
(b) 2 only
(c) 1, 2 and 3
(d) 1, 2 and 4
51.
Which one of the following is NOT a support of uterus, preventing its descent?

(a) Endopelvic fascia
(b) Mackenrodt?s ligament
(c) Inguinal ligament
(d) Pubocervical ligament
52.
As per ICMR guidelines, which one of the following statements is true regarding effects of
COVID-19 on fetus according to current evidence?

(a) There is increased risk of early pregnancy loss
(b) COVID-19 virus is not teratogenic
(c) COVID-19 virus infection is an indication of MTP
(d) There is increased risk of fetal growth restriction
53.
As per ICMR guidelines, which one of the following statements is true regarding COVID-19
infection in pregnancy?

(a) Covid-19 pneumonia in pregnancy is more severe with poor recovery
(b) Pregnant women with heart disease are at higher risk
(c) Vaginal secretions always test positive for COVID-19 in pregnancy
(d) COVID-19 virus is secreted in breast milk
54.
Which one of the following is NOT a method of management of Deep Transverse Arrest with
the living fetus?

(a) Caesarean section
(b) Delivery by ventouse
(c) Delivery by application of forceps to the unrotated head
(d) Manual rotation and application of forceps





FirstRanker.com - FirstRanker's Choice
Combined Medical Services Examination-2020
Paper-II

1.
Indications for fasciotomy in compartment syndrome include all EXCEPT:

(a) Distal sensory disturbance
(b) Compartment pressure > 30 mm Hg
(c) Pain on passive movement of affected muscles
(d) Palpable distal pulses
2.
Which one of the following statements is NOT correct regarding Necrotising Soft Tissue
infections?

(a) Crepitus, skin blistering and focal skin gangrene are typical presenting features
(b) They are monomicrobial in nature
(c) Treatment consists of wide local excision and appropriate antibiotics
(d) Tissue biopsy is required for culture and diagnosis
3.
Which type of surgery is laparoscopic cholecystectomy classified as?

(a) Clean
(b) Clean contaminated
(c) Contaminated
(d) Dirty
4.
A 22-year female has presented with a history of malaise, cough, alternating constipation and
diarrhoea with intermittent abdominal pain for last 6 months. She also complains of
abdominal distension for last 2 days. On examination her abdomen has a doughy feel along
with an ill defined mass over the right lower quadrant. She is most likely suffering from:

(a) Appendicular lump
(b) Ileocaecal tuberculosis
(c) Carcinoma caecum
(d) Ovarian mass
5.
Consider the following statements regarding needle stick injuries:

1. Injured part should be washed under running water
2. Dominant index finger is the commonest site for needle stick injury
3. All needle stick injuries should be reported
4. Hepatitis/HIV testing should be done after needle stick injury

Which of the statements given above are correct?

(a) 1, 2 and 4
(b) 1, 2 and 3
(c) 1, 3 and 4
(d) 2, 3 and 4




6.
Consider the following statements regarding claudication:

1. It is a marker for silent coronary disease
2. Structured exercise program ( 2 hours per week for 3 months) leads to
improvement in symptoms
3. Diabetes mellitus increases the risk and severity of claudication
4. Beta blockers may exacerbate claudication

Which of the above statements are correct?

(a) 1 and 2 only
(b) 1, 3 and 4 only
(c) 2, 3 and 4 only
(d) 1, 2, 3 and 4
7.
A 50-year old lady underwent uneventful bariatric surgery for morbid obesity. On the third
post operative day, she develops breathless and pulmonary embolism is suspected. The next
investigation to confirm the diagnosis will be:

(a) Echocardiography
(b) Duplex venography
(c) CT pulmonary angiography
(d) MR angiography
8.
Medical management of thyrotoxic crisis includes all of the following EXCEPT:

(a) IV fluids
(b) IV propanolol
(c) IV hydrocortisone
(d) IV antibiotics
9.
A patient operated for a parotid gland tumour developed symptoms of sweating and
erytherma (flushing) over the region of surgical excision while eating. The probable
diagnosis is:

(a) Parotid gland fistula
(b) Sialadenitis
(c) Chronic wound infection
(d) Frey?s syndrome
10.
First line hormone therapy for post-menopausal woman with metastatic carcinoma breast is:

(a) Tamoxifen
(b) Ovarian suppression by surgery
(c) Antiprogestins
(d) Anastrazole



11.
All of the following are major subtypes of breast cancer based on Gene array analysis
EXCEPT:

(a) Luminal A and Luminal B
(b) Triple negative
(c) Her-2 receptor positive
(d) Oestrogen receptor positive
12.
All of the following are sequelae of peptic ulcer surgery EXCEPT:

(a) Bilious vomiting
(b) Dumping syndrome
(c) Diarrhoea
(d) Increased appetite
13.
The Child-Turcotte-Pugh (CTP) score for quantifying the severity of chronic liver disease
includes all variables EXCEPT:

(a) Serum bilirubin
(b) Serum albumin
(c) Serum creatinine
(d) INR (International Normalised Ratio)
14.
?Chain of Lakes? appearance due to sacculation with intervening short strictures of pancreatic
duct is seen on:

(a) ERCP
(b) CECT abdomen
(c) Plain X-ray abdomen
(d) Ultrasonography
15.
Which one of the following statements is NOT correct regarding Pyogenic Liver Abscess?

(a) Anorexia, fever, malaise and right upper quadrant abdominal discomfort are the
most common presenting features
(b) It is more common in elderly, diabetics and immunocompromised patients
(c) Treatment is with oral antibiotics alone
(d) Streptococcus milleri and escherichia coli are the most common causative
organisms
16.
?Swiss cheese defects? are seen during laparoscopic repair of:

(a) Ventral hernia
(b) Inguinal hernia
(c) Obturator hernia
(d) Femoral hernia



17.
Which of the following are correct regarding splenic artery aneurysm?

1. Main arterial trunk is the common site
2. Palpable thrill can be felt
3. It is symptomless unless it ruptures

Select the correct answer using the code given below:

(a) 1 and 2 only
(b) 2 and 3 only
(c) 1 and 3 only
(d) 1, 2 and 3
18.
Valentino?s syndrome is:

(a) Pain on per-vaginal examination in pelvic abscess
(b) Pain over left shoulder in left hypochondriac collection
(c) Pain over left groin in perirenal collection
(d) Pain in right iliac fossa in perforated peptic ulcer
19.
Spontaneous bacterial peritonitis occurs due to:

(a) duodenal stump blowout
(b) peptic ulcer perforation
(c) acute bacterial infection of ascites
(d) infection via fallopian tubes
20.
Structure not forming boundaries of the ?Triangle of doom? seen during laparoscopic
inguinal hernia surgery dissection is:

(a) Vas deferens
(b) Inferior epigastric artery
(c) Spermatic cord vessels
(d) Peritoneum
21.
The term mid-line shift is associated with:

(a) Head injury
(b) Chest injury
(c) Abdominal injury
(d) Limb injury
22.
Which one of the following cranial nerves does NOT supply to the external ear?

(a) Cranial nerve V
(b) Cranial nerve VI
(c) Cranial nerve VII
(d) Cranial nerve IX

23.
Left Internal Mammary Artery (LIMA) has become the conduit of choice for Left Anterior
Descending (LAD) artery during coronary artery bypass grafting because:

(a) Long term patency rates are more than 98%
(b) It is close to LAD
(c) Atherosclerosis is never seen in this vessel
(d) It is very easy to harvest
24.
A 50-year old male with significant smoking history presented in the surgical emergency
with sudden severe breathlessness. Chest X-ray shows right sided Pneumothorax. The
appropriate management requires:

(a) Aspiration of air with 16-18 G cannula
(b) Right chest drain of size 8-14 Fr
(c) Oxygen by face mask
(d) Mechanical ventilation
25.
Which one of the following statements regarding Felon is NOT correct?

(a) There is infection of the finger tip between specialised fibrous septa
(b) It is a painless condition
(c) Incision and drainage is the treatment of choice
(d) It is common in diabetics
26.
The most common site for osteosarcoma is:

(a) Proximal femur
(b) Distal femur
(c) Proximal humerus
(d) Distal humerus
27.
Rapid Sequence Induction is indicated in:

(a) Emergency surgery for intestinal obstruction
(b) Elective open hernia surgery
(c) Cardiopulmonary bypass surgery
(d) Elective laparoscopic surgery

28.
Which one of the following statements about Compartment Syndrome is NOT correct?

(a) It is commonest in a closed fracture
(b) Pain is on active movement but not on passive movement of muscles
(c) Fasciotomy is the treatment of choice
(d) Volkmann?s Ischaemic contractive is a late complication




29.
Which one of the following is NOT the strength of ultrasound as a diagnostic modality?

(a) No radiation
(b) Short learning curve
(c) Inexpensive
(d) Allows dynamic studies to be done
30.
Which of the following statements regarding lymphoedema following breast cancer treatment
are correct?

1. Incidence has decreased due to rarely combined therapy of axillary LN dissection and
radiotherapy
2. Precipitating cause like LN metastasis is a major determinant
3. The condition is often painful
4. Oedematous limb is susceptible to bacterial infection

Select the correct answer using the code given below:

(a) 1, 2 and 3
(b) 2, 3 and 4
(c) 1, 3 and 4
(d) 1, 2 and 4
31.
Which one of the following is NOT a risk factor for development of venous thrombosis in
surgical patients?

(a) Age > 60 years
(b) Pregnancy
(c) Obesity (BMI > 30 kg/m
2
)
(d) Diabetes ( HbA1c > 7.5%)
32.
Which one of the following is NOT a complication of massive blood transfusion?

(a) Coagulopathy
(b) Hypocalcaemia
(c) Hyperthermia
(d) Hyperkalemia
33.
Stage III ?Pressure sore? is full thickness skin loss extending:

(a) into subcutaneous tissue but not through fascia
(b) through subcutaneous tissue into fascia
(c) through subcutaneous tissue into fascia and muscles
(d) through subcutaneous tissue into fascia, muscles and bone



34.
During subclavian vein puncture in a surgical ward suddenly a patient developed severe
breathlessness. On auscultation breath sound was absent and the ipsilateral chest was
tympanitic on percussion. The probable diagnosis is:

(a) Iatrogenic pneumothorax
(b) Introgenic hemothorax
(c) Spontaneous pneumothorax
(d) Tension pneumothorax
35.
Kohler?s disease is avascular necrosis of :

(a) Lunate
(b) Capitellum of humerus
(c) First metatarsal head
(d) Navicular
36.
Which one of the following is NOT true of Pyoderma gangrenosum?

(a) It is characterized by cutaneous ulceration with purple undermined edges
(b) It is often secondary to heightened immunological reactivity from another disease
process
(c) Cultures often show Gram positive Staphylococci
(d) Lesions generally respond to steroids
37.
Which of the following is NOT a tissue repair surgery for inguinal hernia repair?

(a) Bassini?s repair
(b) Shouldice repair
(c) Stoppa?s repair
(d) Desarda repair
38.
Which one of the following type of meshes is recommended for intraperitoneal use in
abdominal wall hernia?
(a) Light weight, porous meshes
(b) Heavy weight, porous meshes
(c) Absorbable meshes
(d) Tissue separating meshes
39.
Which one of the following is NOT correct regarding MEN-1 syndrome?

(a) It involves parathyroid glands
(b) It involves pancreas
(c) It involves pituitary gland
(d) It involves pineal gland

40.
Which one of the following is NOT an electronic information site in surgery?

(a) Pubmed
(b) Embase
(c) Cochrane library
(d) National medical library
41.
Which one of the following statements regarding pre-conceptional counseling is NOT
correct?

(a) It is needed only in selected complicated pregnancies
(b) It helps in early detection of risk factors
(c) It helps in reducing maternal morbidity and mortality
(d) It is a part of preventive medicine
42.
Consider the following statements regarding Non Stress Test (NST):

1. Reactive NST indicates a healthy fetus
2. NST is an observed association of fetal breathing with fetal movements
3. NST has a low false negative rate (< 1%) but high false positive rate (>50%)
4. Testing should be started at 20 weeks

Which of the statement(s) given above is/are correct?

(a) 1 and 3
(b) 2 only
(c) 3 only
(d) 1 and 4
43.
Which one of the following is a protective factor for endometrial hyperplasia?

(a) Diabetes
(b) Tamoxifen therapy
(c) Multiparity
(d) Delayed menopause
44.
A woman who is not breast feeding her newborn child is advised to use a contraceptive
method by:

(a) 3
rd
postpartum week
(b) 6
th
postpartum week
(c) 3
rd
postpartum month
(d) 6
th
postpartum month





45.
Pearl index for contraceptive effectiveness is calculated in terms of which of the following?

1. Pregnancy rate
2. Abortion rate
3. Hundred woman years
4. Thousand woman years

Select the correct answer using the code given below:

(a) 1 only
(b) 2 and 3
(c) 1, 2 and 4
(d) 1 and 3
46.
Indications for removal of IUDs are all EXCEPT:

(a) Perforation of uterus
(b) Cyclical menstrual bleeding
(c) Flaring up of salpingitis
(d) Pregnancy with IUD
47.
Contraindications for insertion of IUDs are all EXCEPT:

(a) Suspected pregnancy
(b) Trophoblastic disease
(c) Severe dysmenorrhea
(d) During cesarean section
48.
Which one of the following is NOT a contraindication for use of Mini pill?

(a) Pregnancy
(b) Breast feeding
(c) Thromboembolic disease
(d) History of breast cancer
49.
Which one of the following is the most commonly used surgical method/technique of female
sterilization as recommended by Government of India?

(a) Uchida technique
(b) Irving method
(c) Pomeroy?s method
(d) Madlener technique







50.
Which of the following is/are required for a registered medical practitioner to qualify for
performing Medical Termination of Pregnancy (MTP), as per revised rules of MTP Act?

1. Certified for assisting at least 15 MTP in an authorized centre
2. Diploma or degree in Obstetrics and Gynaecology
3. House surgeon training for 3 months in Obstetrics and Gynaecology
4. Certified training for 6 months in laparoscopic surgeries

Select the correct answer using the code given below:
(a) 1 only
(b) 2 only
(c) 1, 2 and 3
(d) 1, 2 and 4
51.
Which one of the following is NOT a support of uterus, preventing its descent?

(a) Endopelvic fascia
(b) Mackenrodt?s ligament
(c) Inguinal ligament
(d) Pubocervical ligament
52.
As per ICMR guidelines, which one of the following statements is true regarding effects of
COVID-19 on fetus according to current evidence?

(a) There is increased risk of early pregnancy loss
(b) COVID-19 virus is not teratogenic
(c) COVID-19 virus infection is an indication of MTP
(d) There is increased risk of fetal growth restriction
53.
As per ICMR guidelines, which one of the following statements is true regarding COVID-19
infection in pregnancy?

(a) Covid-19 pneumonia in pregnancy is more severe with poor recovery
(b) Pregnant women with heart disease are at higher risk
(c) Vaginal secretions always test positive for COVID-19 in pregnancy
(d) COVID-19 virus is secreted in breast milk
54.
Which one of the following is NOT a method of management of Deep Transverse Arrest with
the living fetus?

(a) Caesarean section
(b) Delivery by ventouse
(c) Delivery by application of forceps to the unrotated head
(d) Manual rotation and application of forceps





55.
Successful version of breech presentation is likely in case all of the following EXCEPT:

(a) Breech with extended legs
(b) Complete breech with sacroanterior position
(c) Non engaged breech
(d) Adequate amniotic fluid
56.
Implantation of a fertilised ovum occurs on which day following fertilisation?

(a) Day 6
(b) Day 10
(c) Day 14
(d) Day 20
57.
During total abdominal hysterectomy the ureter is likely to undergo injury or ligation during
the following steps EXCEPT:

(a) During division and ligation of the round ligaments
(b) During division and ligation of infundibulopelvic ligaments
(c) During division and ligation of mackenrodt?s and uterosacral ligaments
(d) At the vaginal angles while incising the vagina to remove the cervix with the
uterus
58.
The net effect of antenatal care has been the following EXCEPT:

(a) Reduction in maternal mortality
(b) Reduction in perinatal mortality
(c) Reduction in the incidence of institutional delivery
(d) Reduction in maternal morbidity
59.
Which one of the following is NOT a component of active phase in the partograph?

(a) Acceleration phase
(b) Phase of maximum slope
(c) Phase of deceleration
(d) Phase of expulsion
60.
From medicolegal point of view which one of the following is NOT a sign of previous child
birth?

(a) Perineum is lax and there is evidence of scarring
(b) Introitus is gaping and there is presence of carunculae myrtiformis
(c) Abdomen is lax and loose with striae and linea alba
(d) Conical cervix with round external os





FirstRanker.com - FirstRanker's Choice
Combined Medical Services Examination-2020
Paper-II

1.
Indications for fasciotomy in compartment syndrome include all EXCEPT:

(a) Distal sensory disturbance
(b) Compartment pressure > 30 mm Hg
(c) Pain on passive movement of affected muscles
(d) Palpable distal pulses
2.
Which one of the following statements is NOT correct regarding Necrotising Soft Tissue
infections?

(a) Crepitus, skin blistering and focal skin gangrene are typical presenting features
(b) They are monomicrobial in nature
(c) Treatment consists of wide local excision and appropriate antibiotics
(d) Tissue biopsy is required for culture and diagnosis
3.
Which type of surgery is laparoscopic cholecystectomy classified as?

(a) Clean
(b) Clean contaminated
(c) Contaminated
(d) Dirty
4.
A 22-year female has presented with a history of malaise, cough, alternating constipation and
diarrhoea with intermittent abdominal pain for last 6 months. She also complains of
abdominal distension for last 2 days. On examination her abdomen has a doughy feel along
with an ill defined mass over the right lower quadrant. She is most likely suffering from:

(a) Appendicular lump
(b) Ileocaecal tuberculosis
(c) Carcinoma caecum
(d) Ovarian mass
5.
Consider the following statements regarding needle stick injuries:

1. Injured part should be washed under running water
2. Dominant index finger is the commonest site for needle stick injury
3. All needle stick injuries should be reported
4. Hepatitis/HIV testing should be done after needle stick injury

Which of the statements given above are correct?

(a) 1, 2 and 4
(b) 1, 2 and 3
(c) 1, 3 and 4
(d) 2, 3 and 4




6.
Consider the following statements regarding claudication:

1. It is a marker for silent coronary disease
2. Structured exercise program ( 2 hours per week for 3 months) leads to
improvement in symptoms
3. Diabetes mellitus increases the risk and severity of claudication
4. Beta blockers may exacerbate claudication

Which of the above statements are correct?

(a) 1 and 2 only
(b) 1, 3 and 4 only
(c) 2, 3 and 4 only
(d) 1, 2, 3 and 4
7.
A 50-year old lady underwent uneventful bariatric surgery for morbid obesity. On the third
post operative day, she develops breathless and pulmonary embolism is suspected. The next
investigation to confirm the diagnosis will be:

(a) Echocardiography
(b) Duplex venography
(c) CT pulmonary angiography
(d) MR angiography
8.
Medical management of thyrotoxic crisis includes all of the following EXCEPT:

(a) IV fluids
(b) IV propanolol
(c) IV hydrocortisone
(d) IV antibiotics
9.
A patient operated for a parotid gland tumour developed symptoms of sweating and
erytherma (flushing) over the region of surgical excision while eating. The probable
diagnosis is:

(a) Parotid gland fistula
(b) Sialadenitis
(c) Chronic wound infection
(d) Frey?s syndrome
10.
First line hormone therapy for post-menopausal woman with metastatic carcinoma breast is:

(a) Tamoxifen
(b) Ovarian suppression by surgery
(c) Antiprogestins
(d) Anastrazole



11.
All of the following are major subtypes of breast cancer based on Gene array analysis
EXCEPT:

(a) Luminal A and Luminal B
(b) Triple negative
(c) Her-2 receptor positive
(d) Oestrogen receptor positive
12.
All of the following are sequelae of peptic ulcer surgery EXCEPT:

(a) Bilious vomiting
(b) Dumping syndrome
(c) Diarrhoea
(d) Increased appetite
13.
The Child-Turcotte-Pugh (CTP) score for quantifying the severity of chronic liver disease
includes all variables EXCEPT:

(a) Serum bilirubin
(b) Serum albumin
(c) Serum creatinine
(d) INR (International Normalised Ratio)
14.
?Chain of Lakes? appearance due to sacculation with intervening short strictures of pancreatic
duct is seen on:

(a) ERCP
(b) CECT abdomen
(c) Plain X-ray abdomen
(d) Ultrasonography
15.
Which one of the following statements is NOT correct regarding Pyogenic Liver Abscess?

(a) Anorexia, fever, malaise and right upper quadrant abdominal discomfort are the
most common presenting features
(b) It is more common in elderly, diabetics and immunocompromised patients
(c) Treatment is with oral antibiotics alone
(d) Streptococcus milleri and escherichia coli are the most common causative
organisms
16.
?Swiss cheese defects? are seen during laparoscopic repair of:

(a) Ventral hernia
(b) Inguinal hernia
(c) Obturator hernia
(d) Femoral hernia



17.
Which of the following are correct regarding splenic artery aneurysm?

1. Main arterial trunk is the common site
2. Palpable thrill can be felt
3. It is symptomless unless it ruptures

Select the correct answer using the code given below:

(a) 1 and 2 only
(b) 2 and 3 only
(c) 1 and 3 only
(d) 1, 2 and 3
18.
Valentino?s syndrome is:

(a) Pain on per-vaginal examination in pelvic abscess
(b) Pain over left shoulder in left hypochondriac collection
(c) Pain over left groin in perirenal collection
(d) Pain in right iliac fossa in perforated peptic ulcer
19.
Spontaneous bacterial peritonitis occurs due to:

(a) duodenal stump blowout
(b) peptic ulcer perforation
(c) acute bacterial infection of ascites
(d) infection via fallopian tubes
20.
Structure not forming boundaries of the ?Triangle of doom? seen during laparoscopic
inguinal hernia surgery dissection is:

(a) Vas deferens
(b) Inferior epigastric artery
(c) Spermatic cord vessels
(d) Peritoneum
21.
The term mid-line shift is associated with:

(a) Head injury
(b) Chest injury
(c) Abdominal injury
(d) Limb injury
22.
Which one of the following cranial nerves does NOT supply to the external ear?

(a) Cranial nerve V
(b) Cranial nerve VI
(c) Cranial nerve VII
(d) Cranial nerve IX

23.
Left Internal Mammary Artery (LIMA) has become the conduit of choice for Left Anterior
Descending (LAD) artery during coronary artery bypass grafting because:

(a) Long term patency rates are more than 98%
(b) It is close to LAD
(c) Atherosclerosis is never seen in this vessel
(d) It is very easy to harvest
24.
A 50-year old male with significant smoking history presented in the surgical emergency
with sudden severe breathlessness. Chest X-ray shows right sided Pneumothorax. The
appropriate management requires:

(a) Aspiration of air with 16-18 G cannula
(b) Right chest drain of size 8-14 Fr
(c) Oxygen by face mask
(d) Mechanical ventilation
25.
Which one of the following statements regarding Felon is NOT correct?

(a) There is infection of the finger tip between specialised fibrous septa
(b) It is a painless condition
(c) Incision and drainage is the treatment of choice
(d) It is common in diabetics
26.
The most common site for osteosarcoma is:

(a) Proximal femur
(b) Distal femur
(c) Proximal humerus
(d) Distal humerus
27.
Rapid Sequence Induction is indicated in:

(a) Emergency surgery for intestinal obstruction
(b) Elective open hernia surgery
(c) Cardiopulmonary bypass surgery
(d) Elective laparoscopic surgery

28.
Which one of the following statements about Compartment Syndrome is NOT correct?

(a) It is commonest in a closed fracture
(b) Pain is on active movement but not on passive movement of muscles
(c) Fasciotomy is the treatment of choice
(d) Volkmann?s Ischaemic contractive is a late complication




29.
Which one of the following is NOT the strength of ultrasound as a diagnostic modality?

(a) No radiation
(b) Short learning curve
(c) Inexpensive
(d) Allows dynamic studies to be done
30.
Which of the following statements regarding lymphoedema following breast cancer treatment
are correct?

1. Incidence has decreased due to rarely combined therapy of axillary LN dissection and
radiotherapy
2. Precipitating cause like LN metastasis is a major determinant
3. The condition is often painful
4. Oedematous limb is susceptible to bacterial infection

Select the correct answer using the code given below:

(a) 1, 2 and 3
(b) 2, 3 and 4
(c) 1, 3 and 4
(d) 1, 2 and 4
31.
Which one of the following is NOT a risk factor for development of venous thrombosis in
surgical patients?

(a) Age > 60 years
(b) Pregnancy
(c) Obesity (BMI > 30 kg/m
2
)
(d) Diabetes ( HbA1c > 7.5%)
32.
Which one of the following is NOT a complication of massive blood transfusion?

(a) Coagulopathy
(b) Hypocalcaemia
(c) Hyperthermia
(d) Hyperkalemia
33.
Stage III ?Pressure sore? is full thickness skin loss extending:

(a) into subcutaneous tissue but not through fascia
(b) through subcutaneous tissue into fascia
(c) through subcutaneous tissue into fascia and muscles
(d) through subcutaneous tissue into fascia, muscles and bone



34.
During subclavian vein puncture in a surgical ward suddenly a patient developed severe
breathlessness. On auscultation breath sound was absent and the ipsilateral chest was
tympanitic on percussion. The probable diagnosis is:

(a) Iatrogenic pneumothorax
(b) Introgenic hemothorax
(c) Spontaneous pneumothorax
(d) Tension pneumothorax
35.
Kohler?s disease is avascular necrosis of :

(a) Lunate
(b) Capitellum of humerus
(c) First metatarsal head
(d) Navicular
36.
Which one of the following is NOT true of Pyoderma gangrenosum?

(a) It is characterized by cutaneous ulceration with purple undermined edges
(b) It is often secondary to heightened immunological reactivity from another disease
process
(c) Cultures often show Gram positive Staphylococci
(d) Lesions generally respond to steroids
37.
Which of the following is NOT a tissue repair surgery for inguinal hernia repair?

(a) Bassini?s repair
(b) Shouldice repair
(c) Stoppa?s repair
(d) Desarda repair
38.
Which one of the following type of meshes is recommended for intraperitoneal use in
abdominal wall hernia?
(a) Light weight, porous meshes
(b) Heavy weight, porous meshes
(c) Absorbable meshes
(d) Tissue separating meshes
39.
Which one of the following is NOT correct regarding MEN-1 syndrome?

(a) It involves parathyroid glands
(b) It involves pancreas
(c) It involves pituitary gland
(d) It involves pineal gland

40.
Which one of the following is NOT an electronic information site in surgery?

(a) Pubmed
(b) Embase
(c) Cochrane library
(d) National medical library
41.
Which one of the following statements regarding pre-conceptional counseling is NOT
correct?

(a) It is needed only in selected complicated pregnancies
(b) It helps in early detection of risk factors
(c) It helps in reducing maternal morbidity and mortality
(d) It is a part of preventive medicine
42.
Consider the following statements regarding Non Stress Test (NST):

1. Reactive NST indicates a healthy fetus
2. NST is an observed association of fetal breathing with fetal movements
3. NST has a low false negative rate (< 1%) but high false positive rate (>50%)
4. Testing should be started at 20 weeks

Which of the statement(s) given above is/are correct?

(a) 1 and 3
(b) 2 only
(c) 3 only
(d) 1 and 4
43.
Which one of the following is a protective factor for endometrial hyperplasia?

(a) Diabetes
(b) Tamoxifen therapy
(c) Multiparity
(d) Delayed menopause
44.
A woman who is not breast feeding her newborn child is advised to use a contraceptive
method by:

(a) 3
rd
postpartum week
(b) 6
th
postpartum week
(c) 3
rd
postpartum month
(d) 6
th
postpartum month





45.
Pearl index for contraceptive effectiveness is calculated in terms of which of the following?

1. Pregnancy rate
2. Abortion rate
3. Hundred woman years
4. Thousand woman years

Select the correct answer using the code given below:

(a) 1 only
(b) 2 and 3
(c) 1, 2 and 4
(d) 1 and 3
46.
Indications for removal of IUDs are all EXCEPT:

(a) Perforation of uterus
(b) Cyclical menstrual bleeding
(c) Flaring up of salpingitis
(d) Pregnancy with IUD
47.
Contraindications for insertion of IUDs are all EXCEPT:

(a) Suspected pregnancy
(b) Trophoblastic disease
(c) Severe dysmenorrhea
(d) During cesarean section
48.
Which one of the following is NOT a contraindication for use of Mini pill?

(a) Pregnancy
(b) Breast feeding
(c) Thromboembolic disease
(d) History of breast cancer
49.
Which one of the following is the most commonly used surgical method/technique of female
sterilization as recommended by Government of India?

(a) Uchida technique
(b) Irving method
(c) Pomeroy?s method
(d) Madlener technique







50.
Which of the following is/are required for a registered medical practitioner to qualify for
performing Medical Termination of Pregnancy (MTP), as per revised rules of MTP Act?

1. Certified for assisting at least 15 MTP in an authorized centre
2. Diploma or degree in Obstetrics and Gynaecology
3. House surgeon training for 3 months in Obstetrics and Gynaecology
4. Certified training for 6 months in laparoscopic surgeries

Select the correct answer using the code given below:
(a) 1 only
(b) 2 only
(c) 1, 2 and 3
(d) 1, 2 and 4
51.
Which one of the following is NOT a support of uterus, preventing its descent?

(a) Endopelvic fascia
(b) Mackenrodt?s ligament
(c) Inguinal ligament
(d) Pubocervical ligament
52.
As per ICMR guidelines, which one of the following statements is true regarding effects of
COVID-19 on fetus according to current evidence?

(a) There is increased risk of early pregnancy loss
(b) COVID-19 virus is not teratogenic
(c) COVID-19 virus infection is an indication of MTP
(d) There is increased risk of fetal growth restriction
53.
As per ICMR guidelines, which one of the following statements is true regarding COVID-19
infection in pregnancy?

(a) Covid-19 pneumonia in pregnancy is more severe with poor recovery
(b) Pregnant women with heart disease are at higher risk
(c) Vaginal secretions always test positive for COVID-19 in pregnancy
(d) COVID-19 virus is secreted in breast milk
54.
Which one of the following is NOT a method of management of Deep Transverse Arrest with
the living fetus?

(a) Caesarean section
(b) Delivery by ventouse
(c) Delivery by application of forceps to the unrotated head
(d) Manual rotation and application of forceps





55.
Successful version of breech presentation is likely in case all of the following EXCEPT:

(a) Breech with extended legs
(b) Complete breech with sacroanterior position
(c) Non engaged breech
(d) Adequate amniotic fluid
56.
Implantation of a fertilised ovum occurs on which day following fertilisation?

(a) Day 6
(b) Day 10
(c) Day 14
(d) Day 20
57.
During total abdominal hysterectomy the ureter is likely to undergo injury or ligation during
the following steps EXCEPT:

(a) During division and ligation of the round ligaments
(b) During division and ligation of infundibulopelvic ligaments
(c) During division and ligation of mackenrodt?s and uterosacral ligaments
(d) At the vaginal angles while incising the vagina to remove the cervix with the
uterus
58.
The net effect of antenatal care has been the following EXCEPT:

(a) Reduction in maternal mortality
(b) Reduction in perinatal mortality
(c) Reduction in the incidence of institutional delivery
(d) Reduction in maternal morbidity
59.
Which one of the following is NOT a component of active phase in the partograph?

(a) Acceleration phase
(b) Phase of maximum slope
(c) Phase of deceleration
(d) Phase of expulsion
60.
From medicolegal point of view which one of the following is NOT a sign of previous child
birth?

(a) Perineum is lax and there is evidence of scarring
(b) Introitus is gaping and there is presence of carunculae myrtiformis
(c) Abdomen is lax and loose with striae and linea alba
(d) Conical cervix with round external os





61.
The components of partograph are all EXCEPT:

(a) Time
(b) Fetal heart rate
(c) Maternal respiratory rate
(d) Maternal urine analysis
62.
Which of the following information are provided by partograph?

1. Colour of liquor
2. Uterine contractions with duration and frequency
3. Dilatation of cervix

Select the correct answer using the code given below:

(a) 1 and 2 only
(b) 2 and 3 only
(c) 1 and 3 only
(d) 1, 2 and 3
63.
Intraoperative recognition of ureter is by which of the following features?

1. Transparent tubular appearance
2. Pale glistening appearance
3. Longitudinal vessels on surface
4. Circumferential vessels on surface

Select the correct answer using the code given below:

(a) 1 and 3
(b) 2 and 4
(c) 2 and 3
(d) 1 and 4
64.
Hysterosalpingography (HSG) is least helpful in detecting which of the following?

(a) Tubal patency
(b) Pelvic adhesions
(c) Asherman syndrome
(d) Congenital uterine anomaly









FirstRanker.com - FirstRanker's Choice
Combined Medical Services Examination-2020
Paper-II

1.
Indications for fasciotomy in compartment syndrome include all EXCEPT:

(a) Distal sensory disturbance
(b) Compartment pressure > 30 mm Hg
(c) Pain on passive movement of affected muscles
(d) Palpable distal pulses
2.
Which one of the following statements is NOT correct regarding Necrotising Soft Tissue
infections?

(a) Crepitus, skin blistering and focal skin gangrene are typical presenting features
(b) They are monomicrobial in nature
(c) Treatment consists of wide local excision and appropriate antibiotics
(d) Tissue biopsy is required for culture and diagnosis
3.
Which type of surgery is laparoscopic cholecystectomy classified as?

(a) Clean
(b) Clean contaminated
(c) Contaminated
(d) Dirty
4.
A 22-year female has presented with a history of malaise, cough, alternating constipation and
diarrhoea with intermittent abdominal pain for last 6 months. She also complains of
abdominal distension for last 2 days. On examination her abdomen has a doughy feel along
with an ill defined mass over the right lower quadrant. She is most likely suffering from:

(a) Appendicular lump
(b) Ileocaecal tuberculosis
(c) Carcinoma caecum
(d) Ovarian mass
5.
Consider the following statements regarding needle stick injuries:

1. Injured part should be washed under running water
2. Dominant index finger is the commonest site for needle stick injury
3. All needle stick injuries should be reported
4. Hepatitis/HIV testing should be done after needle stick injury

Which of the statements given above are correct?

(a) 1, 2 and 4
(b) 1, 2 and 3
(c) 1, 3 and 4
(d) 2, 3 and 4




6.
Consider the following statements regarding claudication:

1. It is a marker for silent coronary disease
2. Structured exercise program ( 2 hours per week for 3 months) leads to
improvement in symptoms
3. Diabetes mellitus increases the risk and severity of claudication
4. Beta blockers may exacerbate claudication

Which of the above statements are correct?

(a) 1 and 2 only
(b) 1, 3 and 4 only
(c) 2, 3 and 4 only
(d) 1, 2, 3 and 4
7.
A 50-year old lady underwent uneventful bariatric surgery for morbid obesity. On the third
post operative day, she develops breathless and pulmonary embolism is suspected. The next
investigation to confirm the diagnosis will be:

(a) Echocardiography
(b) Duplex venography
(c) CT pulmonary angiography
(d) MR angiography
8.
Medical management of thyrotoxic crisis includes all of the following EXCEPT:

(a) IV fluids
(b) IV propanolol
(c) IV hydrocortisone
(d) IV antibiotics
9.
A patient operated for a parotid gland tumour developed symptoms of sweating and
erytherma (flushing) over the region of surgical excision while eating. The probable
diagnosis is:

(a) Parotid gland fistula
(b) Sialadenitis
(c) Chronic wound infection
(d) Frey?s syndrome
10.
First line hormone therapy for post-menopausal woman with metastatic carcinoma breast is:

(a) Tamoxifen
(b) Ovarian suppression by surgery
(c) Antiprogestins
(d) Anastrazole



11.
All of the following are major subtypes of breast cancer based on Gene array analysis
EXCEPT:

(a) Luminal A and Luminal B
(b) Triple negative
(c) Her-2 receptor positive
(d) Oestrogen receptor positive
12.
All of the following are sequelae of peptic ulcer surgery EXCEPT:

(a) Bilious vomiting
(b) Dumping syndrome
(c) Diarrhoea
(d) Increased appetite
13.
The Child-Turcotte-Pugh (CTP) score for quantifying the severity of chronic liver disease
includes all variables EXCEPT:

(a) Serum bilirubin
(b) Serum albumin
(c) Serum creatinine
(d) INR (International Normalised Ratio)
14.
?Chain of Lakes? appearance due to sacculation with intervening short strictures of pancreatic
duct is seen on:

(a) ERCP
(b) CECT abdomen
(c) Plain X-ray abdomen
(d) Ultrasonography
15.
Which one of the following statements is NOT correct regarding Pyogenic Liver Abscess?

(a) Anorexia, fever, malaise and right upper quadrant abdominal discomfort are the
most common presenting features
(b) It is more common in elderly, diabetics and immunocompromised patients
(c) Treatment is with oral antibiotics alone
(d) Streptococcus milleri and escherichia coli are the most common causative
organisms
16.
?Swiss cheese defects? are seen during laparoscopic repair of:

(a) Ventral hernia
(b) Inguinal hernia
(c) Obturator hernia
(d) Femoral hernia



17.
Which of the following are correct regarding splenic artery aneurysm?

1. Main arterial trunk is the common site
2. Palpable thrill can be felt
3. It is symptomless unless it ruptures

Select the correct answer using the code given below:

(a) 1 and 2 only
(b) 2 and 3 only
(c) 1 and 3 only
(d) 1, 2 and 3
18.
Valentino?s syndrome is:

(a) Pain on per-vaginal examination in pelvic abscess
(b) Pain over left shoulder in left hypochondriac collection
(c) Pain over left groin in perirenal collection
(d) Pain in right iliac fossa in perforated peptic ulcer
19.
Spontaneous bacterial peritonitis occurs due to:

(a) duodenal stump blowout
(b) peptic ulcer perforation
(c) acute bacterial infection of ascites
(d) infection via fallopian tubes
20.
Structure not forming boundaries of the ?Triangle of doom? seen during laparoscopic
inguinal hernia surgery dissection is:

(a) Vas deferens
(b) Inferior epigastric artery
(c) Spermatic cord vessels
(d) Peritoneum
21.
The term mid-line shift is associated with:

(a) Head injury
(b) Chest injury
(c) Abdominal injury
(d) Limb injury
22.
Which one of the following cranial nerves does NOT supply to the external ear?

(a) Cranial nerve V
(b) Cranial nerve VI
(c) Cranial nerve VII
(d) Cranial nerve IX

23.
Left Internal Mammary Artery (LIMA) has become the conduit of choice for Left Anterior
Descending (LAD) artery during coronary artery bypass grafting because:

(a) Long term patency rates are more than 98%
(b) It is close to LAD
(c) Atherosclerosis is never seen in this vessel
(d) It is very easy to harvest
24.
A 50-year old male with significant smoking history presented in the surgical emergency
with sudden severe breathlessness. Chest X-ray shows right sided Pneumothorax. The
appropriate management requires:

(a) Aspiration of air with 16-18 G cannula
(b) Right chest drain of size 8-14 Fr
(c) Oxygen by face mask
(d) Mechanical ventilation
25.
Which one of the following statements regarding Felon is NOT correct?

(a) There is infection of the finger tip between specialised fibrous septa
(b) It is a painless condition
(c) Incision and drainage is the treatment of choice
(d) It is common in diabetics
26.
The most common site for osteosarcoma is:

(a) Proximal femur
(b) Distal femur
(c) Proximal humerus
(d) Distal humerus
27.
Rapid Sequence Induction is indicated in:

(a) Emergency surgery for intestinal obstruction
(b) Elective open hernia surgery
(c) Cardiopulmonary bypass surgery
(d) Elective laparoscopic surgery

28.
Which one of the following statements about Compartment Syndrome is NOT correct?

(a) It is commonest in a closed fracture
(b) Pain is on active movement but not on passive movement of muscles
(c) Fasciotomy is the treatment of choice
(d) Volkmann?s Ischaemic contractive is a late complication




29.
Which one of the following is NOT the strength of ultrasound as a diagnostic modality?

(a) No radiation
(b) Short learning curve
(c) Inexpensive
(d) Allows dynamic studies to be done
30.
Which of the following statements regarding lymphoedema following breast cancer treatment
are correct?

1. Incidence has decreased due to rarely combined therapy of axillary LN dissection and
radiotherapy
2. Precipitating cause like LN metastasis is a major determinant
3. The condition is often painful
4. Oedematous limb is susceptible to bacterial infection

Select the correct answer using the code given below:

(a) 1, 2 and 3
(b) 2, 3 and 4
(c) 1, 3 and 4
(d) 1, 2 and 4
31.
Which one of the following is NOT a risk factor for development of venous thrombosis in
surgical patients?

(a) Age > 60 years
(b) Pregnancy
(c) Obesity (BMI > 30 kg/m
2
)
(d) Diabetes ( HbA1c > 7.5%)
32.
Which one of the following is NOT a complication of massive blood transfusion?

(a) Coagulopathy
(b) Hypocalcaemia
(c) Hyperthermia
(d) Hyperkalemia
33.
Stage III ?Pressure sore? is full thickness skin loss extending:

(a) into subcutaneous tissue but not through fascia
(b) through subcutaneous tissue into fascia
(c) through subcutaneous tissue into fascia and muscles
(d) through subcutaneous tissue into fascia, muscles and bone



34.
During subclavian vein puncture in a surgical ward suddenly a patient developed severe
breathlessness. On auscultation breath sound was absent and the ipsilateral chest was
tympanitic on percussion. The probable diagnosis is:

(a) Iatrogenic pneumothorax
(b) Introgenic hemothorax
(c) Spontaneous pneumothorax
(d) Tension pneumothorax
35.
Kohler?s disease is avascular necrosis of :

(a) Lunate
(b) Capitellum of humerus
(c) First metatarsal head
(d) Navicular
36.
Which one of the following is NOT true of Pyoderma gangrenosum?

(a) It is characterized by cutaneous ulceration with purple undermined edges
(b) It is often secondary to heightened immunological reactivity from another disease
process
(c) Cultures often show Gram positive Staphylococci
(d) Lesions generally respond to steroids
37.
Which of the following is NOT a tissue repair surgery for inguinal hernia repair?

(a) Bassini?s repair
(b) Shouldice repair
(c) Stoppa?s repair
(d) Desarda repair
38.
Which one of the following type of meshes is recommended for intraperitoneal use in
abdominal wall hernia?
(a) Light weight, porous meshes
(b) Heavy weight, porous meshes
(c) Absorbable meshes
(d) Tissue separating meshes
39.
Which one of the following is NOT correct regarding MEN-1 syndrome?

(a) It involves parathyroid glands
(b) It involves pancreas
(c) It involves pituitary gland
(d) It involves pineal gland

40.
Which one of the following is NOT an electronic information site in surgery?

(a) Pubmed
(b) Embase
(c) Cochrane library
(d) National medical library
41.
Which one of the following statements regarding pre-conceptional counseling is NOT
correct?

(a) It is needed only in selected complicated pregnancies
(b) It helps in early detection of risk factors
(c) It helps in reducing maternal morbidity and mortality
(d) It is a part of preventive medicine
42.
Consider the following statements regarding Non Stress Test (NST):

1. Reactive NST indicates a healthy fetus
2. NST is an observed association of fetal breathing with fetal movements
3. NST has a low false negative rate (< 1%) but high false positive rate (>50%)
4. Testing should be started at 20 weeks

Which of the statement(s) given above is/are correct?

(a) 1 and 3
(b) 2 only
(c) 3 only
(d) 1 and 4
43.
Which one of the following is a protective factor for endometrial hyperplasia?

(a) Diabetes
(b) Tamoxifen therapy
(c) Multiparity
(d) Delayed menopause
44.
A woman who is not breast feeding her newborn child is advised to use a contraceptive
method by:

(a) 3
rd
postpartum week
(b) 6
th
postpartum week
(c) 3
rd
postpartum month
(d) 6
th
postpartum month





45.
Pearl index for contraceptive effectiveness is calculated in terms of which of the following?

1. Pregnancy rate
2. Abortion rate
3. Hundred woman years
4. Thousand woman years

Select the correct answer using the code given below:

(a) 1 only
(b) 2 and 3
(c) 1, 2 and 4
(d) 1 and 3
46.
Indications for removal of IUDs are all EXCEPT:

(a) Perforation of uterus
(b) Cyclical menstrual bleeding
(c) Flaring up of salpingitis
(d) Pregnancy with IUD
47.
Contraindications for insertion of IUDs are all EXCEPT:

(a) Suspected pregnancy
(b) Trophoblastic disease
(c) Severe dysmenorrhea
(d) During cesarean section
48.
Which one of the following is NOT a contraindication for use of Mini pill?

(a) Pregnancy
(b) Breast feeding
(c) Thromboembolic disease
(d) History of breast cancer
49.
Which one of the following is the most commonly used surgical method/technique of female
sterilization as recommended by Government of India?

(a) Uchida technique
(b) Irving method
(c) Pomeroy?s method
(d) Madlener technique







50.
Which of the following is/are required for a registered medical practitioner to qualify for
performing Medical Termination of Pregnancy (MTP), as per revised rules of MTP Act?

1. Certified for assisting at least 15 MTP in an authorized centre
2. Diploma or degree in Obstetrics and Gynaecology
3. House surgeon training for 3 months in Obstetrics and Gynaecology
4. Certified training for 6 months in laparoscopic surgeries

Select the correct answer using the code given below:
(a) 1 only
(b) 2 only
(c) 1, 2 and 3
(d) 1, 2 and 4
51.
Which one of the following is NOT a support of uterus, preventing its descent?

(a) Endopelvic fascia
(b) Mackenrodt?s ligament
(c) Inguinal ligament
(d) Pubocervical ligament
52.
As per ICMR guidelines, which one of the following statements is true regarding effects of
COVID-19 on fetus according to current evidence?

(a) There is increased risk of early pregnancy loss
(b) COVID-19 virus is not teratogenic
(c) COVID-19 virus infection is an indication of MTP
(d) There is increased risk of fetal growth restriction
53.
As per ICMR guidelines, which one of the following statements is true regarding COVID-19
infection in pregnancy?

(a) Covid-19 pneumonia in pregnancy is more severe with poor recovery
(b) Pregnant women with heart disease are at higher risk
(c) Vaginal secretions always test positive for COVID-19 in pregnancy
(d) COVID-19 virus is secreted in breast milk
54.
Which one of the following is NOT a method of management of Deep Transverse Arrest with
the living fetus?

(a) Caesarean section
(b) Delivery by ventouse
(c) Delivery by application of forceps to the unrotated head
(d) Manual rotation and application of forceps





55.
Successful version of breech presentation is likely in case all of the following EXCEPT:

(a) Breech with extended legs
(b) Complete breech with sacroanterior position
(c) Non engaged breech
(d) Adequate amniotic fluid
56.
Implantation of a fertilised ovum occurs on which day following fertilisation?

(a) Day 6
(b) Day 10
(c) Day 14
(d) Day 20
57.
During total abdominal hysterectomy the ureter is likely to undergo injury or ligation during
the following steps EXCEPT:

(a) During division and ligation of the round ligaments
(b) During division and ligation of infundibulopelvic ligaments
(c) During division and ligation of mackenrodt?s and uterosacral ligaments
(d) At the vaginal angles while incising the vagina to remove the cervix with the
uterus
58.
The net effect of antenatal care has been the following EXCEPT:

(a) Reduction in maternal mortality
(b) Reduction in perinatal mortality
(c) Reduction in the incidence of institutional delivery
(d) Reduction in maternal morbidity
59.
Which one of the following is NOT a component of active phase in the partograph?

(a) Acceleration phase
(b) Phase of maximum slope
(c) Phase of deceleration
(d) Phase of expulsion
60.
From medicolegal point of view which one of the following is NOT a sign of previous child
birth?

(a) Perineum is lax and there is evidence of scarring
(b) Introitus is gaping and there is presence of carunculae myrtiformis
(c) Abdomen is lax and loose with striae and linea alba
(d) Conical cervix with round external os





61.
The components of partograph are all EXCEPT:

(a) Time
(b) Fetal heart rate
(c) Maternal respiratory rate
(d) Maternal urine analysis
62.
Which of the following information are provided by partograph?

1. Colour of liquor
2. Uterine contractions with duration and frequency
3. Dilatation of cervix

Select the correct answer using the code given below:

(a) 1 and 2 only
(b) 2 and 3 only
(c) 1 and 3 only
(d) 1, 2 and 3
63.
Intraoperative recognition of ureter is by which of the following features?

1. Transparent tubular appearance
2. Pale glistening appearance
3. Longitudinal vessels on surface
4. Circumferential vessels on surface

Select the correct answer using the code given below:

(a) 1 and 3
(b) 2 and 4
(c) 2 and 3
(d) 1 and 4
64.
Hysterosalpingography (HSG) is least helpful in detecting which of the following?

(a) Tubal patency
(b) Pelvic adhesions
(c) Asherman syndrome
(d) Congenital uterine anomaly









65.
Which of the following are characteristics of Trichomonas vaginitis?

1. Presence of greenish frothy discharge
2. Vaginal pH > 4.5
3. Presence of clue cells in microscopic examination
4. Strawberry spots on the vaginal mucosa

Select the correct answer using the code given below:

(a) 1, 2 and 3
(b) 1, 2 and 4
(c) 2, 3 and 4
(d) 1, 3 and 4
66.
Tumor marker of epithelial ovarian carcinoma is:

(a) Ca.125
(b) Alpha feto protein
(c) Beta HCG
(d) LDH
67.
The most common site of cervical cancer is:

(a) Endocervix
(b) Ectocervix
(c) Transformation zone
(d) Isthmus
68.
The placenta synthesizes all EXCEPT:

(a) Oestriol
(b) Corticotrophin releasing hormone
(c) PAPP-A(Pregnancy Associated Plasma Protein A)
(d) Dehydroepiandrosterone
69.
Withdrawal bleeding following administration of progesterone in a case of secondary
amenorrhea indicates all EXCEPT:

(a) Absence of pregnancy
(b) Production endogenous estrogen
(c) Endometrium is responsive to estrogen
(d) Defect in pituitary gland
70.
Monilial vaginitis is commonly associated with all EXCEPT:

(a) Prolonged antibiotic therapy
(b) Diabetes Mellitus
(c) Treatment of malaria with chloroquine
(d) Pregnancy
FirstRanker.com - FirstRanker's Choice
Combined Medical Services Examination-2020
Paper-II

1.
Indications for fasciotomy in compartment syndrome include all EXCEPT:

(a) Distal sensory disturbance
(b) Compartment pressure > 30 mm Hg
(c) Pain on passive movement of affected muscles
(d) Palpable distal pulses
2.
Which one of the following statements is NOT correct regarding Necrotising Soft Tissue
infections?

(a) Crepitus, skin blistering and focal skin gangrene are typical presenting features
(b) They are monomicrobial in nature
(c) Treatment consists of wide local excision and appropriate antibiotics
(d) Tissue biopsy is required for culture and diagnosis
3.
Which type of surgery is laparoscopic cholecystectomy classified as?

(a) Clean
(b) Clean contaminated
(c) Contaminated
(d) Dirty
4.
A 22-year female has presented with a history of malaise, cough, alternating constipation and
diarrhoea with intermittent abdominal pain for last 6 months. She also complains of
abdominal distension for last 2 days. On examination her abdomen has a doughy feel along
with an ill defined mass over the right lower quadrant. She is most likely suffering from:

(a) Appendicular lump
(b) Ileocaecal tuberculosis
(c) Carcinoma caecum
(d) Ovarian mass
5.
Consider the following statements regarding needle stick injuries:

1. Injured part should be washed under running water
2. Dominant index finger is the commonest site for needle stick injury
3. All needle stick injuries should be reported
4. Hepatitis/HIV testing should be done after needle stick injury

Which of the statements given above are correct?

(a) 1, 2 and 4
(b) 1, 2 and 3
(c) 1, 3 and 4
(d) 2, 3 and 4




6.
Consider the following statements regarding claudication:

1. It is a marker for silent coronary disease
2. Structured exercise program ( 2 hours per week for 3 months) leads to
improvement in symptoms
3. Diabetes mellitus increases the risk and severity of claudication
4. Beta blockers may exacerbate claudication

Which of the above statements are correct?

(a) 1 and 2 only
(b) 1, 3 and 4 only
(c) 2, 3 and 4 only
(d) 1, 2, 3 and 4
7.
A 50-year old lady underwent uneventful bariatric surgery for morbid obesity. On the third
post operative day, she develops breathless and pulmonary embolism is suspected. The next
investigation to confirm the diagnosis will be:

(a) Echocardiography
(b) Duplex venography
(c) CT pulmonary angiography
(d) MR angiography
8.
Medical management of thyrotoxic crisis includes all of the following EXCEPT:

(a) IV fluids
(b) IV propanolol
(c) IV hydrocortisone
(d) IV antibiotics
9.
A patient operated for a parotid gland tumour developed symptoms of sweating and
erytherma (flushing) over the region of surgical excision while eating. The probable
diagnosis is:

(a) Parotid gland fistula
(b) Sialadenitis
(c) Chronic wound infection
(d) Frey?s syndrome
10.
First line hormone therapy for post-menopausal woman with metastatic carcinoma breast is:

(a) Tamoxifen
(b) Ovarian suppression by surgery
(c) Antiprogestins
(d) Anastrazole



11.
All of the following are major subtypes of breast cancer based on Gene array analysis
EXCEPT:

(a) Luminal A and Luminal B
(b) Triple negative
(c) Her-2 receptor positive
(d) Oestrogen receptor positive
12.
All of the following are sequelae of peptic ulcer surgery EXCEPT:

(a) Bilious vomiting
(b) Dumping syndrome
(c) Diarrhoea
(d) Increased appetite
13.
The Child-Turcotte-Pugh (CTP) score for quantifying the severity of chronic liver disease
includes all variables EXCEPT:

(a) Serum bilirubin
(b) Serum albumin
(c) Serum creatinine
(d) INR (International Normalised Ratio)
14.
?Chain of Lakes? appearance due to sacculation with intervening short strictures of pancreatic
duct is seen on:

(a) ERCP
(b) CECT abdomen
(c) Plain X-ray abdomen
(d) Ultrasonography
15.
Which one of the following statements is NOT correct regarding Pyogenic Liver Abscess?

(a) Anorexia, fever, malaise and right upper quadrant abdominal discomfort are the
most common presenting features
(b) It is more common in elderly, diabetics and immunocompromised patients
(c) Treatment is with oral antibiotics alone
(d) Streptococcus milleri and escherichia coli are the most common causative
organisms
16.
?Swiss cheese defects? are seen during laparoscopic repair of:

(a) Ventral hernia
(b) Inguinal hernia
(c) Obturator hernia
(d) Femoral hernia



17.
Which of the following are correct regarding splenic artery aneurysm?

1. Main arterial trunk is the common site
2. Palpable thrill can be felt
3. It is symptomless unless it ruptures

Select the correct answer using the code given below:

(a) 1 and 2 only
(b) 2 and 3 only
(c) 1 and 3 only
(d) 1, 2 and 3
18.
Valentino?s syndrome is:

(a) Pain on per-vaginal examination in pelvic abscess
(b) Pain over left shoulder in left hypochondriac collection
(c) Pain over left groin in perirenal collection
(d) Pain in right iliac fossa in perforated peptic ulcer
19.
Spontaneous bacterial peritonitis occurs due to:

(a) duodenal stump blowout
(b) peptic ulcer perforation
(c) acute bacterial infection of ascites
(d) infection via fallopian tubes
20.
Structure not forming boundaries of the ?Triangle of doom? seen during laparoscopic
inguinal hernia surgery dissection is:

(a) Vas deferens
(b) Inferior epigastric artery
(c) Spermatic cord vessels
(d) Peritoneum
21.
The term mid-line shift is associated with:

(a) Head injury
(b) Chest injury
(c) Abdominal injury
(d) Limb injury
22.
Which one of the following cranial nerves does NOT supply to the external ear?

(a) Cranial nerve V
(b) Cranial nerve VI
(c) Cranial nerve VII
(d) Cranial nerve IX

23.
Left Internal Mammary Artery (LIMA) has become the conduit of choice for Left Anterior
Descending (LAD) artery during coronary artery bypass grafting because:

(a) Long term patency rates are more than 98%
(b) It is close to LAD
(c) Atherosclerosis is never seen in this vessel
(d) It is very easy to harvest
24.
A 50-year old male with significant smoking history presented in the surgical emergency
with sudden severe breathlessness. Chest X-ray shows right sided Pneumothorax. The
appropriate management requires:

(a) Aspiration of air with 16-18 G cannula
(b) Right chest drain of size 8-14 Fr
(c) Oxygen by face mask
(d) Mechanical ventilation
25.
Which one of the following statements regarding Felon is NOT correct?

(a) There is infection of the finger tip between specialised fibrous septa
(b) It is a painless condition
(c) Incision and drainage is the treatment of choice
(d) It is common in diabetics
26.
The most common site for osteosarcoma is:

(a) Proximal femur
(b) Distal femur
(c) Proximal humerus
(d) Distal humerus
27.
Rapid Sequence Induction is indicated in:

(a) Emergency surgery for intestinal obstruction
(b) Elective open hernia surgery
(c) Cardiopulmonary bypass surgery
(d) Elective laparoscopic surgery

28.
Which one of the following statements about Compartment Syndrome is NOT correct?

(a) It is commonest in a closed fracture
(b) Pain is on active movement but not on passive movement of muscles
(c) Fasciotomy is the treatment of choice
(d) Volkmann?s Ischaemic contractive is a late complication




29.
Which one of the following is NOT the strength of ultrasound as a diagnostic modality?

(a) No radiation
(b) Short learning curve
(c) Inexpensive
(d) Allows dynamic studies to be done
30.
Which of the following statements regarding lymphoedema following breast cancer treatment
are correct?

1. Incidence has decreased due to rarely combined therapy of axillary LN dissection and
radiotherapy
2. Precipitating cause like LN metastasis is a major determinant
3. The condition is often painful
4. Oedematous limb is susceptible to bacterial infection

Select the correct answer using the code given below:

(a) 1, 2 and 3
(b) 2, 3 and 4
(c) 1, 3 and 4
(d) 1, 2 and 4
31.
Which one of the following is NOT a risk factor for development of venous thrombosis in
surgical patients?

(a) Age > 60 years
(b) Pregnancy
(c) Obesity (BMI > 30 kg/m
2
)
(d) Diabetes ( HbA1c > 7.5%)
32.
Which one of the following is NOT a complication of massive blood transfusion?

(a) Coagulopathy
(b) Hypocalcaemia
(c) Hyperthermia
(d) Hyperkalemia
33.
Stage III ?Pressure sore? is full thickness skin loss extending:

(a) into subcutaneous tissue but not through fascia
(b) through subcutaneous tissue into fascia
(c) through subcutaneous tissue into fascia and muscles
(d) through subcutaneous tissue into fascia, muscles and bone



34.
During subclavian vein puncture in a surgical ward suddenly a patient developed severe
breathlessness. On auscultation breath sound was absent and the ipsilateral chest was
tympanitic on percussion. The probable diagnosis is:

(a) Iatrogenic pneumothorax
(b) Introgenic hemothorax
(c) Spontaneous pneumothorax
(d) Tension pneumothorax
35.
Kohler?s disease is avascular necrosis of :

(a) Lunate
(b) Capitellum of humerus
(c) First metatarsal head
(d) Navicular
36.
Which one of the following is NOT true of Pyoderma gangrenosum?

(a) It is characterized by cutaneous ulceration with purple undermined edges
(b) It is often secondary to heightened immunological reactivity from another disease
process
(c) Cultures often show Gram positive Staphylococci
(d) Lesions generally respond to steroids
37.
Which of the following is NOT a tissue repair surgery for inguinal hernia repair?

(a) Bassini?s repair
(b) Shouldice repair
(c) Stoppa?s repair
(d) Desarda repair
38.
Which one of the following type of meshes is recommended for intraperitoneal use in
abdominal wall hernia?
(a) Light weight, porous meshes
(b) Heavy weight, porous meshes
(c) Absorbable meshes
(d) Tissue separating meshes
39.
Which one of the following is NOT correct regarding MEN-1 syndrome?

(a) It involves parathyroid glands
(b) It involves pancreas
(c) It involves pituitary gland
(d) It involves pineal gland

40.
Which one of the following is NOT an electronic information site in surgery?

(a) Pubmed
(b) Embase
(c) Cochrane library
(d) National medical library
41.
Which one of the following statements regarding pre-conceptional counseling is NOT
correct?

(a) It is needed only in selected complicated pregnancies
(b) It helps in early detection of risk factors
(c) It helps in reducing maternal morbidity and mortality
(d) It is a part of preventive medicine
42.
Consider the following statements regarding Non Stress Test (NST):

1. Reactive NST indicates a healthy fetus
2. NST is an observed association of fetal breathing with fetal movements
3. NST has a low false negative rate (< 1%) but high false positive rate (>50%)
4. Testing should be started at 20 weeks

Which of the statement(s) given above is/are correct?

(a) 1 and 3
(b) 2 only
(c) 3 only
(d) 1 and 4
43.
Which one of the following is a protective factor for endometrial hyperplasia?

(a) Diabetes
(b) Tamoxifen therapy
(c) Multiparity
(d) Delayed menopause
44.
A woman who is not breast feeding her newborn child is advised to use a contraceptive
method by:

(a) 3
rd
postpartum week
(b) 6
th
postpartum week
(c) 3
rd
postpartum month
(d) 6
th
postpartum month





45.
Pearl index for contraceptive effectiveness is calculated in terms of which of the following?

1. Pregnancy rate
2. Abortion rate
3. Hundred woman years
4. Thousand woman years

Select the correct answer using the code given below:

(a) 1 only
(b) 2 and 3
(c) 1, 2 and 4
(d) 1 and 3
46.
Indications for removal of IUDs are all EXCEPT:

(a) Perforation of uterus
(b) Cyclical menstrual bleeding
(c) Flaring up of salpingitis
(d) Pregnancy with IUD
47.
Contraindications for insertion of IUDs are all EXCEPT:

(a) Suspected pregnancy
(b) Trophoblastic disease
(c) Severe dysmenorrhea
(d) During cesarean section
48.
Which one of the following is NOT a contraindication for use of Mini pill?

(a) Pregnancy
(b) Breast feeding
(c) Thromboembolic disease
(d) History of breast cancer
49.
Which one of the following is the most commonly used surgical method/technique of female
sterilization as recommended by Government of India?

(a) Uchida technique
(b) Irving method
(c) Pomeroy?s method
(d) Madlener technique







50.
Which of the following is/are required for a registered medical practitioner to qualify for
performing Medical Termination of Pregnancy (MTP), as per revised rules of MTP Act?

1. Certified for assisting at least 15 MTP in an authorized centre
2. Diploma or degree in Obstetrics and Gynaecology
3. House surgeon training for 3 months in Obstetrics and Gynaecology
4. Certified training for 6 months in laparoscopic surgeries

Select the correct answer using the code given below:
(a) 1 only
(b) 2 only
(c) 1, 2 and 3
(d) 1, 2 and 4
51.
Which one of the following is NOT a support of uterus, preventing its descent?

(a) Endopelvic fascia
(b) Mackenrodt?s ligament
(c) Inguinal ligament
(d) Pubocervical ligament
52.
As per ICMR guidelines, which one of the following statements is true regarding effects of
COVID-19 on fetus according to current evidence?

(a) There is increased risk of early pregnancy loss
(b) COVID-19 virus is not teratogenic
(c) COVID-19 virus infection is an indication of MTP
(d) There is increased risk of fetal growth restriction
53.
As per ICMR guidelines, which one of the following statements is true regarding COVID-19
infection in pregnancy?

(a) Covid-19 pneumonia in pregnancy is more severe with poor recovery
(b) Pregnant women with heart disease are at higher risk
(c) Vaginal secretions always test positive for COVID-19 in pregnancy
(d) COVID-19 virus is secreted in breast milk
54.
Which one of the following is NOT a method of management of Deep Transverse Arrest with
the living fetus?

(a) Caesarean section
(b) Delivery by ventouse
(c) Delivery by application of forceps to the unrotated head
(d) Manual rotation and application of forceps





55.
Successful version of breech presentation is likely in case all of the following EXCEPT:

(a) Breech with extended legs
(b) Complete breech with sacroanterior position
(c) Non engaged breech
(d) Adequate amniotic fluid
56.
Implantation of a fertilised ovum occurs on which day following fertilisation?

(a) Day 6
(b) Day 10
(c) Day 14
(d) Day 20
57.
During total abdominal hysterectomy the ureter is likely to undergo injury or ligation during
the following steps EXCEPT:

(a) During division and ligation of the round ligaments
(b) During division and ligation of infundibulopelvic ligaments
(c) During division and ligation of mackenrodt?s and uterosacral ligaments
(d) At the vaginal angles while incising the vagina to remove the cervix with the
uterus
58.
The net effect of antenatal care has been the following EXCEPT:

(a) Reduction in maternal mortality
(b) Reduction in perinatal mortality
(c) Reduction in the incidence of institutional delivery
(d) Reduction in maternal morbidity
59.
Which one of the following is NOT a component of active phase in the partograph?

(a) Acceleration phase
(b) Phase of maximum slope
(c) Phase of deceleration
(d) Phase of expulsion
60.
From medicolegal point of view which one of the following is NOT a sign of previous child
birth?

(a) Perineum is lax and there is evidence of scarring
(b) Introitus is gaping and there is presence of carunculae myrtiformis
(c) Abdomen is lax and loose with striae and linea alba
(d) Conical cervix with round external os





61.
The components of partograph are all EXCEPT:

(a) Time
(b) Fetal heart rate
(c) Maternal respiratory rate
(d) Maternal urine analysis
62.
Which of the following information are provided by partograph?

1. Colour of liquor
2. Uterine contractions with duration and frequency
3. Dilatation of cervix

Select the correct answer using the code given below:

(a) 1 and 2 only
(b) 2 and 3 only
(c) 1 and 3 only
(d) 1, 2 and 3
63.
Intraoperative recognition of ureter is by which of the following features?

1. Transparent tubular appearance
2. Pale glistening appearance
3. Longitudinal vessels on surface
4. Circumferential vessels on surface

Select the correct answer using the code given below:

(a) 1 and 3
(b) 2 and 4
(c) 2 and 3
(d) 1 and 4
64.
Hysterosalpingography (HSG) is least helpful in detecting which of the following?

(a) Tubal patency
(b) Pelvic adhesions
(c) Asherman syndrome
(d) Congenital uterine anomaly









65.
Which of the following are characteristics of Trichomonas vaginitis?

1. Presence of greenish frothy discharge
2. Vaginal pH > 4.5
3. Presence of clue cells in microscopic examination
4. Strawberry spots on the vaginal mucosa

Select the correct answer using the code given below:

(a) 1, 2 and 3
(b) 1, 2 and 4
(c) 2, 3 and 4
(d) 1, 3 and 4
66.
Tumor marker of epithelial ovarian carcinoma is:

(a) Ca.125
(b) Alpha feto protein
(c) Beta HCG
(d) LDH
67.
The most common site of cervical cancer is:

(a) Endocervix
(b) Ectocervix
(c) Transformation zone
(d) Isthmus
68.
The placenta synthesizes all EXCEPT:

(a) Oestriol
(b) Corticotrophin releasing hormone
(c) PAPP-A(Pregnancy Associated Plasma Protein A)
(d) Dehydroepiandrosterone
69.
Withdrawal bleeding following administration of progesterone in a case of secondary
amenorrhea indicates all EXCEPT:

(a) Absence of pregnancy
(b) Production endogenous estrogen
(c) Endometrium is responsive to estrogen
(d) Defect in pituitary gland
70.
Monilial vaginitis is commonly associated with all EXCEPT:

(a) Prolonged antibiotic therapy
(b) Diabetes Mellitus
(c) Treatment of malaria with chloroquine
(d) Pregnancy
71.
Which one of the following is NOT a risk factor for the development of placenta previa?

(a) Maternal age
(b) Smoking
(c) Previous caesarean section
(d) Maternal anaemia
72.
Common clinical presentations of moderate to severe abruption are all EXCEPT:

(a) Uterine tenderness
(b) Fetal distress
(c) Unexplained pre term labour
(d) Prolonged labour
73.
Common trisomies resulting in spontaneous abortion are all EXCEPT:

(a) Trisomy 21
(b) Trisomy 18
(c) Trisomy 16
(d) Trisomy 1
74.
The initial prevention strategy for antiphospholipid syndrome will be:

1. Steroids
2. Heparin
3. Low dose aspirin
4. Progesterone support

Which of the above is/are correct?

(a) 2 and 3
(b) 3 and 4
(c) 3 only
(d) 1 and 4
75.
Diagnostic criteria for PCOD are:

1. Oligo/amenorrohoea
2. Hyperandrogenism
3. Polycystic ovaries on ultrasound

Which of the above are correct?

(a) 1 and 2 only
(b) 2 and 3 only
(c) 1 and 3 only
(d) 1, 2 and 3


FirstRanker.com - FirstRanker's Choice
Combined Medical Services Examination-2020
Paper-II

1.
Indications for fasciotomy in compartment syndrome include all EXCEPT:

(a) Distal sensory disturbance
(b) Compartment pressure > 30 mm Hg
(c) Pain on passive movement of affected muscles
(d) Palpable distal pulses
2.
Which one of the following statements is NOT correct regarding Necrotising Soft Tissue
infections?

(a) Crepitus, skin blistering and focal skin gangrene are typical presenting features
(b) They are monomicrobial in nature
(c) Treatment consists of wide local excision and appropriate antibiotics
(d) Tissue biopsy is required for culture and diagnosis
3.
Which type of surgery is laparoscopic cholecystectomy classified as?

(a) Clean
(b) Clean contaminated
(c) Contaminated
(d) Dirty
4.
A 22-year female has presented with a history of malaise, cough, alternating constipation and
diarrhoea with intermittent abdominal pain for last 6 months. She also complains of
abdominal distension for last 2 days. On examination her abdomen has a doughy feel along
with an ill defined mass over the right lower quadrant. She is most likely suffering from:

(a) Appendicular lump
(b) Ileocaecal tuberculosis
(c) Carcinoma caecum
(d) Ovarian mass
5.
Consider the following statements regarding needle stick injuries:

1. Injured part should be washed under running water
2. Dominant index finger is the commonest site for needle stick injury
3. All needle stick injuries should be reported
4. Hepatitis/HIV testing should be done after needle stick injury

Which of the statements given above are correct?

(a) 1, 2 and 4
(b) 1, 2 and 3
(c) 1, 3 and 4
(d) 2, 3 and 4




6.
Consider the following statements regarding claudication:

1. It is a marker for silent coronary disease
2. Structured exercise program ( 2 hours per week for 3 months) leads to
improvement in symptoms
3. Diabetes mellitus increases the risk and severity of claudication
4. Beta blockers may exacerbate claudication

Which of the above statements are correct?

(a) 1 and 2 only
(b) 1, 3 and 4 only
(c) 2, 3 and 4 only
(d) 1, 2, 3 and 4
7.
A 50-year old lady underwent uneventful bariatric surgery for morbid obesity. On the third
post operative day, she develops breathless and pulmonary embolism is suspected. The next
investigation to confirm the diagnosis will be:

(a) Echocardiography
(b) Duplex venography
(c) CT pulmonary angiography
(d) MR angiography
8.
Medical management of thyrotoxic crisis includes all of the following EXCEPT:

(a) IV fluids
(b) IV propanolol
(c) IV hydrocortisone
(d) IV antibiotics
9.
A patient operated for a parotid gland tumour developed symptoms of sweating and
erytherma (flushing) over the region of surgical excision while eating. The probable
diagnosis is:

(a) Parotid gland fistula
(b) Sialadenitis
(c) Chronic wound infection
(d) Frey?s syndrome
10.
First line hormone therapy for post-menopausal woman with metastatic carcinoma breast is:

(a) Tamoxifen
(b) Ovarian suppression by surgery
(c) Antiprogestins
(d) Anastrazole



11.
All of the following are major subtypes of breast cancer based on Gene array analysis
EXCEPT:

(a) Luminal A and Luminal B
(b) Triple negative
(c) Her-2 receptor positive
(d) Oestrogen receptor positive
12.
All of the following are sequelae of peptic ulcer surgery EXCEPT:

(a) Bilious vomiting
(b) Dumping syndrome
(c) Diarrhoea
(d) Increased appetite
13.
The Child-Turcotte-Pugh (CTP) score for quantifying the severity of chronic liver disease
includes all variables EXCEPT:

(a) Serum bilirubin
(b) Serum albumin
(c) Serum creatinine
(d) INR (International Normalised Ratio)
14.
?Chain of Lakes? appearance due to sacculation with intervening short strictures of pancreatic
duct is seen on:

(a) ERCP
(b) CECT abdomen
(c) Plain X-ray abdomen
(d) Ultrasonography
15.
Which one of the following statements is NOT correct regarding Pyogenic Liver Abscess?

(a) Anorexia, fever, malaise and right upper quadrant abdominal discomfort are the
most common presenting features
(b) It is more common in elderly, diabetics and immunocompromised patients
(c) Treatment is with oral antibiotics alone
(d) Streptococcus milleri and escherichia coli are the most common causative
organisms
16.
?Swiss cheese defects? are seen during laparoscopic repair of:

(a) Ventral hernia
(b) Inguinal hernia
(c) Obturator hernia
(d) Femoral hernia



17.
Which of the following are correct regarding splenic artery aneurysm?

1. Main arterial trunk is the common site
2. Palpable thrill can be felt
3. It is symptomless unless it ruptures

Select the correct answer using the code given below:

(a) 1 and 2 only
(b) 2 and 3 only
(c) 1 and 3 only
(d) 1, 2 and 3
18.
Valentino?s syndrome is:

(a) Pain on per-vaginal examination in pelvic abscess
(b) Pain over left shoulder in left hypochondriac collection
(c) Pain over left groin in perirenal collection
(d) Pain in right iliac fossa in perforated peptic ulcer
19.
Spontaneous bacterial peritonitis occurs due to:

(a) duodenal stump blowout
(b) peptic ulcer perforation
(c) acute bacterial infection of ascites
(d) infection via fallopian tubes
20.
Structure not forming boundaries of the ?Triangle of doom? seen during laparoscopic
inguinal hernia surgery dissection is:

(a) Vas deferens
(b) Inferior epigastric artery
(c) Spermatic cord vessels
(d) Peritoneum
21.
The term mid-line shift is associated with:

(a) Head injury
(b) Chest injury
(c) Abdominal injury
(d) Limb injury
22.
Which one of the following cranial nerves does NOT supply to the external ear?

(a) Cranial nerve V
(b) Cranial nerve VI
(c) Cranial nerve VII
(d) Cranial nerve IX

23.
Left Internal Mammary Artery (LIMA) has become the conduit of choice for Left Anterior
Descending (LAD) artery during coronary artery bypass grafting because:

(a) Long term patency rates are more than 98%
(b) It is close to LAD
(c) Atherosclerosis is never seen in this vessel
(d) It is very easy to harvest
24.
A 50-year old male with significant smoking history presented in the surgical emergency
with sudden severe breathlessness. Chest X-ray shows right sided Pneumothorax. The
appropriate management requires:

(a) Aspiration of air with 16-18 G cannula
(b) Right chest drain of size 8-14 Fr
(c) Oxygen by face mask
(d) Mechanical ventilation
25.
Which one of the following statements regarding Felon is NOT correct?

(a) There is infection of the finger tip between specialised fibrous septa
(b) It is a painless condition
(c) Incision and drainage is the treatment of choice
(d) It is common in diabetics
26.
The most common site for osteosarcoma is:

(a) Proximal femur
(b) Distal femur
(c) Proximal humerus
(d) Distal humerus
27.
Rapid Sequence Induction is indicated in:

(a) Emergency surgery for intestinal obstruction
(b) Elective open hernia surgery
(c) Cardiopulmonary bypass surgery
(d) Elective laparoscopic surgery

28.
Which one of the following statements about Compartment Syndrome is NOT correct?

(a) It is commonest in a closed fracture
(b) Pain is on active movement but not on passive movement of muscles
(c) Fasciotomy is the treatment of choice
(d) Volkmann?s Ischaemic contractive is a late complication




29.
Which one of the following is NOT the strength of ultrasound as a diagnostic modality?

(a) No radiation
(b) Short learning curve
(c) Inexpensive
(d) Allows dynamic studies to be done
30.
Which of the following statements regarding lymphoedema following breast cancer treatment
are correct?

1. Incidence has decreased due to rarely combined therapy of axillary LN dissection and
radiotherapy
2. Precipitating cause like LN metastasis is a major determinant
3. The condition is often painful
4. Oedematous limb is susceptible to bacterial infection

Select the correct answer using the code given below:

(a) 1, 2 and 3
(b) 2, 3 and 4
(c) 1, 3 and 4
(d) 1, 2 and 4
31.
Which one of the following is NOT a risk factor for development of venous thrombosis in
surgical patients?

(a) Age > 60 years
(b) Pregnancy
(c) Obesity (BMI > 30 kg/m
2
)
(d) Diabetes ( HbA1c > 7.5%)
32.
Which one of the following is NOT a complication of massive blood transfusion?

(a) Coagulopathy
(b) Hypocalcaemia
(c) Hyperthermia
(d) Hyperkalemia
33.
Stage III ?Pressure sore? is full thickness skin loss extending:

(a) into subcutaneous tissue but not through fascia
(b) through subcutaneous tissue into fascia
(c) through subcutaneous tissue into fascia and muscles
(d) through subcutaneous tissue into fascia, muscles and bone



34.
During subclavian vein puncture in a surgical ward suddenly a patient developed severe
breathlessness. On auscultation breath sound was absent and the ipsilateral chest was
tympanitic on percussion. The probable diagnosis is:

(a) Iatrogenic pneumothorax
(b) Introgenic hemothorax
(c) Spontaneous pneumothorax
(d) Tension pneumothorax
35.
Kohler?s disease is avascular necrosis of :

(a) Lunate
(b) Capitellum of humerus
(c) First metatarsal head
(d) Navicular
36.
Which one of the following is NOT true of Pyoderma gangrenosum?

(a) It is characterized by cutaneous ulceration with purple undermined edges
(b) It is often secondary to heightened immunological reactivity from another disease
process
(c) Cultures often show Gram positive Staphylococci
(d) Lesions generally respond to steroids
37.
Which of the following is NOT a tissue repair surgery for inguinal hernia repair?

(a) Bassini?s repair
(b) Shouldice repair
(c) Stoppa?s repair
(d) Desarda repair
38.
Which one of the following type of meshes is recommended for intraperitoneal use in
abdominal wall hernia?
(a) Light weight, porous meshes
(b) Heavy weight, porous meshes
(c) Absorbable meshes
(d) Tissue separating meshes
39.
Which one of the following is NOT correct regarding MEN-1 syndrome?

(a) It involves parathyroid glands
(b) It involves pancreas
(c) It involves pituitary gland
(d) It involves pineal gland

40.
Which one of the following is NOT an electronic information site in surgery?

(a) Pubmed
(b) Embase
(c) Cochrane library
(d) National medical library
41.
Which one of the following statements regarding pre-conceptional counseling is NOT
correct?

(a) It is needed only in selected complicated pregnancies
(b) It helps in early detection of risk factors
(c) It helps in reducing maternal morbidity and mortality
(d) It is a part of preventive medicine
42.
Consider the following statements regarding Non Stress Test (NST):

1. Reactive NST indicates a healthy fetus
2. NST is an observed association of fetal breathing with fetal movements
3. NST has a low false negative rate (< 1%) but high false positive rate (>50%)
4. Testing should be started at 20 weeks

Which of the statement(s) given above is/are correct?

(a) 1 and 3
(b) 2 only
(c) 3 only
(d) 1 and 4
43.
Which one of the following is a protective factor for endometrial hyperplasia?

(a) Diabetes
(b) Tamoxifen therapy
(c) Multiparity
(d) Delayed menopause
44.
A woman who is not breast feeding her newborn child is advised to use a contraceptive
method by:

(a) 3
rd
postpartum week
(b) 6
th
postpartum week
(c) 3
rd
postpartum month
(d) 6
th
postpartum month





45.
Pearl index for contraceptive effectiveness is calculated in terms of which of the following?

1. Pregnancy rate
2. Abortion rate
3. Hundred woman years
4. Thousand woman years

Select the correct answer using the code given below:

(a) 1 only
(b) 2 and 3
(c) 1, 2 and 4
(d) 1 and 3
46.
Indications for removal of IUDs are all EXCEPT:

(a) Perforation of uterus
(b) Cyclical menstrual bleeding
(c) Flaring up of salpingitis
(d) Pregnancy with IUD
47.
Contraindications for insertion of IUDs are all EXCEPT:

(a) Suspected pregnancy
(b) Trophoblastic disease
(c) Severe dysmenorrhea
(d) During cesarean section
48.
Which one of the following is NOT a contraindication for use of Mini pill?

(a) Pregnancy
(b) Breast feeding
(c) Thromboembolic disease
(d) History of breast cancer
49.
Which one of the following is the most commonly used surgical method/technique of female
sterilization as recommended by Government of India?

(a) Uchida technique
(b) Irving method
(c) Pomeroy?s method
(d) Madlener technique







50.
Which of the following is/are required for a registered medical practitioner to qualify for
performing Medical Termination of Pregnancy (MTP), as per revised rules of MTP Act?

1. Certified for assisting at least 15 MTP in an authorized centre
2. Diploma or degree in Obstetrics and Gynaecology
3. House surgeon training for 3 months in Obstetrics and Gynaecology
4. Certified training for 6 months in laparoscopic surgeries

Select the correct answer using the code given below:
(a) 1 only
(b) 2 only
(c) 1, 2 and 3
(d) 1, 2 and 4
51.
Which one of the following is NOT a support of uterus, preventing its descent?

(a) Endopelvic fascia
(b) Mackenrodt?s ligament
(c) Inguinal ligament
(d) Pubocervical ligament
52.
As per ICMR guidelines, which one of the following statements is true regarding effects of
COVID-19 on fetus according to current evidence?

(a) There is increased risk of early pregnancy loss
(b) COVID-19 virus is not teratogenic
(c) COVID-19 virus infection is an indication of MTP
(d) There is increased risk of fetal growth restriction
53.
As per ICMR guidelines, which one of the following statements is true regarding COVID-19
infection in pregnancy?

(a) Covid-19 pneumonia in pregnancy is more severe with poor recovery
(b) Pregnant women with heart disease are at higher risk
(c) Vaginal secretions always test positive for COVID-19 in pregnancy
(d) COVID-19 virus is secreted in breast milk
54.
Which one of the following is NOT a method of management of Deep Transverse Arrest with
the living fetus?

(a) Caesarean section
(b) Delivery by ventouse
(c) Delivery by application of forceps to the unrotated head
(d) Manual rotation and application of forceps





55.
Successful version of breech presentation is likely in case all of the following EXCEPT:

(a) Breech with extended legs
(b) Complete breech with sacroanterior position
(c) Non engaged breech
(d) Adequate amniotic fluid
56.
Implantation of a fertilised ovum occurs on which day following fertilisation?

(a) Day 6
(b) Day 10
(c) Day 14
(d) Day 20
57.
During total abdominal hysterectomy the ureter is likely to undergo injury or ligation during
the following steps EXCEPT:

(a) During division and ligation of the round ligaments
(b) During division and ligation of infundibulopelvic ligaments
(c) During division and ligation of mackenrodt?s and uterosacral ligaments
(d) At the vaginal angles while incising the vagina to remove the cervix with the
uterus
58.
The net effect of antenatal care has been the following EXCEPT:

(a) Reduction in maternal mortality
(b) Reduction in perinatal mortality
(c) Reduction in the incidence of institutional delivery
(d) Reduction in maternal morbidity
59.
Which one of the following is NOT a component of active phase in the partograph?

(a) Acceleration phase
(b) Phase of maximum slope
(c) Phase of deceleration
(d) Phase of expulsion
60.
From medicolegal point of view which one of the following is NOT a sign of previous child
birth?

(a) Perineum is lax and there is evidence of scarring
(b) Introitus is gaping and there is presence of carunculae myrtiformis
(c) Abdomen is lax and loose with striae and linea alba
(d) Conical cervix with round external os





61.
The components of partograph are all EXCEPT:

(a) Time
(b) Fetal heart rate
(c) Maternal respiratory rate
(d) Maternal urine analysis
62.
Which of the following information are provided by partograph?

1. Colour of liquor
2. Uterine contractions with duration and frequency
3. Dilatation of cervix

Select the correct answer using the code given below:

(a) 1 and 2 only
(b) 2 and 3 only
(c) 1 and 3 only
(d) 1, 2 and 3
63.
Intraoperative recognition of ureter is by which of the following features?

1. Transparent tubular appearance
2. Pale glistening appearance
3. Longitudinal vessels on surface
4. Circumferential vessels on surface

Select the correct answer using the code given below:

(a) 1 and 3
(b) 2 and 4
(c) 2 and 3
(d) 1 and 4
64.
Hysterosalpingography (HSG) is least helpful in detecting which of the following?

(a) Tubal patency
(b) Pelvic adhesions
(c) Asherman syndrome
(d) Congenital uterine anomaly









65.
Which of the following are characteristics of Trichomonas vaginitis?

1. Presence of greenish frothy discharge
2. Vaginal pH > 4.5
3. Presence of clue cells in microscopic examination
4. Strawberry spots on the vaginal mucosa

Select the correct answer using the code given below:

(a) 1, 2 and 3
(b) 1, 2 and 4
(c) 2, 3 and 4
(d) 1, 3 and 4
66.
Tumor marker of epithelial ovarian carcinoma is:

(a) Ca.125
(b) Alpha feto protein
(c) Beta HCG
(d) LDH
67.
The most common site of cervical cancer is:

(a) Endocervix
(b) Ectocervix
(c) Transformation zone
(d) Isthmus
68.
The placenta synthesizes all EXCEPT:

(a) Oestriol
(b) Corticotrophin releasing hormone
(c) PAPP-A(Pregnancy Associated Plasma Protein A)
(d) Dehydroepiandrosterone
69.
Withdrawal bleeding following administration of progesterone in a case of secondary
amenorrhea indicates all EXCEPT:

(a) Absence of pregnancy
(b) Production endogenous estrogen
(c) Endometrium is responsive to estrogen
(d) Defect in pituitary gland
70.
Monilial vaginitis is commonly associated with all EXCEPT:

(a) Prolonged antibiotic therapy
(b) Diabetes Mellitus
(c) Treatment of malaria with chloroquine
(d) Pregnancy
71.
Which one of the following is NOT a risk factor for the development of placenta previa?

(a) Maternal age
(b) Smoking
(c) Previous caesarean section
(d) Maternal anaemia
72.
Common clinical presentations of moderate to severe abruption are all EXCEPT:

(a) Uterine tenderness
(b) Fetal distress
(c) Unexplained pre term labour
(d) Prolonged labour
73.
Common trisomies resulting in spontaneous abortion are all EXCEPT:

(a) Trisomy 21
(b) Trisomy 18
(c) Trisomy 16
(d) Trisomy 1
74.
The initial prevention strategy for antiphospholipid syndrome will be:

1. Steroids
2. Heparin
3. Low dose aspirin
4. Progesterone support

Which of the above is/are correct?

(a) 2 and 3
(b) 3 and 4
(c) 3 only
(d) 1 and 4
75.
Diagnostic criteria for PCOD are:

1. Oligo/amenorrohoea
2. Hyperandrogenism
3. Polycystic ovaries on ultrasound

Which of the above are correct?

(a) 1 and 2 only
(b) 2 and 3 only
(c) 1 and 3 only
(d) 1, 2 and 3


76.
Which of the following symptoms can be associated with pelvic organ prolapse?

1. Difficulty in passing urine
2. Incomplete evacuation of urine
3. Urgency and frequency

Select the correct answer using the code given below:

(a) 1 and 2 only
(b) 2 and 3 only
(c) 1 and 3 only
(d) 1, 2 and 3
77.
A 30 year old lady, P
2
L
2
presents with painful unilateral swelling in vulva for 3 days. Which
of the following statements are true regarding the above case?

1. Bartholin?s abscess may be the likely diagnosis
2. It is to be managed by marsupialisation
3. Gonococcus is the most common pathogenic organism

Select the correct answer using the code given below:
(a) 1 and 3 only
(b) 3 only
(c) 1 and 2 only
(d) 1, 2 and 3
78.
Which one of the following is NOT a sign of separation of placenta?

(a) Uterus becomes globular, firm and ballotable
(b) The fundal height reduces further
(c) Slight bulging in the suprapubic region
(d) Apparent lengthenic of the cord with slight gush of vaginal bleeding
79.
Consider the following regarding examination of a rape victim:

1. Emergency pill is provided
2. Internal examination must be performed
3. HIV testing is done

Which of the above statements is/are correct?

(a) 1 and 3 only
(b) 2 only
(c) 1, 2 and 3
(d) 3 only




FirstRanker.com - FirstRanker's Choice
Combined Medical Services Examination-2020
Paper-II

1.
Indications for fasciotomy in compartment syndrome include all EXCEPT:

(a) Distal sensory disturbance
(b) Compartment pressure > 30 mm Hg
(c) Pain on passive movement of affected muscles
(d) Palpable distal pulses
2.
Which one of the following statements is NOT correct regarding Necrotising Soft Tissue
infections?

(a) Crepitus, skin blistering and focal skin gangrene are typical presenting features
(b) They are monomicrobial in nature
(c) Treatment consists of wide local excision and appropriate antibiotics
(d) Tissue biopsy is required for culture and diagnosis
3.
Which type of surgery is laparoscopic cholecystectomy classified as?

(a) Clean
(b) Clean contaminated
(c) Contaminated
(d) Dirty
4.
A 22-year female has presented with a history of malaise, cough, alternating constipation and
diarrhoea with intermittent abdominal pain for last 6 months. She also complains of
abdominal distension for last 2 days. On examination her abdomen has a doughy feel along
with an ill defined mass over the right lower quadrant. She is most likely suffering from:

(a) Appendicular lump
(b) Ileocaecal tuberculosis
(c) Carcinoma caecum
(d) Ovarian mass
5.
Consider the following statements regarding needle stick injuries:

1. Injured part should be washed under running water
2. Dominant index finger is the commonest site for needle stick injury
3. All needle stick injuries should be reported
4. Hepatitis/HIV testing should be done after needle stick injury

Which of the statements given above are correct?

(a) 1, 2 and 4
(b) 1, 2 and 3
(c) 1, 3 and 4
(d) 2, 3 and 4




6.
Consider the following statements regarding claudication:

1. It is a marker for silent coronary disease
2. Structured exercise program ( 2 hours per week for 3 months) leads to
improvement in symptoms
3. Diabetes mellitus increases the risk and severity of claudication
4. Beta blockers may exacerbate claudication

Which of the above statements are correct?

(a) 1 and 2 only
(b) 1, 3 and 4 only
(c) 2, 3 and 4 only
(d) 1, 2, 3 and 4
7.
A 50-year old lady underwent uneventful bariatric surgery for morbid obesity. On the third
post operative day, she develops breathless and pulmonary embolism is suspected. The next
investigation to confirm the diagnosis will be:

(a) Echocardiography
(b) Duplex venography
(c) CT pulmonary angiography
(d) MR angiography
8.
Medical management of thyrotoxic crisis includes all of the following EXCEPT:

(a) IV fluids
(b) IV propanolol
(c) IV hydrocortisone
(d) IV antibiotics
9.
A patient operated for a parotid gland tumour developed symptoms of sweating and
erytherma (flushing) over the region of surgical excision while eating. The probable
diagnosis is:

(a) Parotid gland fistula
(b) Sialadenitis
(c) Chronic wound infection
(d) Frey?s syndrome
10.
First line hormone therapy for post-menopausal woman with metastatic carcinoma breast is:

(a) Tamoxifen
(b) Ovarian suppression by surgery
(c) Antiprogestins
(d) Anastrazole



11.
All of the following are major subtypes of breast cancer based on Gene array analysis
EXCEPT:

(a) Luminal A and Luminal B
(b) Triple negative
(c) Her-2 receptor positive
(d) Oestrogen receptor positive
12.
All of the following are sequelae of peptic ulcer surgery EXCEPT:

(a) Bilious vomiting
(b) Dumping syndrome
(c) Diarrhoea
(d) Increased appetite
13.
The Child-Turcotte-Pugh (CTP) score for quantifying the severity of chronic liver disease
includes all variables EXCEPT:

(a) Serum bilirubin
(b) Serum albumin
(c) Serum creatinine
(d) INR (International Normalised Ratio)
14.
?Chain of Lakes? appearance due to sacculation with intervening short strictures of pancreatic
duct is seen on:

(a) ERCP
(b) CECT abdomen
(c) Plain X-ray abdomen
(d) Ultrasonography
15.
Which one of the following statements is NOT correct regarding Pyogenic Liver Abscess?

(a) Anorexia, fever, malaise and right upper quadrant abdominal discomfort are the
most common presenting features
(b) It is more common in elderly, diabetics and immunocompromised patients
(c) Treatment is with oral antibiotics alone
(d) Streptococcus milleri and escherichia coli are the most common causative
organisms
16.
?Swiss cheese defects? are seen during laparoscopic repair of:

(a) Ventral hernia
(b) Inguinal hernia
(c) Obturator hernia
(d) Femoral hernia



17.
Which of the following are correct regarding splenic artery aneurysm?

1. Main arterial trunk is the common site
2. Palpable thrill can be felt
3. It is symptomless unless it ruptures

Select the correct answer using the code given below:

(a) 1 and 2 only
(b) 2 and 3 only
(c) 1 and 3 only
(d) 1, 2 and 3
18.
Valentino?s syndrome is:

(a) Pain on per-vaginal examination in pelvic abscess
(b) Pain over left shoulder in left hypochondriac collection
(c) Pain over left groin in perirenal collection
(d) Pain in right iliac fossa in perforated peptic ulcer
19.
Spontaneous bacterial peritonitis occurs due to:

(a) duodenal stump blowout
(b) peptic ulcer perforation
(c) acute bacterial infection of ascites
(d) infection via fallopian tubes
20.
Structure not forming boundaries of the ?Triangle of doom? seen during laparoscopic
inguinal hernia surgery dissection is:

(a) Vas deferens
(b) Inferior epigastric artery
(c) Spermatic cord vessels
(d) Peritoneum
21.
The term mid-line shift is associated with:

(a) Head injury
(b) Chest injury
(c) Abdominal injury
(d) Limb injury
22.
Which one of the following cranial nerves does NOT supply to the external ear?

(a) Cranial nerve V
(b) Cranial nerve VI
(c) Cranial nerve VII
(d) Cranial nerve IX

23.
Left Internal Mammary Artery (LIMA) has become the conduit of choice for Left Anterior
Descending (LAD) artery during coronary artery bypass grafting because:

(a) Long term patency rates are more than 98%
(b) It is close to LAD
(c) Atherosclerosis is never seen in this vessel
(d) It is very easy to harvest
24.
A 50-year old male with significant smoking history presented in the surgical emergency
with sudden severe breathlessness. Chest X-ray shows right sided Pneumothorax. The
appropriate management requires:

(a) Aspiration of air with 16-18 G cannula
(b) Right chest drain of size 8-14 Fr
(c) Oxygen by face mask
(d) Mechanical ventilation
25.
Which one of the following statements regarding Felon is NOT correct?

(a) There is infection of the finger tip between specialised fibrous septa
(b) It is a painless condition
(c) Incision and drainage is the treatment of choice
(d) It is common in diabetics
26.
The most common site for osteosarcoma is:

(a) Proximal femur
(b) Distal femur
(c) Proximal humerus
(d) Distal humerus
27.
Rapid Sequence Induction is indicated in:

(a) Emergency surgery for intestinal obstruction
(b) Elective open hernia surgery
(c) Cardiopulmonary bypass surgery
(d) Elective laparoscopic surgery

28.
Which one of the following statements about Compartment Syndrome is NOT correct?

(a) It is commonest in a closed fracture
(b) Pain is on active movement but not on passive movement of muscles
(c) Fasciotomy is the treatment of choice
(d) Volkmann?s Ischaemic contractive is a late complication




29.
Which one of the following is NOT the strength of ultrasound as a diagnostic modality?

(a) No radiation
(b) Short learning curve
(c) Inexpensive
(d) Allows dynamic studies to be done
30.
Which of the following statements regarding lymphoedema following breast cancer treatment
are correct?

1. Incidence has decreased due to rarely combined therapy of axillary LN dissection and
radiotherapy
2. Precipitating cause like LN metastasis is a major determinant
3. The condition is often painful
4. Oedematous limb is susceptible to bacterial infection

Select the correct answer using the code given below:

(a) 1, 2 and 3
(b) 2, 3 and 4
(c) 1, 3 and 4
(d) 1, 2 and 4
31.
Which one of the following is NOT a risk factor for development of venous thrombosis in
surgical patients?

(a) Age > 60 years
(b) Pregnancy
(c) Obesity (BMI > 30 kg/m
2
)
(d) Diabetes ( HbA1c > 7.5%)
32.
Which one of the following is NOT a complication of massive blood transfusion?

(a) Coagulopathy
(b) Hypocalcaemia
(c) Hyperthermia
(d) Hyperkalemia
33.
Stage III ?Pressure sore? is full thickness skin loss extending:

(a) into subcutaneous tissue but not through fascia
(b) through subcutaneous tissue into fascia
(c) through subcutaneous tissue into fascia and muscles
(d) through subcutaneous tissue into fascia, muscles and bone



34.
During subclavian vein puncture in a surgical ward suddenly a patient developed severe
breathlessness. On auscultation breath sound was absent and the ipsilateral chest was
tympanitic on percussion. The probable diagnosis is:

(a) Iatrogenic pneumothorax
(b) Introgenic hemothorax
(c) Spontaneous pneumothorax
(d) Tension pneumothorax
35.
Kohler?s disease is avascular necrosis of :

(a) Lunate
(b) Capitellum of humerus
(c) First metatarsal head
(d) Navicular
36.
Which one of the following is NOT true of Pyoderma gangrenosum?

(a) It is characterized by cutaneous ulceration with purple undermined edges
(b) It is often secondary to heightened immunological reactivity from another disease
process
(c) Cultures often show Gram positive Staphylococci
(d) Lesions generally respond to steroids
37.
Which of the following is NOT a tissue repair surgery for inguinal hernia repair?

(a) Bassini?s repair
(b) Shouldice repair
(c) Stoppa?s repair
(d) Desarda repair
38.
Which one of the following type of meshes is recommended for intraperitoneal use in
abdominal wall hernia?
(a) Light weight, porous meshes
(b) Heavy weight, porous meshes
(c) Absorbable meshes
(d) Tissue separating meshes
39.
Which one of the following is NOT correct regarding MEN-1 syndrome?

(a) It involves parathyroid glands
(b) It involves pancreas
(c) It involves pituitary gland
(d) It involves pineal gland

40.
Which one of the following is NOT an electronic information site in surgery?

(a) Pubmed
(b) Embase
(c) Cochrane library
(d) National medical library
41.
Which one of the following statements regarding pre-conceptional counseling is NOT
correct?

(a) It is needed only in selected complicated pregnancies
(b) It helps in early detection of risk factors
(c) It helps in reducing maternal morbidity and mortality
(d) It is a part of preventive medicine
42.
Consider the following statements regarding Non Stress Test (NST):

1. Reactive NST indicates a healthy fetus
2. NST is an observed association of fetal breathing with fetal movements
3. NST has a low false negative rate (< 1%) but high false positive rate (>50%)
4. Testing should be started at 20 weeks

Which of the statement(s) given above is/are correct?

(a) 1 and 3
(b) 2 only
(c) 3 only
(d) 1 and 4
43.
Which one of the following is a protective factor for endometrial hyperplasia?

(a) Diabetes
(b) Tamoxifen therapy
(c) Multiparity
(d) Delayed menopause
44.
A woman who is not breast feeding her newborn child is advised to use a contraceptive
method by:

(a) 3
rd
postpartum week
(b) 6
th
postpartum week
(c) 3
rd
postpartum month
(d) 6
th
postpartum month





45.
Pearl index for contraceptive effectiveness is calculated in terms of which of the following?

1. Pregnancy rate
2. Abortion rate
3. Hundred woman years
4. Thousand woman years

Select the correct answer using the code given below:

(a) 1 only
(b) 2 and 3
(c) 1, 2 and 4
(d) 1 and 3
46.
Indications for removal of IUDs are all EXCEPT:

(a) Perforation of uterus
(b) Cyclical menstrual bleeding
(c) Flaring up of salpingitis
(d) Pregnancy with IUD
47.
Contraindications for insertion of IUDs are all EXCEPT:

(a) Suspected pregnancy
(b) Trophoblastic disease
(c) Severe dysmenorrhea
(d) During cesarean section
48.
Which one of the following is NOT a contraindication for use of Mini pill?

(a) Pregnancy
(b) Breast feeding
(c) Thromboembolic disease
(d) History of breast cancer
49.
Which one of the following is the most commonly used surgical method/technique of female
sterilization as recommended by Government of India?

(a) Uchida technique
(b) Irving method
(c) Pomeroy?s method
(d) Madlener technique







50.
Which of the following is/are required for a registered medical practitioner to qualify for
performing Medical Termination of Pregnancy (MTP), as per revised rules of MTP Act?

1. Certified for assisting at least 15 MTP in an authorized centre
2. Diploma or degree in Obstetrics and Gynaecology
3. House surgeon training for 3 months in Obstetrics and Gynaecology
4. Certified training for 6 months in laparoscopic surgeries

Select the correct answer using the code given below:
(a) 1 only
(b) 2 only
(c) 1, 2 and 3
(d) 1, 2 and 4
51.
Which one of the following is NOT a support of uterus, preventing its descent?

(a) Endopelvic fascia
(b) Mackenrodt?s ligament
(c) Inguinal ligament
(d) Pubocervical ligament
52.
As per ICMR guidelines, which one of the following statements is true regarding effects of
COVID-19 on fetus according to current evidence?

(a) There is increased risk of early pregnancy loss
(b) COVID-19 virus is not teratogenic
(c) COVID-19 virus infection is an indication of MTP
(d) There is increased risk of fetal growth restriction
53.
As per ICMR guidelines, which one of the following statements is true regarding COVID-19
infection in pregnancy?

(a) Covid-19 pneumonia in pregnancy is more severe with poor recovery
(b) Pregnant women with heart disease are at higher risk
(c) Vaginal secretions always test positive for COVID-19 in pregnancy
(d) COVID-19 virus is secreted in breast milk
54.
Which one of the following is NOT a method of management of Deep Transverse Arrest with
the living fetus?

(a) Caesarean section
(b) Delivery by ventouse
(c) Delivery by application of forceps to the unrotated head
(d) Manual rotation and application of forceps





55.
Successful version of breech presentation is likely in case all of the following EXCEPT:

(a) Breech with extended legs
(b) Complete breech with sacroanterior position
(c) Non engaged breech
(d) Adequate amniotic fluid
56.
Implantation of a fertilised ovum occurs on which day following fertilisation?

(a) Day 6
(b) Day 10
(c) Day 14
(d) Day 20
57.
During total abdominal hysterectomy the ureter is likely to undergo injury or ligation during
the following steps EXCEPT:

(a) During division and ligation of the round ligaments
(b) During division and ligation of infundibulopelvic ligaments
(c) During division and ligation of mackenrodt?s and uterosacral ligaments
(d) At the vaginal angles while incising the vagina to remove the cervix with the
uterus
58.
The net effect of antenatal care has been the following EXCEPT:

(a) Reduction in maternal mortality
(b) Reduction in perinatal mortality
(c) Reduction in the incidence of institutional delivery
(d) Reduction in maternal morbidity
59.
Which one of the following is NOT a component of active phase in the partograph?

(a) Acceleration phase
(b) Phase of maximum slope
(c) Phase of deceleration
(d) Phase of expulsion
60.
From medicolegal point of view which one of the following is NOT a sign of previous child
birth?

(a) Perineum is lax and there is evidence of scarring
(b) Introitus is gaping and there is presence of carunculae myrtiformis
(c) Abdomen is lax and loose with striae and linea alba
(d) Conical cervix with round external os





61.
The components of partograph are all EXCEPT:

(a) Time
(b) Fetal heart rate
(c) Maternal respiratory rate
(d) Maternal urine analysis
62.
Which of the following information are provided by partograph?

1. Colour of liquor
2. Uterine contractions with duration and frequency
3. Dilatation of cervix

Select the correct answer using the code given below:

(a) 1 and 2 only
(b) 2 and 3 only
(c) 1 and 3 only
(d) 1, 2 and 3
63.
Intraoperative recognition of ureter is by which of the following features?

1. Transparent tubular appearance
2. Pale glistening appearance
3. Longitudinal vessels on surface
4. Circumferential vessels on surface

Select the correct answer using the code given below:

(a) 1 and 3
(b) 2 and 4
(c) 2 and 3
(d) 1 and 4
64.
Hysterosalpingography (HSG) is least helpful in detecting which of the following?

(a) Tubal patency
(b) Pelvic adhesions
(c) Asherman syndrome
(d) Congenital uterine anomaly









65.
Which of the following are characteristics of Trichomonas vaginitis?

1. Presence of greenish frothy discharge
2. Vaginal pH > 4.5
3. Presence of clue cells in microscopic examination
4. Strawberry spots on the vaginal mucosa

Select the correct answer using the code given below:

(a) 1, 2 and 3
(b) 1, 2 and 4
(c) 2, 3 and 4
(d) 1, 3 and 4
66.
Tumor marker of epithelial ovarian carcinoma is:

(a) Ca.125
(b) Alpha feto protein
(c) Beta HCG
(d) LDH
67.
The most common site of cervical cancer is:

(a) Endocervix
(b) Ectocervix
(c) Transformation zone
(d) Isthmus
68.
The placenta synthesizes all EXCEPT:

(a) Oestriol
(b) Corticotrophin releasing hormone
(c) PAPP-A(Pregnancy Associated Plasma Protein A)
(d) Dehydroepiandrosterone
69.
Withdrawal bleeding following administration of progesterone in a case of secondary
amenorrhea indicates all EXCEPT:

(a) Absence of pregnancy
(b) Production endogenous estrogen
(c) Endometrium is responsive to estrogen
(d) Defect in pituitary gland
70.
Monilial vaginitis is commonly associated with all EXCEPT:

(a) Prolonged antibiotic therapy
(b) Diabetes Mellitus
(c) Treatment of malaria with chloroquine
(d) Pregnancy
71.
Which one of the following is NOT a risk factor for the development of placenta previa?

(a) Maternal age
(b) Smoking
(c) Previous caesarean section
(d) Maternal anaemia
72.
Common clinical presentations of moderate to severe abruption are all EXCEPT:

(a) Uterine tenderness
(b) Fetal distress
(c) Unexplained pre term labour
(d) Prolonged labour
73.
Common trisomies resulting in spontaneous abortion are all EXCEPT:

(a) Trisomy 21
(b) Trisomy 18
(c) Trisomy 16
(d) Trisomy 1
74.
The initial prevention strategy for antiphospholipid syndrome will be:

1. Steroids
2. Heparin
3. Low dose aspirin
4. Progesterone support

Which of the above is/are correct?

(a) 2 and 3
(b) 3 and 4
(c) 3 only
(d) 1 and 4
75.
Diagnostic criteria for PCOD are:

1. Oligo/amenorrohoea
2. Hyperandrogenism
3. Polycystic ovaries on ultrasound

Which of the above are correct?

(a) 1 and 2 only
(b) 2 and 3 only
(c) 1 and 3 only
(d) 1, 2 and 3


76.
Which of the following symptoms can be associated with pelvic organ prolapse?

1. Difficulty in passing urine
2. Incomplete evacuation of urine
3. Urgency and frequency

Select the correct answer using the code given below:

(a) 1 and 2 only
(b) 2 and 3 only
(c) 1 and 3 only
(d) 1, 2 and 3
77.
A 30 year old lady, P
2
L
2
presents with painful unilateral swelling in vulva for 3 days. Which
of the following statements are true regarding the above case?

1. Bartholin?s abscess may be the likely diagnosis
2. It is to be managed by marsupialisation
3. Gonococcus is the most common pathogenic organism

Select the correct answer using the code given below:
(a) 1 and 3 only
(b) 3 only
(c) 1 and 2 only
(d) 1, 2 and 3
78.
Which one of the following is NOT a sign of separation of placenta?

(a) Uterus becomes globular, firm and ballotable
(b) The fundal height reduces further
(c) Slight bulging in the suprapubic region
(d) Apparent lengthenic of the cord with slight gush of vaginal bleeding
79.
Consider the following regarding examination of a rape victim:

1. Emergency pill is provided
2. Internal examination must be performed
3. HIV testing is done

Which of the above statements is/are correct?

(a) 1 and 3 only
(b) 2 only
(c) 1, 2 and 3
(d) 3 only




80.
Consider the following cardinal movements of mechanism of normal labor:
1. Engagement
2. Internal rotation
3. Flexion
4. Restitution
5. Crowning
6. External rotation

What is the correct sequence of movements in labor in occipito-lateral position?

(a) 1, 2, 3, 4, 5 and 6
(b) 1, 3, 2, 5, 4 and 6
(c) 2, 1, 3, 4, 5 and 6
(d) 3, 1, 2, 4, 6 and 5
81.
Which of the following represent the properties of an ideal disinfectant?

1. It is broad spectrum
2. It is fast acting
3. It is non-toxic

Select the correct answer using the code given below:
(a) 1 and 2 only
(b) 2 and 3 only
(c) 1 and 3 only
(d) 1, 2 and 3
82.
Which of the following measures can help reduce the risk of systemic hypertension?

1. Reduction in dietary intake of common salt
2. Controlling weight for age
3. Increasing potassium rich foods in the diet

Select the correct answer using the code given below:

(a) 1 and 2 only
(b) 2 and 3 only
(c) 1 and 3 only
(d) 1, 2 and 3








FirstRanker.com - FirstRanker's Choice
Combined Medical Services Examination-2020
Paper-II

1.
Indications for fasciotomy in compartment syndrome include all EXCEPT:

(a) Distal sensory disturbance
(b) Compartment pressure > 30 mm Hg
(c) Pain on passive movement of affected muscles
(d) Palpable distal pulses
2.
Which one of the following statements is NOT correct regarding Necrotising Soft Tissue
infections?

(a) Crepitus, skin blistering and focal skin gangrene are typical presenting features
(b) They are monomicrobial in nature
(c) Treatment consists of wide local excision and appropriate antibiotics
(d) Tissue biopsy is required for culture and diagnosis
3.
Which type of surgery is laparoscopic cholecystectomy classified as?

(a) Clean
(b) Clean contaminated
(c) Contaminated
(d) Dirty
4.
A 22-year female has presented with a history of malaise, cough, alternating constipation and
diarrhoea with intermittent abdominal pain for last 6 months. She also complains of
abdominal distension for last 2 days. On examination her abdomen has a doughy feel along
with an ill defined mass over the right lower quadrant. She is most likely suffering from:

(a) Appendicular lump
(b) Ileocaecal tuberculosis
(c) Carcinoma caecum
(d) Ovarian mass
5.
Consider the following statements regarding needle stick injuries:

1. Injured part should be washed under running water
2. Dominant index finger is the commonest site for needle stick injury
3. All needle stick injuries should be reported
4. Hepatitis/HIV testing should be done after needle stick injury

Which of the statements given above are correct?

(a) 1, 2 and 4
(b) 1, 2 and 3
(c) 1, 3 and 4
(d) 2, 3 and 4




6.
Consider the following statements regarding claudication:

1. It is a marker for silent coronary disease
2. Structured exercise program ( 2 hours per week for 3 months) leads to
improvement in symptoms
3. Diabetes mellitus increases the risk and severity of claudication
4. Beta blockers may exacerbate claudication

Which of the above statements are correct?

(a) 1 and 2 only
(b) 1, 3 and 4 only
(c) 2, 3 and 4 only
(d) 1, 2, 3 and 4
7.
A 50-year old lady underwent uneventful bariatric surgery for morbid obesity. On the third
post operative day, she develops breathless and pulmonary embolism is suspected. The next
investigation to confirm the diagnosis will be:

(a) Echocardiography
(b) Duplex venography
(c) CT pulmonary angiography
(d) MR angiography
8.
Medical management of thyrotoxic crisis includes all of the following EXCEPT:

(a) IV fluids
(b) IV propanolol
(c) IV hydrocortisone
(d) IV antibiotics
9.
A patient operated for a parotid gland tumour developed symptoms of sweating and
erytherma (flushing) over the region of surgical excision while eating. The probable
diagnosis is:

(a) Parotid gland fistula
(b) Sialadenitis
(c) Chronic wound infection
(d) Frey?s syndrome
10.
First line hormone therapy for post-menopausal woman with metastatic carcinoma breast is:

(a) Tamoxifen
(b) Ovarian suppression by surgery
(c) Antiprogestins
(d) Anastrazole



11.
All of the following are major subtypes of breast cancer based on Gene array analysis
EXCEPT:

(a) Luminal A and Luminal B
(b) Triple negative
(c) Her-2 receptor positive
(d) Oestrogen receptor positive
12.
All of the following are sequelae of peptic ulcer surgery EXCEPT:

(a) Bilious vomiting
(b) Dumping syndrome
(c) Diarrhoea
(d) Increased appetite
13.
The Child-Turcotte-Pugh (CTP) score for quantifying the severity of chronic liver disease
includes all variables EXCEPT:

(a) Serum bilirubin
(b) Serum albumin
(c) Serum creatinine
(d) INR (International Normalised Ratio)
14.
?Chain of Lakes? appearance due to sacculation with intervening short strictures of pancreatic
duct is seen on:

(a) ERCP
(b) CECT abdomen
(c) Plain X-ray abdomen
(d) Ultrasonography
15.
Which one of the following statements is NOT correct regarding Pyogenic Liver Abscess?

(a) Anorexia, fever, malaise and right upper quadrant abdominal discomfort are the
most common presenting features
(b) It is more common in elderly, diabetics and immunocompromised patients
(c) Treatment is with oral antibiotics alone
(d) Streptococcus milleri and escherichia coli are the most common causative
organisms
16.
?Swiss cheese defects? are seen during laparoscopic repair of:

(a) Ventral hernia
(b) Inguinal hernia
(c) Obturator hernia
(d) Femoral hernia



17.
Which of the following are correct regarding splenic artery aneurysm?

1. Main arterial trunk is the common site
2. Palpable thrill can be felt
3. It is symptomless unless it ruptures

Select the correct answer using the code given below:

(a) 1 and 2 only
(b) 2 and 3 only
(c) 1 and 3 only
(d) 1, 2 and 3
18.
Valentino?s syndrome is:

(a) Pain on per-vaginal examination in pelvic abscess
(b) Pain over left shoulder in left hypochondriac collection
(c) Pain over left groin in perirenal collection
(d) Pain in right iliac fossa in perforated peptic ulcer
19.
Spontaneous bacterial peritonitis occurs due to:

(a) duodenal stump blowout
(b) peptic ulcer perforation
(c) acute bacterial infection of ascites
(d) infection via fallopian tubes
20.
Structure not forming boundaries of the ?Triangle of doom? seen during laparoscopic
inguinal hernia surgery dissection is:

(a) Vas deferens
(b) Inferior epigastric artery
(c) Spermatic cord vessels
(d) Peritoneum
21.
The term mid-line shift is associated with:

(a) Head injury
(b) Chest injury
(c) Abdominal injury
(d) Limb injury
22.
Which one of the following cranial nerves does NOT supply to the external ear?

(a) Cranial nerve V
(b) Cranial nerve VI
(c) Cranial nerve VII
(d) Cranial nerve IX

23.
Left Internal Mammary Artery (LIMA) has become the conduit of choice for Left Anterior
Descending (LAD) artery during coronary artery bypass grafting because:

(a) Long term patency rates are more than 98%
(b) It is close to LAD
(c) Atherosclerosis is never seen in this vessel
(d) It is very easy to harvest
24.
A 50-year old male with significant smoking history presented in the surgical emergency
with sudden severe breathlessness. Chest X-ray shows right sided Pneumothorax. The
appropriate management requires:

(a) Aspiration of air with 16-18 G cannula
(b) Right chest drain of size 8-14 Fr
(c) Oxygen by face mask
(d) Mechanical ventilation
25.
Which one of the following statements regarding Felon is NOT correct?

(a) There is infection of the finger tip between specialised fibrous septa
(b) It is a painless condition
(c) Incision and drainage is the treatment of choice
(d) It is common in diabetics
26.
The most common site for osteosarcoma is:

(a) Proximal femur
(b) Distal femur
(c) Proximal humerus
(d) Distal humerus
27.
Rapid Sequence Induction is indicated in:

(a) Emergency surgery for intestinal obstruction
(b) Elective open hernia surgery
(c) Cardiopulmonary bypass surgery
(d) Elective laparoscopic surgery

28.
Which one of the following statements about Compartment Syndrome is NOT correct?

(a) It is commonest in a closed fracture
(b) Pain is on active movement but not on passive movement of muscles
(c) Fasciotomy is the treatment of choice
(d) Volkmann?s Ischaemic contractive is a late complication




29.
Which one of the following is NOT the strength of ultrasound as a diagnostic modality?

(a) No radiation
(b) Short learning curve
(c) Inexpensive
(d) Allows dynamic studies to be done
30.
Which of the following statements regarding lymphoedema following breast cancer treatment
are correct?

1. Incidence has decreased due to rarely combined therapy of axillary LN dissection and
radiotherapy
2. Precipitating cause like LN metastasis is a major determinant
3. The condition is often painful
4. Oedematous limb is susceptible to bacterial infection

Select the correct answer using the code given below:

(a) 1, 2 and 3
(b) 2, 3 and 4
(c) 1, 3 and 4
(d) 1, 2 and 4
31.
Which one of the following is NOT a risk factor for development of venous thrombosis in
surgical patients?

(a) Age > 60 years
(b) Pregnancy
(c) Obesity (BMI > 30 kg/m
2
)
(d) Diabetes ( HbA1c > 7.5%)
32.
Which one of the following is NOT a complication of massive blood transfusion?

(a) Coagulopathy
(b) Hypocalcaemia
(c) Hyperthermia
(d) Hyperkalemia
33.
Stage III ?Pressure sore? is full thickness skin loss extending:

(a) into subcutaneous tissue but not through fascia
(b) through subcutaneous tissue into fascia
(c) through subcutaneous tissue into fascia and muscles
(d) through subcutaneous tissue into fascia, muscles and bone



34.
During subclavian vein puncture in a surgical ward suddenly a patient developed severe
breathlessness. On auscultation breath sound was absent and the ipsilateral chest was
tympanitic on percussion. The probable diagnosis is:

(a) Iatrogenic pneumothorax
(b) Introgenic hemothorax
(c) Spontaneous pneumothorax
(d) Tension pneumothorax
35.
Kohler?s disease is avascular necrosis of :

(a) Lunate
(b) Capitellum of humerus
(c) First metatarsal head
(d) Navicular
36.
Which one of the following is NOT true of Pyoderma gangrenosum?

(a) It is characterized by cutaneous ulceration with purple undermined edges
(b) It is often secondary to heightened immunological reactivity from another disease
process
(c) Cultures often show Gram positive Staphylococci
(d) Lesions generally respond to steroids
37.
Which of the following is NOT a tissue repair surgery for inguinal hernia repair?

(a) Bassini?s repair
(b) Shouldice repair
(c) Stoppa?s repair
(d) Desarda repair
38.
Which one of the following type of meshes is recommended for intraperitoneal use in
abdominal wall hernia?
(a) Light weight, porous meshes
(b) Heavy weight, porous meshes
(c) Absorbable meshes
(d) Tissue separating meshes
39.
Which one of the following is NOT correct regarding MEN-1 syndrome?

(a) It involves parathyroid glands
(b) It involves pancreas
(c) It involves pituitary gland
(d) It involves pineal gland

40.
Which one of the following is NOT an electronic information site in surgery?

(a) Pubmed
(b) Embase
(c) Cochrane library
(d) National medical library
41.
Which one of the following statements regarding pre-conceptional counseling is NOT
correct?

(a) It is needed only in selected complicated pregnancies
(b) It helps in early detection of risk factors
(c) It helps in reducing maternal morbidity and mortality
(d) It is a part of preventive medicine
42.
Consider the following statements regarding Non Stress Test (NST):

1. Reactive NST indicates a healthy fetus
2. NST is an observed association of fetal breathing with fetal movements
3. NST has a low false negative rate (< 1%) but high false positive rate (>50%)
4. Testing should be started at 20 weeks

Which of the statement(s) given above is/are correct?

(a) 1 and 3
(b) 2 only
(c) 3 only
(d) 1 and 4
43.
Which one of the following is a protective factor for endometrial hyperplasia?

(a) Diabetes
(b) Tamoxifen therapy
(c) Multiparity
(d) Delayed menopause
44.
A woman who is not breast feeding her newborn child is advised to use a contraceptive
method by:

(a) 3
rd
postpartum week
(b) 6
th
postpartum week
(c) 3
rd
postpartum month
(d) 6
th
postpartum month





45.
Pearl index for contraceptive effectiveness is calculated in terms of which of the following?

1. Pregnancy rate
2. Abortion rate
3. Hundred woman years
4. Thousand woman years

Select the correct answer using the code given below:

(a) 1 only
(b) 2 and 3
(c) 1, 2 and 4
(d) 1 and 3
46.
Indications for removal of IUDs are all EXCEPT:

(a) Perforation of uterus
(b) Cyclical menstrual bleeding
(c) Flaring up of salpingitis
(d) Pregnancy with IUD
47.
Contraindications for insertion of IUDs are all EXCEPT:

(a) Suspected pregnancy
(b) Trophoblastic disease
(c) Severe dysmenorrhea
(d) During cesarean section
48.
Which one of the following is NOT a contraindication for use of Mini pill?

(a) Pregnancy
(b) Breast feeding
(c) Thromboembolic disease
(d) History of breast cancer
49.
Which one of the following is the most commonly used surgical method/technique of female
sterilization as recommended by Government of India?

(a) Uchida technique
(b) Irving method
(c) Pomeroy?s method
(d) Madlener technique







50.
Which of the following is/are required for a registered medical practitioner to qualify for
performing Medical Termination of Pregnancy (MTP), as per revised rules of MTP Act?

1. Certified for assisting at least 15 MTP in an authorized centre
2. Diploma or degree in Obstetrics and Gynaecology
3. House surgeon training for 3 months in Obstetrics and Gynaecology
4. Certified training for 6 months in laparoscopic surgeries

Select the correct answer using the code given below:
(a) 1 only
(b) 2 only
(c) 1, 2 and 3
(d) 1, 2 and 4
51.
Which one of the following is NOT a support of uterus, preventing its descent?

(a) Endopelvic fascia
(b) Mackenrodt?s ligament
(c) Inguinal ligament
(d) Pubocervical ligament
52.
As per ICMR guidelines, which one of the following statements is true regarding effects of
COVID-19 on fetus according to current evidence?

(a) There is increased risk of early pregnancy loss
(b) COVID-19 virus is not teratogenic
(c) COVID-19 virus infection is an indication of MTP
(d) There is increased risk of fetal growth restriction
53.
As per ICMR guidelines, which one of the following statements is true regarding COVID-19
infection in pregnancy?

(a) Covid-19 pneumonia in pregnancy is more severe with poor recovery
(b) Pregnant women with heart disease are at higher risk
(c) Vaginal secretions always test positive for COVID-19 in pregnancy
(d) COVID-19 virus is secreted in breast milk
54.
Which one of the following is NOT a method of management of Deep Transverse Arrest with
the living fetus?

(a) Caesarean section
(b) Delivery by ventouse
(c) Delivery by application of forceps to the unrotated head
(d) Manual rotation and application of forceps





55.
Successful version of breech presentation is likely in case all of the following EXCEPT:

(a) Breech with extended legs
(b) Complete breech with sacroanterior position
(c) Non engaged breech
(d) Adequate amniotic fluid
56.
Implantation of a fertilised ovum occurs on which day following fertilisation?

(a) Day 6
(b) Day 10
(c) Day 14
(d) Day 20
57.
During total abdominal hysterectomy the ureter is likely to undergo injury or ligation during
the following steps EXCEPT:

(a) During division and ligation of the round ligaments
(b) During division and ligation of infundibulopelvic ligaments
(c) During division and ligation of mackenrodt?s and uterosacral ligaments
(d) At the vaginal angles while incising the vagina to remove the cervix with the
uterus
58.
The net effect of antenatal care has been the following EXCEPT:

(a) Reduction in maternal mortality
(b) Reduction in perinatal mortality
(c) Reduction in the incidence of institutional delivery
(d) Reduction in maternal morbidity
59.
Which one of the following is NOT a component of active phase in the partograph?

(a) Acceleration phase
(b) Phase of maximum slope
(c) Phase of deceleration
(d) Phase of expulsion
60.
From medicolegal point of view which one of the following is NOT a sign of previous child
birth?

(a) Perineum is lax and there is evidence of scarring
(b) Introitus is gaping and there is presence of carunculae myrtiformis
(c) Abdomen is lax and loose with striae and linea alba
(d) Conical cervix with round external os





61.
The components of partograph are all EXCEPT:

(a) Time
(b) Fetal heart rate
(c) Maternal respiratory rate
(d) Maternal urine analysis
62.
Which of the following information are provided by partograph?

1. Colour of liquor
2. Uterine contractions with duration and frequency
3. Dilatation of cervix

Select the correct answer using the code given below:

(a) 1 and 2 only
(b) 2 and 3 only
(c) 1 and 3 only
(d) 1, 2 and 3
63.
Intraoperative recognition of ureter is by which of the following features?

1. Transparent tubular appearance
2. Pale glistening appearance
3. Longitudinal vessels on surface
4. Circumferential vessels on surface

Select the correct answer using the code given below:

(a) 1 and 3
(b) 2 and 4
(c) 2 and 3
(d) 1 and 4
64.
Hysterosalpingography (HSG) is least helpful in detecting which of the following?

(a) Tubal patency
(b) Pelvic adhesions
(c) Asherman syndrome
(d) Congenital uterine anomaly









65.
Which of the following are characteristics of Trichomonas vaginitis?

1. Presence of greenish frothy discharge
2. Vaginal pH > 4.5
3. Presence of clue cells in microscopic examination
4. Strawberry spots on the vaginal mucosa

Select the correct answer using the code given below:

(a) 1, 2 and 3
(b) 1, 2 and 4
(c) 2, 3 and 4
(d) 1, 3 and 4
66.
Tumor marker of epithelial ovarian carcinoma is:

(a) Ca.125
(b) Alpha feto protein
(c) Beta HCG
(d) LDH
67.
The most common site of cervical cancer is:

(a) Endocervix
(b) Ectocervix
(c) Transformation zone
(d) Isthmus
68.
The placenta synthesizes all EXCEPT:

(a) Oestriol
(b) Corticotrophin releasing hormone
(c) PAPP-A(Pregnancy Associated Plasma Protein A)
(d) Dehydroepiandrosterone
69.
Withdrawal bleeding following administration of progesterone in a case of secondary
amenorrhea indicates all EXCEPT:

(a) Absence of pregnancy
(b) Production endogenous estrogen
(c) Endometrium is responsive to estrogen
(d) Defect in pituitary gland
70.
Monilial vaginitis is commonly associated with all EXCEPT:

(a) Prolonged antibiotic therapy
(b) Diabetes Mellitus
(c) Treatment of malaria with chloroquine
(d) Pregnancy
71.
Which one of the following is NOT a risk factor for the development of placenta previa?

(a) Maternal age
(b) Smoking
(c) Previous caesarean section
(d) Maternal anaemia
72.
Common clinical presentations of moderate to severe abruption are all EXCEPT:

(a) Uterine tenderness
(b) Fetal distress
(c) Unexplained pre term labour
(d) Prolonged labour
73.
Common trisomies resulting in spontaneous abortion are all EXCEPT:

(a) Trisomy 21
(b) Trisomy 18
(c) Trisomy 16
(d) Trisomy 1
74.
The initial prevention strategy for antiphospholipid syndrome will be:

1. Steroids
2. Heparin
3. Low dose aspirin
4. Progesterone support

Which of the above is/are correct?

(a) 2 and 3
(b) 3 and 4
(c) 3 only
(d) 1 and 4
75.
Diagnostic criteria for PCOD are:

1. Oligo/amenorrohoea
2. Hyperandrogenism
3. Polycystic ovaries on ultrasound

Which of the above are correct?

(a) 1 and 2 only
(b) 2 and 3 only
(c) 1 and 3 only
(d) 1, 2 and 3


76.
Which of the following symptoms can be associated with pelvic organ prolapse?

1. Difficulty in passing urine
2. Incomplete evacuation of urine
3. Urgency and frequency

Select the correct answer using the code given below:

(a) 1 and 2 only
(b) 2 and 3 only
(c) 1 and 3 only
(d) 1, 2 and 3
77.
A 30 year old lady, P
2
L
2
presents with painful unilateral swelling in vulva for 3 days. Which
of the following statements are true regarding the above case?

1. Bartholin?s abscess may be the likely diagnosis
2. It is to be managed by marsupialisation
3. Gonococcus is the most common pathogenic organism

Select the correct answer using the code given below:
(a) 1 and 3 only
(b) 3 only
(c) 1 and 2 only
(d) 1, 2 and 3
78.
Which one of the following is NOT a sign of separation of placenta?

(a) Uterus becomes globular, firm and ballotable
(b) The fundal height reduces further
(c) Slight bulging in the suprapubic region
(d) Apparent lengthenic of the cord with slight gush of vaginal bleeding
79.
Consider the following regarding examination of a rape victim:

1. Emergency pill is provided
2. Internal examination must be performed
3. HIV testing is done

Which of the above statements is/are correct?

(a) 1 and 3 only
(b) 2 only
(c) 1, 2 and 3
(d) 3 only




80.
Consider the following cardinal movements of mechanism of normal labor:
1. Engagement
2. Internal rotation
3. Flexion
4. Restitution
5. Crowning
6. External rotation

What is the correct sequence of movements in labor in occipito-lateral position?

(a) 1, 2, 3, 4, 5 and 6
(b) 1, 3, 2, 5, 4 and 6
(c) 2, 1, 3, 4, 5 and 6
(d) 3, 1, 2, 4, 6 and 5
81.
Which of the following represent the properties of an ideal disinfectant?

1. It is broad spectrum
2. It is fast acting
3. It is non-toxic

Select the correct answer using the code given below:
(a) 1 and 2 only
(b) 2 and 3 only
(c) 1 and 3 only
(d) 1, 2 and 3
82.
Which of the following measures can help reduce the risk of systemic hypertension?

1. Reduction in dietary intake of common salt
2. Controlling weight for age
3. Increasing potassium rich foods in the diet

Select the correct answer using the code given below:

(a) 1 and 2 only
(b) 2 and 3 only
(c) 1 and 3 only
(d) 1, 2 and 3








83.
Keeping biological determinants in perspective, consider the following statements:

1. Presence of a normal karyotype is the first requisite for human health
2. Genetic screening can play an important role in prevention of wide spectrum of
diseases
3. If an individual is allowed to live in healthy relationship with the environment, the
person?s genetic potentialities can transform into phenotypic realities

Which of the above statements is/are correct?

(a) 1 and 2 only
(b) 1 and 3 only
(c) 1, 2 and 3
(d) 2 and 3 only
84.
NITI Aayog has the following roles EXCEPT:

(a) It provides critical directional and strategic input in the development process
(b) It provides relevant technical advice with focus on technology upgradation
(c) It focuses on capacity building
(d) It focuses on the development of ?Referral Service Complex?
85.
Poor hand hygiene of a mess worker in a university college mess led to Hepatitis A cases in
the hostel inmates. What type of epidemic will this exposure present with?

1. Propagated
2. Common source-continuous exposure
3. Common source-point exposure

Select the correct answer using the code given below:

(a) 1 and 2
(b) 1 and 3
(c) 1 only
(d) 2 only












FirstRanker.com - FirstRanker's Choice
Combined Medical Services Examination-2020
Paper-II

1.
Indications for fasciotomy in compartment syndrome include all EXCEPT:

(a) Distal sensory disturbance
(b) Compartment pressure > 30 mm Hg
(c) Pain on passive movement of affected muscles
(d) Palpable distal pulses
2.
Which one of the following statements is NOT correct regarding Necrotising Soft Tissue
infections?

(a) Crepitus, skin blistering and focal skin gangrene are typical presenting features
(b) They are monomicrobial in nature
(c) Treatment consists of wide local excision and appropriate antibiotics
(d) Tissue biopsy is required for culture and diagnosis
3.
Which type of surgery is laparoscopic cholecystectomy classified as?

(a) Clean
(b) Clean contaminated
(c) Contaminated
(d) Dirty
4.
A 22-year female has presented with a history of malaise, cough, alternating constipation and
diarrhoea with intermittent abdominal pain for last 6 months. She also complains of
abdominal distension for last 2 days. On examination her abdomen has a doughy feel along
with an ill defined mass over the right lower quadrant. She is most likely suffering from:

(a) Appendicular lump
(b) Ileocaecal tuberculosis
(c) Carcinoma caecum
(d) Ovarian mass
5.
Consider the following statements regarding needle stick injuries:

1. Injured part should be washed under running water
2. Dominant index finger is the commonest site for needle stick injury
3. All needle stick injuries should be reported
4. Hepatitis/HIV testing should be done after needle stick injury

Which of the statements given above are correct?

(a) 1, 2 and 4
(b) 1, 2 and 3
(c) 1, 3 and 4
(d) 2, 3 and 4




6.
Consider the following statements regarding claudication:

1. It is a marker for silent coronary disease
2. Structured exercise program ( 2 hours per week for 3 months) leads to
improvement in symptoms
3. Diabetes mellitus increases the risk and severity of claudication
4. Beta blockers may exacerbate claudication

Which of the above statements are correct?

(a) 1 and 2 only
(b) 1, 3 and 4 only
(c) 2, 3 and 4 only
(d) 1, 2, 3 and 4
7.
A 50-year old lady underwent uneventful bariatric surgery for morbid obesity. On the third
post operative day, she develops breathless and pulmonary embolism is suspected. The next
investigation to confirm the diagnosis will be:

(a) Echocardiography
(b) Duplex venography
(c) CT pulmonary angiography
(d) MR angiography
8.
Medical management of thyrotoxic crisis includes all of the following EXCEPT:

(a) IV fluids
(b) IV propanolol
(c) IV hydrocortisone
(d) IV antibiotics
9.
A patient operated for a parotid gland tumour developed symptoms of sweating and
erytherma (flushing) over the region of surgical excision while eating. The probable
diagnosis is:

(a) Parotid gland fistula
(b) Sialadenitis
(c) Chronic wound infection
(d) Frey?s syndrome
10.
First line hormone therapy for post-menopausal woman with metastatic carcinoma breast is:

(a) Tamoxifen
(b) Ovarian suppression by surgery
(c) Antiprogestins
(d) Anastrazole



11.
All of the following are major subtypes of breast cancer based on Gene array analysis
EXCEPT:

(a) Luminal A and Luminal B
(b) Triple negative
(c) Her-2 receptor positive
(d) Oestrogen receptor positive
12.
All of the following are sequelae of peptic ulcer surgery EXCEPT:

(a) Bilious vomiting
(b) Dumping syndrome
(c) Diarrhoea
(d) Increased appetite
13.
The Child-Turcotte-Pugh (CTP) score for quantifying the severity of chronic liver disease
includes all variables EXCEPT:

(a) Serum bilirubin
(b) Serum albumin
(c) Serum creatinine
(d) INR (International Normalised Ratio)
14.
?Chain of Lakes? appearance due to sacculation with intervening short strictures of pancreatic
duct is seen on:

(a) ERCP
(b) CECT abdomen
(c) Plain X-ray abdomen
(d) Ultrasonography
15.
Which one of the following statements is NOT correct regarding Pyogenic Liver Abscess?

(a) Anorexia, fever, malaise and right upper quadrant abdominal discomfort are the
most common presenting features
(b) It is more common in elderly, diabetics and immunocompromised patients
(c) Treatment is with oral antibiotics alone
(d) Streptococcus milleri and escherichia coli are the most common causative
organisms
16.
?Swiss cheese defects? are seen during laparoscopic repair of:

(a) Ventral hernia
(b) Inguinal hernia
(c) Obturator hernia
(d) Femoral hernia



17.
Which of the following are correct regarding splenic artery aneurysm?

1. Main arterial trunk is the common site
2. Palpable thrill can be felt
3. It is symptomless unless it ruptures

Select the correct answer using the code given below:

(a) 1 and 2 only
(b) 2 and 3 only
(c) 1 and 3 only
(d) 1, 2 and 3
18.
Valentino?s syndrome is:

(a) Pain on per-vaginal examination in pelvic abscess
(b) Pain over left shoulder in left hypochondriac collection
(c) Pain over left groin in perirenal collection
(d) Pain in right iliac fossa in perforated peptic ulcer
19.
Spontaneous bacterial peritonitis occurs due to:

(a) duodenal stump blowout
(b) peptic ulcer perforation
(c) acute bacterial infection of ascites
(d) infection via fallopian tubes
20.
Structure not forming boundaries of the ?Triangle of doom? seen during laparoscopic
inguinal hernia surgery dissection is:

(a) Vas deferens
(b) Inferior epigastric artery
(c) Spermatic cord vessels
(d) Peritoneum
21.
The term mid-line shift is associated with:

(a) Head injury
(b) Chest injury
(c) Abdominal injury
(d) Limb injury
22.
Which one of the following cranial nerves does NOT supply to the external ear?

(a) Cranial nerve V
(b) Cranial nerve VI
(c) Cranial nerve VII
(d) Cranial nerve IX

23.
Left Internal Mammary Artery (LIMA) has become the conduit of choice for Left Anterior
Descending (LAD) artery during coronary artery bypass grafting because:

(a) Long term patency rates are more than 98%
(b) It is close to LAD
(c) Atherosclerosis is never seen in this vessel
(d) It is very easy to harvest
24.
A 50-year old male with significant smoking history presented in the surgical emergency
with sudden severe breathlessness. Chest X-ray shows right sided Pneumothorax. The
appropriate management requires:

(a) Aspiration of air with 16-18 G cannula
(b) Right chest drain of size 8-14 Fr
(c) Oxygen by face mask
(d) Mechanical ventilation
25.
Which one of the following statements regarding Felon is NOT correct?

(a) There is infection of the finger tip between specialised fibrous septa
(b) It is a painless condition
(c) Incision and drainage is the treatment of choice
(d) It is common in diabetics
26.
The most common site for osteosarcoma is:

(a) Proximal femur
(b) Distal femur
(c) Proximal humerus
(d) Distal humerus
27.
Rapid Sequence Induction is indicated in:

(a) Emergency surgery for intestinal obstruction
(b) Elective open hernia surgery
(c) Cardiopulmonary bypass surgery
(d) Elective laparoscopic surgery

28.
Which one of the following statements about Compartment Syndrome is NOT correct?

(a) It is commonest in a closed fracture
(b) Pain is on active movement but not on passive movement of muscles
(c) Fasciotomy is the treatment of choice
(d) Volkmann?s Ischaemic contractive is a late complication




29.
Which one of the following is NOT the strength of ultrasound as a diagnostic modality?

(a) No radiation
(b) Short learning curve
(c) Inexpensive
(d) Allows dynamic studies to be done
30.
Which of the following statements regarding lymphoedema following breast cancer treatment
are correct?

1. Incidence has decreased due to rarely combined therapy of axillary LN dissection and
radiotherapy
2. Precipitating cause like LN metastasis is a major determinant
3. The condition is often painful
4. Oedematous limb is susceptible to bacterial infection

Select the correct answer using the code given below:

(a) 1, 2 and 3
(b) 2, 3 and 4
(c) 1, 3 and 4
(d) 1, 2 and 4
31.
Which one of the following is NOT a risk factor for development of venous thrombosis in
surgical patients?

(a) Age > 60 years
(b) Pregnancy
(c) Obesity (BMI > 30 kg/m
2
)
(d) Diabetes ( HbA1c > 7.5%)
32.
Which one of the following is NOT a complication of massive blood transfusion?

(a) Coagulopathy
(b) Hypocalcaemia
(c) Hyperthermia
(d) Hyperkalemia
33.
Stage III ?Pressure sore? is full thickness skin loss extending:

(a) into subcutaneous tissue but not through fascia
(b) through subcutaneous tissue into fascia
(c) through subcutaneous tissue into fascia and muscles
(d) through subcutaneous tissue into fascia, muscles and bone



34.
During subclavian vein puncture in a surgical ward suddenly a patient developed severe
breathlessness. On auscultation breath sound was absent and the ipsilateral chest was
tympanitic on percussion. The probable diagnosis is:

(a) Iatrogenic pneumothorax
(b) Introgenic hemothorax
(c) Spontaneous pneumothorax
(d) Tension pneumothorax
35.
Kohler?s disease is avascular necrosis of :

(a) Lunate
(b) Capitellum of humerus
(c) First metatarsal head
(d) Navicular
36.
Which one of the following is NOT true of Pyoderma gangrenosum?

(a) It is characterized by cutaneous ulceration with purple undermined edges
(b) It is often secondary to heightened immunological reactivity from another disease
process
(c) Cultures often show Gram positive Staphylococci
(d) Lesions generally respond to steroids
37.
Which of the following is NOT a tissue repair surgery for inguinal hernia repair?

(a) Bassini?s repair
(b) Shouldice repair
(c) Stoppa?s repair
(d) Desarda repair
38.
Which one of the following type of meshes is recommended for intraperitoneal use in
abdominal wall hernia?
(a) Light weight, porous meshes
(b) Heavy weight, porous meshes
(c) Absorbable meshes
(d) Tissue separating meshes
39.
Which one of the following is NOT correct regarding MEN-1 syndrome?

(a) It involves parathyroid glands
(b) It involves pancreas
(c) It involves pituitary gland
(d) It involves pineal gland

40.
Which one of the following is NOT an electronic information site in surgery?

(a) Pubmed
(b) Embase
(c) Cochrane library
(d) National medical library
41.
Which one of the following statements regarding pre-conceptional counseling is NOT
correct?

(a) It is needed only in selected complicated pregnancies
(b) It helps in early detection of risk factors
(c) It helps in reducing maternal morbidity and mortality
(d) It is a part of preventive medicine
42.
Consider the following statements regarding Non Stress Test (NST):

1. Reactive NST indicates a healthy fetus
2. NST is an observed association of fetal breathing with fetal movements
3. NST has a low false negative rate (< 1%) but high false positive rate (>50%)
4. Testing should be started at 20 weeks

Which of the statement(s) given above is/are correct?

(a) 1 and 3
(b) 2 only
(c) 3 only
(d) 1 and 4
43.
Which one of the following is a protective factor for endometrial hyperplasia?

(a) Diabetes
(b) Tamoxifen therapy
(c) Multiparity
(d) Delayed menopause
44.
A woman who is not breast feeding her newborn child is advised to use a contraceptive
method by:

(a) 3
rd
postpartum week
(b) 6
th
postpartum week
(c) 3
rd
postpartum month
(d) 6
th
postpartum month





45.
Pearl index for contraceptive effectiveness is calculated in terms of which of the following?

1. Pregnancy rate
2. Abortion rate
3. Hundred woman years
4. Thousand woman years

Select the correct answer using the code given below:

(a) 1 only
(b) 2 and 3
(c) 1, 2 and 4
(d) 1 and 3
46.
Indications for removal of IUDs are all EXCEPT:

(a) Perforation of uterus
(b) Cyclical menstrual bleeding
(c) Flaring up of salpingitis
(d) Pregnancy with IUD
47.
Contraindications for insertion of IUDs are all EXCEPT:

(a) Suspected pregnancy
(b) Trophoblastic disease
(c) Severe dysmenorrhea
(d) During cesarean section
48.
Which one of the following is NOT a contraindication for use of Mini pill?

(a) Pregnancy
(b) Breast feeding
(c) Thromboembolic disease
(d) History of breast cancer
49.
Which one of the following is the most commonly used surgical method/technique of female
sterilization as recommended by Government of India?

(a) Uchida technique
(b) Irving method
(c) Pomeroy?s method
(d) Madlener technique







50.
Which of the following is/are required for a registered medical practitioner to qualify for
performing Medical Termination of Pregnancy (MTP), as per revised rules of MTP Act?

1. Certified for assisting at least 15 MTP in an authorized centre
2. Diploma or degree in Obstetrics and Gynaecology
3. House surgeon training for 3 months in Obstetrics and Gynaecology
4. Certified training for 6 months in laparoscopic surgeries

Select the correct answer using the code given below:
(a) 1 only
(b) 2 only
(c) 1, 2 and 3
(d) 1, 2 and 4
51.
Which one of the following is NOT a support of uterus, preventing its descent?

(a) Endopelvic fascia
(b) Mackenrodt?s ligament
(c) Inguinal ligament
(d) Pubocervical ligament
52.
As per ICMR guidelines, which one of the following statements is true regarding effects of
COVID-19 on fetus according to current evidence?

(a) There is increased risk of early pregnancy loss
(b) COVID-19 virus is not teratogenic
(c) COVID-19 virus infection is an indication of MTP
(d) There is increased risk of fetal growth restriction
53.
As per ICMR guidelines, which one of the following statements is true regarding COVID-19
infection in pregnancy?

(a) Covid-19 pneumonia in pregnancy is more severe with poor recovery
(b) Pregnant women with heart disease are at higher risk
(c) Vaginal secretions always test positive for COVID-19 in pregnancy
(d) COVID-19 virus is secreted in breast milk
54.
Which one of the following is NOT a method of management of Deep Transverse Arrest with
the living fetus?

(a) Caesarean section
(b) Delivery by ventouse
(c) Delivery by application of forceps to the unrotated head
(d) Manual rotation and application of forceps





55.
Successful version of breech presentation is likely in case all of the following EXCEPT:

(a) Breech with extended legs
(b) Complete breech with sacroanterior position
(c) Non engaged breech
(d) Adequate amniotic fluid
56.
Implantation of a fertilised ovum occurs on which day following fertilisation?

(a) Day 6
(b) Day 10
(c) Day 14
(d) Day 20
57.
During total abdominal hysterectomy the ureter is likely to undergo injury or ligation during
the following steps EXCEPT:

(a) During division and ligation of the round ligaments
(b) During division and ligation of infundibulopelvic ligaments
(c) During division and ligation of mackenrodt?s and uterosacral ligaments
(d) At the vaginal angles while incising the vagina to remove the cervix with the
uterus
58.
The net effect of antenatal care has been the following EXCEPT:

(a) Reduction in maternal mortality
(b) Reduction in perinatal mortality
(c) Reduction in the incidence of institutional delivery
(d) Reduction in maternal morbidity
59.
Which one of the following is NOT a component of active phase in the partograph?

(a) Acceleration phase
(b) Phase of maximum slope
(c) Phase of deceleration
(d) Phase of expulsion
60.
From medicolegal point of view which one of the following is NOT a sign of previous child
birth?

(a) Perineum is lax and there is evidence of scarring
(b) Introitus is gaping and there is presence of carunculae myrtiformis
(c) Abdomen is lax and loose with striae and linea alba
(d) Conical cervix with round external os





61.
The components of partograph are all EXCEPT:

(a) Time
(b) Fetal heart rate
(c) Maternal respiratory rate
(d) Maternal urine analysis
62.
Which of the following information are provided by partograph?

1. Colour of liquor
2. Uterine contractions with duration and frequency
3. Dilatation of cervix

Select the correct answer using the code given below:

(a) 1 and 2 only
(b) 2 and 3 only
(c) 1 and 3 only
(d) 1, 2 and 3
63.
Intraoperative recognition of ureter is by which of the following features?

1. Transparent tubular appearance
2. Pale glistening appearance
3. Longitudinal vessels on surface
4. Circumferential vessels on surface

Select the correct answer using the code given below:

(a) 1 and 3
(b) 2 and 4
(c) 2 and 3
(d) 1 and 4
64.
Hysterosalpingography (HSG) is least helpful in detecting which of the following?

(a) Tubal patency
(b) Pelvic adhesions
(c) Asherman syndrome
(d) Congenital uterine anomaly









65.
Which of the following are characteristics of Trichomonas vaginitis?

1. Presence of greenish frothy discharge
2. Vaginal pH > 4.5
3. Presence of clue cells in microscopic examination
4. Strawberry spots on the vaginal mucosa

Select the correct answer using the code given below:

(a) 1, 2 and 3
(b) 1, 2 and 4
(c) 2, 3 and 4
(d) 1, 3 and 4
66.
Tumor marker of epithelial ovarian carcinoma is:

(a) Ca.125
(b) Alpha feto protein
(c) Beta HCG
(d) LDH
67.
The most common site of cervical cancer is:

(a) Endocervix
(b) Ectocervix
(c) Transformation zone
(d) Isthmus
68.
The placenta synthesizes all EXCEPT:

(a) Oestriol
(b) Corticotrophin releasing hormone
(c) PAPP-A(Pregnancy Associated Plasma Protein A)
(d) Dehydroepiandrosterone
69.
Withdrawal bleeding following administration of progesterone in a case of secondary
amenorrhea indicates all EXCEPT:

(a) Absence of pregnancy
(b) Production endogenous estrogen
(c) Endometrium is responsive to estrogen
(d) Defect in pituitary gland
70.
Monilial vaginitis is commonly associated with all EXCEPT:

(a) Prolonged antibiotic therapy
(b) Diabetes Mellitus
(c) Treatment of malaria with chloroquine
(d) Pregnancy
71.
Which one of the following is NOT a risk factor for the development of placenta previa?

(a) Maternal age
(b) Smoking
(c) Previous caesarean section
(d) Maternal anaemia
72.
Common clinical presentations of moderate to severe abruption are all EXCEPT:

(a) Uterine tenderness
(b) Fetal distress
(c) Unexplained pre term labour
(d) Prolonged labour
73.
Common trisomies resulting in spontaneous abortion are all EXCEPT:

(a) Trisomy 21
(b) Trisomy 18
(c) Trisomy 16
(d) Trisomy 1
74.
The initial prevention strategy for antiphospholipid syndrome will be:

1. Steroids
2. Heparin
3. Low dose aspirin
4. Progesterone support

Which of the above is/are correct?

(a) 2 and 3
(b) 3 and 4
(c) 3 only
(d) 1 and 4
75.
Diagnostic criteria for PCOD are:

1. Oligo/amenorrohoea
2. Hyperandrogenism
3. Polycystic ovaries on ultrasound

Which of the above are correct?

(a) 1 and 2 only
(b) 2 and 3 only
(c) 1 and 3 only
(d) 1, 2 and 3


76.
Which of the following symptoms can be associated with pelvic organ prolapse?

1. Difficulty in passing urine
2. Incomplete evacuation of urine
3. Urgency and frequency

Select the correct answer using the code given below:

(a) 1 and 2 only
(b) 2 and 3 only
(c) 1 and 3 only
(d) 1, 2 and 3
77.
A 30 year old lady, P
2
L
2
presents with painful unilateral swelling in vulva for 3 days. Which
of the following statements are true regarding the above case?

1. Bartholin?s abscess may be the likely diagnosis
2. It is to be managed by marsupialisation
3. Gonococcus is the most common pathogenic organism

Select the correct answer using the code given below:
(a) 1 and 3 only
(b) 3 only
(c) 1 and 2 only
(d) 1, 2 and 3
78.
Which one of the following is NOT a sign of separation of placenta?

(a) Uterus becomes globular, firm and ballotable
(b) The fundal height reduces further
(c) Slight bulging in the suprapubic region
(d) Apparent lengthenic of the cord with slight gush of vaginal bleeding
79.
Consider the following regarding examination of a rape victim:

1. Emergency pill is provided
2. Internal examination must be performed
3. HIV testing is done

Which of the above statements is/are correct?

(a) 1 and 3 only
(b) 2 only
(c) 1, 2 and 3
(d) 3 only




80.
Consider the following cardinal movements of mechanism of normal labor:
1. Engagement
2. Internal rotation
3. Flexion
4. Restitution
5. Crowning
6. External rotation

What is the correct sequence of movements in labor in occipito-lateral position?

(a) 1, 2, 3, 4, 5 and 6
(b) 1, 3, 2, 5, 4 and 6
(c) 2, 1, 3, 4, 5 and 6
(d) 3, 1, 2, 4, 6 and 5
81.
Which of the following represent the properties of an ideal disinfectant?

1. It is broad spectrum
2. It is fast acting
3. It is non-toxic

Select the correct answer using the code given below:
(a) 1 and 2 only
(b) 2 and 3 only
(c) 1 and 3 only
(d) 1, 2 and 3
82.
Which of the following measures can help reduce the risk of systemic hypertension?

1. Reduction in dietary intake of common salt
2. Controlling weight for age
3. Increasing potassium rich foods in the diet

Select the correct answer using the code given below:

(a) 1 and 2 only
(b) 2 and 3 only
(c) 1 and 3 only
(d) 1, 2 and 3








83.
Keeping biological determinants in perspective, consider the following statements:

1. Presence of a normal karyotype is the first requisite for human health
2. Genetic screening can play an important role in prevention of wide spectrum of
diseases
3. If an individual is allowed to live in healthy relationship with the environment, the
person?s genetic potentialities can transform into phenotypic realities

Which of the above statements is/are correct?

(a) 1 and 2 only
(b) 1 and 3 only
(c) 1, 2 and 3
(d) 2 and 3 only
84.
NITI Aayog has the following roles EXCEPT:

(a) It provides critical directional and strategic input in the development process
(b) It provides relevant technical advice with focus on technology upgradation
(c) It focuses on capacity building
(d) It focuses on the development of ?Referral Service Complex?
85.
Poor hand hygiene of a mess worker in a university college mess led to Hepatitis A cases in
the hostel inmates. What type of epidemic will this exposure present with?

1. Propagated
2. Common source-continuous exposure
3. Common source-point exposure

Select the correct answer using the code given below:

(a) 1 and 2
(b) 1 and 3
(c) 1 only
(d) 2 only












86.
What is the specificity of sputum microscopy in detection of Pulmonary Tuberculosis (PTB)
as per the information given below?

PTB Total
Sputum microscopy Present Absent
Positive 270 20 290
Negative 30 180 210
Total 300 200 500

(a) 10 %
(b) 36 %
(c) 90 %
(d) 94 %
87.
In a cohort of 500 women attending antenatal clinic, 70 % had ultrasonography (USG). This
cohort was followed up at delivery. Of the women who had USG, 70 delivered low birth
weight (LBW) babies; whereas of the women, who did not undergo USG, 50 delivered LBW
babies. The incidence of LBW babies among women who had USG is:

(a) 10 %
(b) 15 %
(c) 20 %
(d) 25 %
88.
Major source of vitamin K
1
is:

(a) Fresh dark green vegetables
(b) Exposure of body to sunlight
(c) Citrus fruits
(d) Foods rich in polyunsaturated fatty acids
89.
Which one of the following is NOT a function of Epidemiology?

(a) To study historically the rise and fall of disease in the population
(b) Searching for the causes and risk factor
(c) Identifying syndromes
(d) Making clinical diagnosis
90.
Which one of the following is NOT a contagious disease?

(a) Malaria
(b) Scabies
(c) Trachoma
(d) Leprosy


FirstRanker.com - FirstRanker's Choice
Combined Medical Services Examination-2020
Paper-II

1.
Indications for fasciotomy in compartment syndrome include all EXCEPT:

(a) Distal sensory disturbance
(b) Compartment pressure > 30 mm Hg
(c) Pain on passive movement of affected muscles
(d) Palpable distal pulses
2.
Which one of the following statements is NOT correct regarding Necrotising Soft Tissue
infections?

(a) Crepitus, skin blistering and focal skin gangrene are typical presenting features
(b) They are monomicrobial in nature
(c) Treatment consists of wide local excision and appropriate antibiotics
(d) Tissue biopsy is required for culture and diagnosis
3.
Which type of surgery is laparoscopic cholecystectomy classified as?

(a) Clean
(b) Clean contaminated
(c) Contaminated
(d) Dirty
4.
A 22-year female has presented with a history of malaise, cough, alternating constipation and
diarrhoea with intermittent abdominal pain for last 6 months. She also complains of
abdominal distension for last 2 days. On examination her abdomen has a doughy feel along
with an ill defined mass over the right lower quadrant. She is most likely suffering from:

(a) Appendicular lump
(b) Ileocaecal tuberculosis
(c) Carcinoma caecum
(d) Ovarian mass
5.
Consider the following statements regarding needle stick injuries:

1. Injured part should be washed under running water
2. Dominant index finger is the commonest site for needle stick injury
3. All needle stick injuries should be reported
4. Hepatitis/HIV testing should be done after needle stick injury

Which of the statements given above are correct?

(a) 1, 2 and 4
(b) 1, 2 and 3
(c) 1, 3 and 4
(d) 2, 3 and 4




6.
Consider the following statements regarding claudication:

1. It is a marker for silent coronary disease
2. Structured exercise program ( 2 hours per week for 3 months) leads to
improvement in symptoms
3. Diabetes mellitus increases the risk and severity of claudication
4. Beta blockers may exacerbate claudication

Which of the above statements are correct?

(a) 1 and 2 only
(b) 1, 3 and 4 only
(c) 2, 3 and 4 only
(d) 1, 2, 3 and 4
7.
A 50-year old lady underwent uneventful bariatric surgery for morbid obesity. On the third
post operative day, she develops breathless and pulmonary embolism is suspected. The next
investigation to confirm the diagnosis will be:

(a) Echocardiography
(b) Duplex venography
(c) CT pulmonary angiography
(d) MR angiography
8.
Medical management of thyrotoxic crisis includes all of the following EXCEPT:

(a) IV fluids
(b) IV propanolol
(c) IV hydrocortisone
(d) IV antibiotics
9.
A patient operated for a parotid gland tumour developed symptoms of sweating and
erytherma (flushing) over the region of surgical excision while eating. The probable
diagnosis is:

(a) Parotid gland fistula
(b) Sialadenitis
(c) Chronic wound infection
(d) Frey?s syndrome
10.
First line hormone therapy for post-menopausal woman with metastatic carcinoma breast is:

(a) Tamoxifen
(b) Ovarian suppression by surgery
(c) Antiprogestins
(d) Anastrazole



11.
All of the following are major subtypes of breast cancer based on Gene array analysis
EXCEPT:

(a) Luminal A and Luminal B
(b) Triple negative
(c) Her-2 receptor positive
(d) Oestrogen receptor positive
12.
All of the following are sequelae of peptic ulcer surgery EXCEPT:

(a) Bilious vomiting
(b) Dumping syndrome
(c) Diarrhoea
(d) Increased appetite
13.
The Child-Turcotte-Pugh (CTP) score for quantifying the severity of chronic liver disease
includes all variables EXCEPT:

(a) Serum bilirubin
(b) Serum albumin
(c) Serum creatinine
(d) INR (International Normalised Ratio)
14.
?Chain of Lakes? appearance due to sacculation with intervening short strictures of pancreatic
duct is seen on:

(a) ERCP
(b) CECT abdomen
(c) Plain X-ray abdomen
(d) Ultrasonography
15.
Which one of the following statements is NOT correct regarding Pyogenic Liver Abscess?

(a) Anorexia, fever, malaise and right upper quadrant abdominal discomfort are the
most common presenting features
(b) It is more common in elderly, diabetics and immunocompromised patients
(c) Treatment is with oral antibiotics alone
(d) Streptococcus milleri and escherichia coli are the most common causative
organisms
16.
?Swiss cheese defects? are seen during laparoscopic repair of:

(a) Ventral hernia
(b) Inguinal hernia
(c) Obturator hernia
(d) Femoral hernia



17.
Which of the following are correct regarding splenic artery aneurysm?

1. Main arterial trunk is the common site
2. Palpable thrill can be felt
3. It is symptomless unless it ruptures

Select the correct answer using the code given below:

(a) 1 and 2 only
(b) 2 and 3 only
(c) 1 and 3 only
(d) 1, 2 and 3
18.
Valentino?s syndrome is:

(a) Pain on per-vaginal examination in pelvic abscess
(b) Pain over left shoulder in left hypochondriac collection
(c) Pain over left groin in perirenal collection
(d) Pain in right iliac fossa in perforated peptic ulcer
19.
Spontaneous bacterial peritonitis occurs due to:

(a) duodenal stump blowout
(b) peptic ulcer perforation
(c) acute bacterial infection of ascites
(d) infection via fallopian tubes
20.
Structure not forming boundaries of the ?Triangle of doom? seen during laparoscopic
inguinal hernia surgery dissection is:

(a) Vas deferens
(b) Inferior epigastric artery
(c) Spermatic cord vessels
(d) Peritoneum
21.
The term mid-line shift is associated with:

(a) Head injury
(b) Chest injury
(c) Abdominal injury
(d) Limb injury
22.
Which one of the following cranial nerves does NOT supply to the external ear?

(a) Cranial nerve V
(b) Cranial nerve VI
(c) Cranial nerve VII
(d) Cranial nerve IX

23.
Left Internal Mammary Artery (LIMA) has become the conduit of choice for Left Anterior
Descending (LAD) artery during coronary artery bypass grafting because:

(a) Long term patency rates are more than 98%
(b) It is close to LAD
(c) Atherosclerosis is never seen in this vessel
(d) It is very easy to harvest
24.
A 50-year old male with significant smoking history presented in the surgical emergency
with sudden severe breathlessness. Chest X-ray shows right sided Pneumothorax. The
appropriate management requires:

(a) Aspiration of air with 16-18 G cannula
(b) Right chest drain of size 8-14 Fr
(c) Oxygen by face mask
(d) Mechanical ventilation
25.
Which one of the following statements regarding Felon is NOT correct?

(a) There is infection of the finger tip between specialised fibrous septa
(b) It is a painless condition
(c) Incision and drainage is the treatment of choice
(d) It is common in diabetics
26.
The most common site for osteosarcoma is:

(a) Proximal femur
(b) Distal femur
(c) Proximal humerus
(d) Distal humerus
27.
Rapid Sequence Induction is indicated in:

(a) Emergency surgery for intestinal obstruction
(b) Elective open hernia surgery
(c) Cardiopulmonary bypass surgery
(d) Elective laparoscopic surgery

28.
Which one of the following statements about Compartment Syndrome is NOT correct?

(a) It is commonest in a closed fracture
(b) Pain is on active movement but not on passive movement of muscles
(c) Fasciotomy is the treatment of choice
(d) Volkmann?s Ischaemic contractive is a late complication




29.
Which one of the following is NOT the strength of ultrasound as a diagnostic modality?

(a) No radiation
(b) Short learning curve
(c) Inexpensive
(d) Allows dynamic studies to be done
30.
Which of the following statements regarding lymphoedema following breast cancer treatment
are correct?

1. Incidence has decreased due to rarely combined therapy of axillary LN dissection and
radiotherapy
2. Precipitating cause like LN metastasis is a major determinant
3. The condition is often painful
4. Oedematous limb is susceptible to bacterial infection

Select the correct answer using the code given below:

(a) 1, 2 and 3
(b) 2, 3 and 4
(c) 1, 3 and 4
(d) 1, 2 and 4
31.
Which one of the following is NOT a risk factor for development of venous thrombosis in
surgical patients?

(a) Age > 60 years
(b) Pregnancy
(c) Obesity (BMI > 30 kg/m
2
)
(d) Diabetes ( HbA1c > 7.5%)
32.
Which one of the following is NOT a complication of massive blood transfusion?

(a) Coagulopathy
(b) Hypocalcaemia
(c) Hyperthermia
(d) Hyperkalemia
33.
Stage III ?Pressure sore? is full thickness skin loss extending:

(a) into subcutaneous tissue but not through fascia
(b) through subcutaneous tissue into fascia
(c) through subcutaneous tissue into fascia and muscles
(d) through subcutaneous tissue into fascia, muscles and bone



34.
During subclavian vein puncture in a surgical ward suddenly a patient developed severe
breathlessness. On auscultation breath sound was absent and the ipsilateral chest was
tympanitic on percussion. The probable diagnosis is:

(a) Iatrogenic pneumothorax
(b) Introgenic hemothorax
(c) Spontaneous pneumothorax
(d) Tension pneumothorax
35.
Kohler?s disease is avascular necrosis of :

(a) Lunate
(b) Capitellum of humerus
(c) First metatarsal head
(d) Navicular
36.
Which one of the following is NOT true of Pyoderma gangrenosum?

(a) It is characterized by cutaneous ulceration with purple undermined edges
(b) It is often secondary to heightened immunological reactivity from another disease
process
(c) Cultures often show Gram positive Staphylococci
(d) Lesions generally respond to steroids
37.
Which of the following is NOT a tissue repair surgery for inguinal hernia repair?

(a) Bassini?s repair
(b) Shouldice repair
(c) Stoppa?s repair
(d) Desarda repair
38.
Which one of the following type of meshes is recommended for intraperitoneal use in
abdominal wall hernia?
(a) Light weight, porous meshes
(b) Heavy weight, porous meshes
(c) Absorbable meshes
(d) Tissue separating meshes
39.
Which one of the following is NOT correct regarding MEN-1 syndrome?

(a) It involves parathyroid glands
(b) It involves pancreas
(c) It involves pituitary gland
(d) It involves pineal gland

40.
Which one of the following is NOT an electronic information site in surgery?

(a) Pubmed
(b) Embase
(c) Cochrane library
(d) National medical library
41.
Which one of the following statements regarding pre-conceptional counseling is NOT
correct?

(a) It is needed only in selected complicated pregnancies
(b) It helps in early detection of risk factors
(c) It helps in reducing maternal morbidity and mortality
(d) It is a part of preventive medicine
42.
Consider the following statements regarding Non Stress Test (NST):

1. Reactive NST indicates a healthy fetus
2. NST is an observed association of fetal breathing with fetal movements
3. NST has a low false negative rate (< 1%) but high false positive rate (>50%)
4. Testing should be started at 20 weeks

Which of the statement(s) given above is/are correct?

(a) 1 and 3
(b) 2 only
(c) 3 only
(d) 1 and 4
43.
Which one of the following is a protective factor for endometrial hyperplasia?

(a) Diabetes
(b) Tamoxifen therapy
(c) Multiparity
(d) Delayed menopause
44.
A woman who is not breast feeding her newborn child is advised to use a contraceptive
method by:

(a) 3
rd
postpartum week
(b) 6
th
postpartum week
(c) 3
rd
postpartum month
(d) 6
th
postpartum month





45.
Pearl index for contraceptive effectiveness is calculated in terms of which of the following?

1. Pregnancy rate
2. Abortion rate
3. Hundred woman years
4. Thousand woman years

Select the correct answer using the code given below:

(a) 1 only
(b) 2 and 3
(c) 1, 2 and 4
(d) 1 and 3
46.
Indications for removal of IUDs are all EXCEPT:

(a) Perforation of uterus
(b) Cyclical menstrual bleeding
(c) Flaring up of salpingitis
(d) Pregnancy with IUD
47.
Contraindications for insertion of IUDs are all EXCEPT:

(a) Suspected pregnancy
(b) Trophoblastic disease
(c) Severe dysmenorrhea
(d) During cesarean section
48.
Which one of the following is NOT a contraindication for use of Mini pill?

(a) Pregnancy
(b) Breast feeding
(c) Thromboembolic disease
(d) History of breast cancer
49.
Which one of the following is the most commonly used surgical method/technique of female
sterilization as recommended by Government of India?

(a) Uchida technique
(b) Irving method
(c) Pomeroy?s method
(d) Madlener technique







50.
Which of the following is/are required for a registered medical practitioner to qualify for
performing Medical Termination of Pregnancy (MTP), as per revised rules of MTP Act?

1. Certified for assisting at least 15 MTP in an authorized centre
2. Diploma or degree in Obstetrics and Gynaecology
3. House surgeon training for 3 months in Obstetrics and Gynaecology
4. Certified training for 6 months in laparoscopic surgeries

Select the correct answer using the code given below:
(a) 1 only
(b) 2 only
(c) 1, 2 and 3
(d) 1, 2 and 4
51.
Which one of the following is NOT a support of uterus, preventing its descent?

(a) Endopelvic fascia
(b) Mackenrodt?s ligament
(c) Inguinal ligament
(d) Pubocervical ligament
52.
As per ICMR guidelines, which one of the following statements is true regarding effects of
COVID-19 on fetus according to current evidence?

(a) There is increased risk of early pregnancy loss
(b) COVID-19 virus is not teratogenic
(c) COVID-19 virus infection is an indication of MTP
(d) There is increased risk of fetal growth restriction
53.
As per ICMR guidelines, which one of the following statements is true regarding COVID-19
infection in pregnancy?

(a) Covid-19 pneumonia in pregnancy is more severe with poor recovery
(b) Pregnant women with heart disease are at higher risk
(c) Vaginal secretions always test positive for COVID-19 in pregnancy
(d) COVID-19 virus is secreted in breast milk
54.
Which one of the following is NOT a method of management of Deep Transverse Arrest with
the living fetus?

(a) Caesarean section
(b) Delivery by ventouse
(c) Delivery by application of forceps to the unrotated head
(d) Manual rotation and application of forceps





55.
Successful version of breech presentation is likely in case all of the following EXCEPT:

(a) Breech with extended legs
(b) Complete breech with sacroanterior position
(c) Non engaged breech
(d) Adequate amniotic fluid
56.
Implantation of a fertilised ovum occurs on which day following fertilisation?

(a) Day 6
(b) Day 10
(c) Day 14
(d) Day 20
57.
During total abdominal hysterectomy the ureter is likely to undergo injury or ligation during
the following steps EXCEPT:

(a) During division and ligation of the round ligaments
(b) During division and ligation of infundibulopelvic ligaments
(c) During division and ligation of mackenrodt?s and uterosacral ligaments
(d) At the vaginal angles while incising the vagina to remove the cervix with the
uterus
58.
The net effect of antenatal care has been the following EXCEPT:

(a) Reduction in maternal mortality
(b) Reduction in perinatal mortality
(c) Reduction in the incidence of institutional delivery
(d) Reduction in maternal morbidity
59.
Which one of the following is NOT a component of active phase in the partograph?

(a) Acceleration phase
(b) Phase of maximum slope
(c) Phase of deceleration
(d) Phase of expulsion
60.
From medicolegal point of view which one of the following is NOT a sign of previous child
birth?

(a) Perineum is lax and there is evidence of scarring
(b) Introitus is gaping and there is presence of carunculae myrtiformis
(c) Abdomen is lax and loose with striae and linea alba
(d) Conical cervix with round external os





61.
The components of partograph are all EXCEPT:

(a) Time
(b) Fetal heart rate
(c) Maternal respiratory rate
(d) Maternal urine analysis
62.
Which of the following information are provided by partograph?

1. Colour of liquor
2. Uterine contractions with duration and frequency
3. Dilatation of cervix

Select the correct answer using the code given below:

(a) 1 and 2 only
(b) 2 and 3 only
(c) 1 and 3 only
(d) 1, 2 and 3
63.
Intraoperative recognition of ureter is by which of the following features?

1. Transparent tubular appearance
2. Pale glistening appearance
3. Longitudinal vessels on surface
4. Circumferential vessels on surface

Select the correct answer using the code given below:

(a) 1 and 3
(b) 2 and 4
(c) 2 and 3
(d) 1 and 4
64.
Hysterosalpingography (HSG) is least helpful in detecting which of the following?

(a) Tubal patency
(b) Pelvic adhesions
(c) Asherman syndrome
(d) Congenital uterine anomaly









65.
Which of the following are characteristics of Trichomonas vaginitis?

1. Presence of greenish frothy discharge
2. Vaginal pH > 4.5
3. Presence of clue cells in microscopic examination
4. Strawberry spots on the vaginal mucosa

Select the correct answer using the code given below:

(a) 1, 2 and 3
(b) 1, 2 and 4
(c) 2, 3 and 4
(d) 1, 3 and 4
66.
Tumor marker of epithelial ovarian carcinoma is:

(a) Ca.125
(b) Alpha feto protein
(c) Beta HCG
(d) LDH
67.
The most common site of cervical cancer is:

(a) Endocervix
(b) Ectocervix
(c) Transformation zone
(d) Isthmus
68.
The placenta synthesizes all EXCEPT:

(a) Oestriol
(b) Corticotrophin releasing hormone
(c) PAPP-A(Pregnancy Associated Plasma Protein A)
(d) Dehydroepiandrosterone
69.
Withdrawal bleeding following administration of progesterone in a case of secondary
amenorrhea indicates all EXCEPT:

(a) Absence of pregnancy
(b) Production endogenous estrogen
(c) Endometrium is responsive to estrogen
(d) Defect in pituitary gland
70.
Monilial vaginitis is commonly associated with all EXCEPT:

(a) Prolonged antibiotic therapy
(b) Diabetes Mellitus
(c) Treatment of malaria with chloroquine
(d) Pregnancy
71.
Which one of the following is NOT a risk factor for the development of placenta previa?

(a) Maternal age
(b) Smoking
(c) Previous caesarean section
(d) Maternal anaemia
72.
Common clinical presentations of moderate to severe abruption are all EXCEPT:

(a) Uterine tenderness
(b) Fetal distress
(c) Unexplained pre term labour
(d) Prolonged labour
73.
Common trisomies resulting in spontaneous abortion are all EXCEPT:

(a) Trisomy 21
(b) Trisomy 18
(c) Trisomy 16
(d) Trisomy 1
74.
The initial prevention strategy for antiphospholipid syndrome will be:

1. Steroids
2. Heparin
3. Low dose aspirin
4. Progesterone support

Which of the above is/are correct?

(a) 2 and 3
(b) 3 and 4
(c) 3 only
(d) 1 and 4
75.
Diagnostic criteria for PCOD are:

1. Oligo/amenorrohoea
2. Hyperandrogenism
3. Polycystic ovaries on ultrasound

Which of the above are correct?

(a) 1 and 2 only
(b) 2 and 3 only
(c) 1 and 3 only
(d) 1, 2 and 3


76.
Which of the following symptoms can be associated with pelvic organ prolapse?

1. Difficulty in passing urine
2. Incomplete evacuation of urine
3. Urgency and frequency

Select the correct answer using the code given below:

(a) 1 and 2 only
(b) 2 and 3 only
(c) 1 and 3 only
(d) 1, 2 and 3
77.
A 30 year old lady, P
2
L
2
presents with painful unilateral swelling in vulva for 3 days. Which
of the following statements are true regarding the above case?

1. Bartholin?s abscess may be the likely diagnosis
2. It is to be managed by marsupialisation
3. Gonococcus is the most common pathogenic organism

Select the correct answer using the code given below:
(a) 1 and 3 only
(b) 3 only
(c) 1 and 2 only
(d) 1, 2 and 3
78.
Which one of the following is NOT a sign of separation of placenta?

(a) Uterus becomes globular, firm and ballotable
(b) The fundal height reduces further
(c) Slight bulging in the suprapubic region
(d) Apparent lengthenic of the cord with slight gush of vaginal bleeding
79.
Consider the following regarding examination of a rape victim:

1. Emergency pill is provided
2. Internal examination must be performed
3. HIV testing is done

Which of the above statements is/are correct?

(a) 1 and 3 only
(b) 2 only
(c) 1, 2 and 3
(d) 3 only




80.
Consider the following cardinal movements of mechanism of normal labor:
1. Engagement
2. Internal rotation
3. Flexion
4. Restitution
5. Crowning
6. External rotation

What is the correct sequence of movements in labor in occipito-lateral position?

(a) 1, 2, 3, 4, 5 and 6
(b) 1, 3, 2, 5, 4 and 6
(c) 2, 1, 3, 4, 5 and 6
(d) 3, 1, 2, 4, 6 and 5
81.
Which of the following represent the properties of an ideal disinfectant?

1. It is broad spectrum
2. It is fast acting
3. It is non-toxic

Select the correct answer using the code given below:
(a) 1 and 2 only
(b) 2 and 3 only
(c) 1 and 3 only
(d) 1, 2 and 3
82.
Which of the following measures can help reduce the risk of systemic hypertension?

1. Reduction in dietary intake of common salt
2. Controlling weight for age
3. Increasing potassium rich foods in the diet

Select the correct answer using the code given below:

(a) 1 and 2 only
(b) 2 and 3 only
(c) 1 and 3 only
(d) 1, 2 and 3








83.
Keeping biological determinants in perspective, consider the following statements:

1. Presence of a normal karyotype is the first requisite for human health
2. Genetic screening can play an important role in prevention of wide spectrum of
diseases
3. If an individual is allowed to live in healthy relationship with the environment, the
person?s genetic potentialities can transform into phenotypic realities

Which of the above statements is/are correct?

(a) 1 and 2 only
(b) 1 and 3 only
(c) 1, 2 and 3
(d) 2 and 3 only
84.
NITI Aayog has the following roles EXCEPT:

(a) It provides critical directional and strategic input in the development process
(b) It provides relevant technical advice with focus on technology upgradation
(c) It focuses on capacity building
(d) It focuses on the development of ?Referral Service Complex?
85.
Poor hand hygiene of a mess worker in a university college mess led to Hepatitis A cases in
the hostel inmates. What type of epidemic will this exposure present with?

1. Propagated
2. Common source-continuous exposure
3. Common source-point exposure

Select the correct answer using the code given below:

(a) 1 and 2
(b) 1 and 3
(c) 1 only
(d) 2 only












86.
What is the specificity of sputum microscopy in detection of Pulmonary Tuberculosis (PTB)
as per the information given below?

PTB Total
Sputum microscopy Present Absent
Positive 270 20 290
Negative 30 180 210
Total 300 200 500

(a) 10 %
(b) 36 %
(c) 90 %
(d) 94 %
87.
In a cohort of 500 women attending antenatal clinic, 70 % had ultrasonography (USG). This
cohort was followed up at delivery. Of the women who had USG, 70 delivered low birth
weight (LBW) babies; whereas of the women, who did not undergo USG, 50 delivered LBW
babies. The incidence of LBW babies among women who had USG is:

(a) 10 %
(b) 15 %
(c) 20 %
(d) 25 %
88.
Major source of vitamin K
1
is:

(a) Fresh dark green vegetables
(b) Exposure of body to sunlight
(c) Citrus fruits
(d) Foods rich in polyunsaturated fatty acids
89.
Which one of the following is NOT a function of Epidemiology?

(a) To study historically the rise and fall of disease in the population
(b) Searching for the causes and risk factor
(c) Identifying syndromes
(d) Making clinical diagnosis
90.
Which one of the following is NOT a contagious disease?

(a) Malaria
(b) Scabies
(c) Trachoma
(d) Leprosy


91.
In primary immune response, how much more antigenic dose is required to induce IgG
antibodies as against the induction of IgM antibodies?

(a) 10 times more
(b) 25 times more
(c) 50 times more
(d) 100 times more
92.
Pentavalent vaccine provides protection against which of the following diseases?

(a) Diphtheria, Pertussis, Tuberculosis, Measles and Hepatitis B
(b) Diphtheria, Pertussis, Measles, Hepatitis B and Hib
(c) Diphtheria, Pertussis, Tetanus, Hepatitis B and Rubella
(d) Diphtheria, Pertussis, Tetanus, Hepatitis B and Hib
93.
Consider the following data for a country:
Population in 0-14 years of age ? 391,558,367
Population between 15?64 years of age ? 856,076,200
Population above 65 years of age ? 71,943,390
What shall be the dependency ratio of this country?

(a) 42.4 %
(b) 54.1 %
(c) 66.2 %
(d) 78.6 %
94.
Which one of the following statements regarding Rabies Immunoglobulin is NOT true?

(a) It should be administered only once as soon as possible after the initiation of post
exposure prophylaxis
(b) It should be administered all into or around the wound sites
(c) There is no scientific ground for performing a skin test prior to administering equine
immunoglobulin
(d) It can be administered till 15 days after the first dose of anti-rabies vaccine
95.
All of the following are true about Bedaquiline (BDQ) EXCEPT:

(a) It specifically targets mycobacterial ATP synthase
(b) It is a bacteriostatic drug
(c) It has extended half life
(d) It has high volume of tissue distribution
96.
Visual inspection based screening test with 5 % acetic acid is used for the screening of which
one of the following cancers?

(a) Lung cancer
(b) Cervix cancer
(c) Oral cancer
(d) Breast cancer
FirstRanker.com - FirstRanker's Choice
Combined Medical Services Examination-2020
Paper-II

1.
Indications for fasciotomy in compartment syndrome include all EXCEPT:

(a) Distal sensory disturbance
(b) Compartment pressure > 30 mm Hg
(c) Pain on passive movement of affected muscles
(d) Palpable distal pulses
2.
Which one of the following statements is NOT correct regarding Necrotising Soft Tissue
infections?

(a) Crepitus, skin blistering and focal skin gangrene are typical presenting features
(b) They are monomicrobial in nature
(c) Treatment consists of wide local excision and appropriate antibiotics
(d) Tissue biopsy is required for culture and diagnosis
3.
Which type of surgery is laparoscopic cholecystectomy classified as?

(a) Clean
(b) Clean contaminated
(c) Contaminated
(d) Dirty
4.
A 22-year female has presented with a history of malaise, cough, alternating constipation and
diarrhoea with intermittent abdominal pain for last 6 months. She also complains of
abdominal distension for last 2 days. On examination her abdomen has a doughy feel along
with an ill defined mass over the right lower quadrant. She is most likely suffering from:

(a) Appendicular lump
(b) Ileocaecal tuberculosis
(c) Carcinoma caecum
(d) Ovarian mass
5.
Consider the following statements regarding needle stick injuries:

1. Injured part should be washed under running water
2. Dominant index finger is the commonest site for needle stick injury
3. All needle stick injuries should be reported
4. Hepatitis/HIV testing should be done after needle stick injury

Which of the statements given above are correct?

(a) 1, 2 and 4
(b) 1, 2 and 3
(c) 1, 3 and 4
(d) 2, 3 and 4




6.
Consider the following statements regarding claudication:

1. It is a marker for silent coronary disease
2. Structured exercise program ( 2 hours per week for 3 months) leads to
improvement in symptoms
3. Diabetes mellitus increases the risk and severity of claudication
4. Beta blockers may exacerbate claudication

Which of the above statements are correct?

(a) 1 and 2 only
(b) 1, 3 and 4 only
(c) 2, 3 and 4 only
(d) 1, 2, 3 and 4
7.
A 50-year old lady underwent uneventful bariatric surgery for morbid obesity. On the third
post operative day, she develops breathless and pulmonary embolism is suspected. The next
investigation to confirm the diagnosis will be:

(a) Echocardiography
(b) Duplex venography
(c) CT pulmonary angiography
(d) MR angiography
8.
Medical management of thyrotoxic crisis includes all of the following EXCEPT:

(a) IV fluids
(b) IV propanolol
(c) IV hydrocortisone
(d) IV antibiotics
9.
A patient operated for a parotid gland tumour developed symptoms of sweating and
erytherma (flushing) over the region of surgical excision while eating. The probable
diagnosis is:

(a) Parotid gland fistula
(b) Sialadenitis
(c) Chronic wound infection
(d) Frey?s syndrome
10.
First line hormone therapy for post-menopausal woman with metastatic carcinoma breast is:

(a) Tamoxifen
(b) Ovarian suppression by surgery
(c) Antiprogestins
(d) Anastrazole



11.
All of the following are major subtypes of breast cancer based on Gene array analysis
EXCEPT:

(a) Luminal A and Luminal B
(b) Triple negative
(c) Her-2 receptor positive
(d) Oestrogen receptor positive
12.
All of the following are sequelae of peptic ulcer surgery EXCEPT:

(a) Bilious vomiting
(b) Dumping syndrome
(c) Diarrhoea
(d) Increased appetite
13.
The Child-Turcotte-Pugh (CTP) score for quantifying the severity of chronic liver disease
includes all variables EXCEPT:

(a) Serum bilirubin
(b) Serum albumin
(c) Serum creatinine
(d) INR (International Normalised Ratio)
14.
?Chain of Lakes? appearance due to sacculation with intervening short strictures of pancreatic
duct is seen on:

(a) ERCP
(b) CECT abdomen
(c) Plain X-ray abdomen
(d) Ultrasonography
15.
Which one of the following statements is NOT correct regarding Pyogenic Liver Abscess?

(a) Anorexia, fever, malaise and right upper quadrant abdominal discomfort are the
most common presenting features
(b) It is more common in elderly, diabetics and immunocompromised patients
(c) Treatment is with oral antibiotics alone
(d) Streptococcus milleri and escherichia coli are the most common causative
organisms
16.
?Swiss cheese defects? are seen during laparoscopic repair of:

(a) Ventral hernia
(b) Inguinal hernia
(c) Obturator hernia
(d) Femoral hernia



17.
Which of the following are correct regarding splenic artery aneurysm?

1. Main arterial trunk is the common site
2. Palpable thrill can be felt
3. It is symptomless unless it ruptures

Select the correct answer using the code given below:

(a) 1 and 2 only
(b) 2 and 3 only
(c) 1 and 3 only
(d) 1, 2 and 3
18.
Valentino?s syndrome is:

(a) Pain on per-vaginal examination in pelvic abscess
(b) Pain over left shoulder in left hypochondriac collection
(c) Pain over left groin in perirenal collection
(d) Pain in right iliac fossa in perforated peptic ulcer
19.
Spontaneous bacterial peritonitis occurs due to:

(a) duodenal stump blowout
(b) peptic ulcer perforation
(c) acute bacterial infection of ascites
(d) infection via fallopian tubes
20.
Structure not forming boundaries of the ?Triangle of doom? seen during laparoscopic
inguinal hernia surgery dissection is:

(a) Vas deferens
(b) Inferior epigastric artery
(c) Spermatic cord vessels
(d) Peritoneum
21.
The term mid-line shift is associated with:

(a) Head injury
(b) Chest injury
(c) Abdominal injury
(d) Limb injury
22.
Which one of the following cranial nerves does NOT supply to the external ear?

(a) Cranial nerve V
(b) Cranial nerve VI
(c) Cranial nerve VII
(d) Cranial nerve IX

23.
Left Internal Mammary Artery (LIMA) has become the conduit of choice for Left Anterior
Descending (LAD) artery during coronary artery bypass grafting because:

(a) Long term patency rates are more than 98%
(b) It is close to LAD
(c) Atherosclerosis is never seen in this vessel
(d) It is very easy to harvest
24.
A 50-year old male with significant smoking history presented in the surgical emergency
with sudden severe breathlessness. Chest X-ray shows right sided Pneumothorax. The
appropriate management requires:

(a) Aspiration of air with 16-18 G cannula
(b) Right chest drain of size 8-14 Fr
(c) Oxygen by face mask
(d) Mechanical ventilation
25.
Which one of the following statements regarding Felon is NOT correct?

(a) There is infection of the finger tip between specialised fibrous septa
(b) It is a painless condition
(c) Incision and drainage is the treatment of choice
(d) It is common in diabetics
26.
The most common site for osteosarcoma is:

(a) Proximal femur
(b) Distal femur
(c) Proximal humerus
(d) Distal humerus
27.
Rapid Sequence Induction is indicated in:

(a) Emergency surgery for intestinal obstruction
(b) Elective open hernia surgery
(c) Cardiopulmonary bypass surgery
(d) Elective laparoscopic surgery

28.
Which one of the following statements about Compartment Syndrome is NOT correct?

(a) It is commonest in a closed fracture
(b) Pain is on active movement but not on passive movement of muscles
(c) Fasciotomy is the treatment of choice
(d) Volkmann?s Ischaemic contractive is a late complication




29.
Which one of the following is NOT the strength of ultrasound as a diagnostic modality?

(a) No radiation
(b) Short learning curve
(c) Inexpensive
(d) Allows dynamic studies to be done
30.
Which of the following statements regarding lymphoedema following breast cancer treatment
are correct?

1. Incidence has decreased due to rarely combined therapy of axillary LN dissection and
radiotherapy
2. Precipitating cause like LN metastasis is a major determinant
3. The condition is often painful
4. Oedematous limb is susceptible to bacterial infection

Select the correct answer using the code given below:

(a) 1, 2 and 3
(b) 2, 3 and 4
(c) 1, 3 and 4
(d) 1, 2 and 4
31.
Which one of the following is NOT a risk factor for development of venous thrombosis in
surgical patients?

(a) Age > 60 years
(b) Pregnancy
(c) Obesity (BMI > 30 kg/m
2
)
(d) Diabetes ( HbA1c > 7.5%)
32.
Which one of the following is NOT a complication of massive blood transfusion?

(a) Coagulopathy
(b) Hypocalcaemia
(c) Hyperthermia
(d) Hyperkalemia
33.
Stage III ?Pressure sore? is full thickness skin loss extending:

(a) into subcutaneous tissue but not through fascia
(b) through subcutaneous tissue into fascia
(c) through subcutaneous tissue into fascia and muscles
(d) through subcutaneous tissue into fascia, muscles and bone



34.
During subclavian vein puncture in a surgical ward suddenly a patient developed severe
breathlessness. On auscultation breath sound was absent and the ipsilateral chest was
tympanitic on percussion. The probable diagnosis is:

(a) Iatrogenic pneumothorax
(b) Introgenic hemothorax
(c) Spontaneous pneumothorax
(d) Tension pneumothorax
35.
Kohler?s disease is avascular necrosis of :

(a) Lunate
(b) Capitellum of humerus
(c) First metatarsal head
(d) Navicular
36.
Which one of the following is NOT true of Pyoderma gangrenosum?

(a) It is characterized by cutaneous ulceration with purple undermined edges
(b) It is often secondary to heightened immunological reactivity from another disease
process
(c) Cultures often show Gram positive Staphylococci
(d) Lesions generally respond to steroids
37.
Which of the following is NOT a tissue repair surgery for inguinal hernia repair?

(a) Bassini?s repair
(b) Shouldice repair
(c) Stoppa?s repair
(d) Desarda repair
38.
Which one of the following type of meshes is recommended for intraperitoneal use in
abdominal wall hernia?
(a) Light weight, porous meshes
(b) Heavy weight, porous meshes
(c) Absorbable meshes
(d) Tissue separating meshes
39.
Which one of the following is NOT correct regarding MEN-1 syndrome?

(a) It involves parathyroid glands
(b) It involves pancreas
(c) It involves pituitary gland
(d) It involves pineal gland

40.
Which one of the following is NOT an electronic information site in surgery?

(a) Pubmed
(b) Embase
(c) Cochrane library
(d) National medical library
41.
Which one of the following statements regarding pre-conceptional counseling is NOT
correct?

(a) It is needed only in selected complicated pregnancies
(b) It helps in early detection of risk factors
(c) It helps in reducing maternal morbidity and mortality
(d) It is a part of preventive medicine
42.
Consider the following statements regarding Non Stress Test (NST):

1. Reactive NST indicates a healthy fetus
2. NST is an observed association of fetal breathing with fetal movements
3. NST has a low false negative rate (< 1%) but high false positive rate (>50%)
4. Testing should be started at 20 weeks

Which of the statement(s) given above is/are correct?

(a) 1 and 3
(b) 2 only
(c) 3 only
(d) 1 and 4
43.
Which one of the following is a protective factor for endometrial hyperplasia?

(a) Diabetes
(b) Tamoxifen therapy
(c) Multiparity
(d) Delayed menopause
44.
A woman who is not breast feeding her newborn child is advised to use a contraceptive
method by:

(a) 3
rd
postpartum week
(b) 6
th
postpartum week
(c) 3
rd
postpartum month
(d) 6
th
postpartum month





45.
Pearl index for contraceptive effectiveness is calculated in terms of which of the following?

1. Pregnancy rate
2. Abortion rate
3. Hundred woman years
4. Thousand woman years

Select the correct answer using the code given below:

(a) 1 only
(b) 2 and 3
(c) 1, 2 and 4
(d) 1 and 3
46.
Indications for removal of IUDs are all EXCEPT:

(a) Perforation of uterus
(b) Cyclical menstrual bleeding
(c) Flaring up of salpingitis
(d) Pregnancy with IUD
47.
Contraindications for insertion of IUDs are all EXCEPT:

(a) Suspected pregnancy
(b) Trophoblastic disease
(c) Severe dysmenorrhea
(d) During cesarean section
48.
Which one of the following is NOT a contraindication for use of Mini pill?

(a) Pregnancy
(b) Breast feeding
(c) Thromboembolic disease
(d) History of breast cancer
49.
Which one of the following is the most commonly used surgical method/technique of female
sterilization as recommended by Government of India?

(a) Uchida technique
(b) Irving method
(c) Pomeroy?s method
(d) Madlener technique







50.
Which of the following is/are required for a registered medical practitioner to qualify for
performing Medical Termination of Pregnancy (MTP), as per revised rules of MTP Act?

1. Certified for assisting at least 15 MTP in an authorized centre
2. Diploma or degree in Obstetrics and Gynaecology
3. House surgeon training for 3 months in Obstetrics and Gynaecology
4. Certified training for 6 months in laparoscopic surgeries

Select the correct answer using the code given below:
(a) 1 only
(b) 2 only
(c) 1, 2 and 3
(d) 1, 2 and 4
51.
Which one of the following is NOT a support of uterus, preventing its descent?

(a) Endopelvic fascia
(b) Mackenrodt?s ligament
(c) Inguinal ligament
(d) Pubocervical ligament
52.
As per ICMR guidelines, which one of the following statements is true regarding effects of
COVID-19 on fetus according to current evidence?

(a) There is increased risk of early pregnancy loss
(b) COVID-19 virus is not teratogenic
(c) COVID-19 virus infection is an indication of MTP
(d) There is increased risk of fetal growth restriction
53.
As per ICMR guidelines, which one of the following statements is true regarding COVID-19
infection in pregnancy?

(a) Covid-19 pneumonia in pregnancy is more severe with poor recovery
(b) Pregnant women with heart disease are at higher risk
(c) Vaginal secretions always test positive for COVID-19 in pregnancy
(d) COVID-19 virus is secreted in breast milk
54.
Which one of the following is NOT a method of management of Deep Transverse Arrest with
the living fetus?

(a) Caesarean section
(b) Delivery by ventouse
(c) Delivery by application of forceps to the unrotated head
(d) Manual rotation and application of forceps





55.
Successful version of breech presentation is likely in case all of the following EXCEPT:

(a) Breech with extended legs
(b) Complete breech with sacroanterior position
(c) Non engaged breech
(d) Adequate amniotic fluid
56.
Implantation of a fertilised ovum occurs on which day following fertilisation?

(a) Day 6
(b) Day 10
(c) Day 14
(d) Day 20
57.
During total abdominal hysterectomy the ureter is likely to undergo injury or ligation during
the following steps EXCEPT:

(a) During division and ligation of the round ligaments
(b) During division and ligation of infundibulopelvic ligaments
(c) During division and ligation of mackenrodt?s and uterosacral ligaments
(d) At the vaginal angles while incising the vagina to remove the cervix with the
uterus
58.
The net effect of antenatal care has been the following EXCEPT:

(a) Reduction in maternal mortality
(b) Reduction in perinatal mortality
(c) Reduction in the incidence of institutional delivery
(d) Reduction in maternal morbidity
59.
Which one of the following is NOT a component of active phase in the partograph?

(a) Acceleration phase
(b) Phase of maximum slope
(c) Phase of deceleration
(d) Phase of expulsion
60.
From medicolegal point of view which one of the following is NOT a sign of previous child
birth?

(a) Perineum is lax and there is evidence of scarring
(b) Introitus is gaping and there is presence of carunculae myrtiformis
(c) Abdomen is lax and loose with striae and linea alba
(d) Conical cervix with round external os





61.
The components of partograph are all EXCEPT:

(a) Time
(b) Fetal heart rate
(c) Maternal respiratory rate
(d) Maternal urine analysis
62.
Which of the following information are provided by partograph?

1. Colour of liquor
2. Uterine contractions with duration and frequency
3. Dilatation of cervix

Select the correct answer using the code given below:

(a) 1 and 2 only
(b) 2 and 3 only
(c) 1 and 3 only
(d) 1, 2 and 3
63.
Intraoperative recognition of ureter is by which of the following features?

1. Transparent tubular appearance
2. Pale glistening appearance
3. Longitudinal vessels on surface
4. Circumferential vessels on surface

Select the correct answer using the code given below:

(a) 1 and 3
(b) 2 and 4
(c) 2 and 3
(d) 1 and 4
64.
Hysterosalpingography (HSG) is least helpful in detecting which of the following?

(a) Tubal patency
(b) Pelvic adhesions
(c) Asherman syndrome
(d) Congenital uterine anomaly









65.
Which of the following are characteristics of Trichomonas vaginitis?

1. Presence of greenish frothy discharge
2. Vaginal pH > 4.5
3. Presence of clue cells in microscopic examination
4. Strawberry spots on the vaginal mucosa

Select the correct answer using the code given below:

(a) 1, 2 and 3
(b) 1, 2 and 4
(c) 2, 3 and 4
(d) 1, 3 and 4
66.
Tumor marker of epithelial ovarian carcinoma is:

(a) Ca.125
(b) Alpha feto protein
(c) Beta HCG
(d) LDH
67.
The most common site of cervical cancer is:

(a) Endocervix
(b) Ectocervix
(c) Transformation zone
(d) Isthmus
68.
The placenta synthesizes all EXCEPT:

(a) Oestriol
(b) Corticotrophin releasing hormone
(c) PAPP-A(Pregnancy Associated Plasma Protein A)
(d) Dehydroepiandrosterone
69.
Withdrawal bleeding following administration of progesterone in a case of secondary
amenorrhea indicates all EXCEPT:

(a) Absence of pregnancy
(b) Production endogenous estrogen
(c) Endometrium is responsive to estrogen
(d) Defect in pituitary gland
70.
Monilial vaginitis is commonly associated with all EXCEPT:

(a) Prolonged antibiotic therapy
(b) Diabetes Mellitus
(c) Treatment of malaria with chloroquine
(d) Pregnancy
71.
Which one of the following is NOT a risk factor for the development of placenta previa?

(a) Maternal age
(b) Smoking
(c) Previous caesarean section
(d) Maternal anaemia
72.
Common clinical presentations of moderate to severe abruption are all EXCEPT:

(a) Uterine tenderness
(b) Fetal distress
(c) Unexplained pre term labour
(d) Prolonged labour
73.
Common trisomies resulting in spontaneous abortion are all EXCEPT:

(a) Trisomy 21
(b) Trisomy 18
(c) Trisomy 16
(d) Trisomy 1
74.
The initial prevention strategy for antiphospholipid syndrome will be:

1. Steroids
2. Heparin
3. Low dose aspirin
4. Progesterone support

Which of the above is/are correct?

(a) 2 and 3
(b) 3 and 4
(c) 3 only
(d) 1 and 4
75.
Diagnostic criteria for PCOD are:

1. Oligo/amenorrohoea
2. Hyperandrogenism
3. Polycystic ovaries on ultrasound

Which of the above are correct?

(a) 1 and 2 only
(b) 2 and 3 only
(c) 1 and 3 only
(d) 1, 2 and 3


76.
Which of the following symptoms can be associated with pelvic organ prolapse?

1. Difficulty in passing urine
2. Incomplete evacuation of urine
3. Urgency and frequency

Select the correct answer using the code given below:

(a) 1 and 2 only
(b) 2 and 3 only
(c) 1 and 3 only
(d) 1, 2 and 3
77.
A 30 year old lady, P
2
L
2
presents with painful unilateral swelling in vulva for 3 days. Which
of the following statements are true regarding the above case?

1. Bartholin?s abscess may be the likely diagnosis
2. It is to be managed by marsupialisation
3. Gonococcus is the most common pathogenic organism

Select the correct answer using the code given below:
(a) 1 and 3 only
(b) 3 only
(c) 1 and 2 only
(d) 1, 2 and 3
78.
Which one of the following is NOT a sign of separation of placenta?

(a) Uterus becomes globular, firm and ballotable
(b) The fundal height reduces further
(c) Slight bulging in the suprapubic region
(d) Apparent lengthenic of the cord with slight gush of vaginal bleeding
79.
Consider the following regarding examination of a rape victim:

1. Emergency pill is provided
2. Internal examination must be performed
3. HIV testing is done

Which of the above statements is/are correct?

(a) 1 and 3 only
(b) 2 only
(c) 1, 2 and 3
(d) 3 only




80.
Consider the following cardinal movements of mechanism of normal labor:
1. Engagement
2. Internal rotation
3. Flexion
4. Restitution
5. Crowning
6. External rotation

What is the correct sequence of movements in labor in occipito-lateral position?

(a) 1, 2, 3, 4, 5 and 6
(b) 1, 3, 2, 5, 4 and 6
(c) 2, 1, 3, 4, 5 and 6
(d) 3, 1, 2, 4, 6 and 5
81.
Which of the following represent the properties of an ideal disinfectant?

1. It is broad spectrum
2. It is fast acting
3. It is non-toxic

Select the correct answer using the code given below:
(a) 1 and 2 only
(b) 2 and 3 only
(c) 1 and 3 only
(d) 1, 2 and 3
82.
Which of the following measures can help reduce the risk of systemic hypertension?

1. Reduction in dietary intake of common salt
2. Controlling weight for age
3. Increasing potassium rich foods in the diet

Select the correct answer using the code given below:

(a) 1 and 2 only
(b) 2 and 3 only
(c) 1 and 3 only
(d) 1, 2 and 3








83.
Keeping biological determinants in perspective, consider the following statements:

1. Presence of a normal karyotype is the first requisite for human health
2. Genetic screening can play an important role in prevention of wide spectrum of
diseases
3. If an individual is allowed to live in healthy relationship with the environment, the
person?s genetic potentialities can transform into phenotypic realities

Which of the above statements is/are correct?

(a) 1 and 2 only
(b) 1 and 3 only
(c) 1, 2 and 3
(d) 2 and 3 only
84.
NITI Aayog has the following roles EXCEPT:

(a) It provides critical directional and strategic input in the development process
(b) It provides relevant technical advice with focus on technology upgradation
(c) It focuses on capacity building
(d) It focuses on the development of ?Referral Service Complex?
85.
Poor hand hygiene of a mess worker in a university college mess led to Hepatitis A cases in
the hostel inmates. What type of epidemic will this exposure present with?

1. Propagated
2. Common source-continuous exposure
3. Common source-point exposure

Select the correct answer using the code given below:

(a) 1 and 2
(b) 1 and 3
(c) 1 only
(d) 2 only












86.
What is the specificity of sputum microscopy in detection of Pulmonary Tuberculosis (PTB)
as per the information given below?

PTB Total
Sputum microscopy Present Absent
Positive 270 20 290
Negative 30 180 210
Total 300 200 500

(a) 10 %
(b) 36 %
(c) 90 %
(d) 94 %
87.
In a cohort of 500 women attending antenatal clinic, 70 % had ultrasonography (USG). This
cohort was followed up at delivery. Of the women who had USG, 70 delivered low birth
weight (LBW) babies; whereas of the women, who did not undergo USG, 50 delivered LBW
babies. The incidence of LBW babies among women who had USG is:

(a) 10 %
(b) 15 %
(c) 20 %
(d) 25 %
88.
Major source of vitamin K
1
is:

(a) Fresh dark green vegetables
(b) Exposure of body to sunlight
(c) Citrus fruits
(d) Foods rich in polyunsaturated fatty acids
89.
Which one of the following is NOT a function of Epidemiology?

(a) To study historically the rise and fall of disease in the population
(b) Searching for the causes and risk factor
(c) Identifying syndromes
(d) Making clinical diagnosis
90.
Which one of the following is NOT a contagious disease?

(a) Malaria
(b) Scabies
(c) Trachoma
(d) Leprosy


91.
In primary immune response, how much more antigenic dose is required to induce IgG
antibodies as against the induction of IgM antibodies?

(a) 10 times more
(b) 25 times more
(c) 50 times more
(d) 100 times more
92.
Pentavalent vaccine provides protection against which of the following diseases?

(a) Diphtheria, Pertussis, Tuberculosis, Measles and Hepatitis B
(b) Diphtheria, Pertussis, Measles, Hepatitis B and Hib
(c) Diphtheria, Pertussis, Tetanus, Hepatitis B and Rubella
(d) Diphtheria, Pertussis, Tetanus, Hepatitis B and Hib
93.
Consider the following data for a country:
Population in 0-14 years of age ? 391,558,367
Population between 15?64 years of age ? 856,076,200
Population above 65 years of age ? 71,943,390
What shall be the dependency ratio of this country?

(a) 42.4 %
(b) 54.1 %
(c) 66.2 %
(d) 78.6 %
94.
Which one of the following statements regarding Rabies Immunoglobulin is NOT true?

(a) It should be administered only once as soon as possible after the initiation of post
exposure prophylaxis
(b) It should be administered all into or around the wound sites
(c) There is no scientific ground for performing a skin test prior to administering equine
immunoglobulin
(d) It can be administered till 15 days after the first dose of anti-rabies vaccine
95.
All of the following are true about Bedaquiline (BDQ) EXCEPT:

(a) It specifically targets mycobacterial ATP synthase
(b) It is a bacteriostatic drug
(c) It has extended half life
(d) It has high volume of tissue distribution
96.
Visual inspection based screening test with 5 % acetic acid is used for the screening of which
one of the following cancers?

(a) Lung cancer
(b) Cervix cancer
(c) Oral cancer
(d) Breast cancer
97.
Predictive accuracy of a screening test depends on the following EXCEPT:

(a) Disease prevalence
(b) Disease incidence
(c) Sensitivity of screening test
(d) Specificity of screening test
98.
How much of Zinc supplement is recommended by WHO and UNICEF for infants less than 6
months of age after an episode of acute diarrhoea?

(a) 20 mg per day for 10?14 days
(b) 10 mg per day for 10?14 days
(c) 5 mg per day for 7 days
(d) 6 mg per day for 7 days
99.
Which one of the following statements regarding sequential administration of Inactivated
Polio Vaccine (IPV) and Oral Polio Vaccine (OPV) is NOT correct?

(a) It will be cost effective in developing countries for Polio prevention
(b) The combined schedules of IPV and OPV appear to reduce or prevent Vaccine
Associated Paralytic Polio (VAPP)
(c) Intestinal mucosal immunity is lost due to IPV administration
(d) IPV and OPV together may optimize both the humoral and mucosal immunogenicity
of Polio vaccine
100.
What is the recommended dose regimen of Vitamin A for the treatment of early stages of
Xerophthalmia?

(a) Single massive dose of 2 lac International Units (IU)
(b) 2 lac IU on two successive days
(c) 2 doses of 1 lac IU in two successive days
(d) 2 doses of 1 lac IU at a gap of one week
101.
Which is the most specific causative agent of Rabies?

(a) Lyssavirus serotype 1
(b) Lyssavirus serotype 2
(c) Lyssavirus serotype 3
(d) Lyssavirus serotype 4
102.
Which one of the following is the antibiotic of choice for the prevention of Rheumatic heart
disease?

(a) Benzathine Benzyl Penicillin
(b) Procaine Penicillin
(c) Doxycycline
(d) Ciprofloxacin


FirstRanker.com - FirstRanker's Choice
Combined Medical Services Examination-2020
Paper-II

1.
Indications for fasciotomy in compartment syndrome include all EXCEPT:

(a) Distal sensory disturbance
(b) Compartment pressure > 30 mm Hg
(c) Pain on passive movement of affected muscles
(d) Palpable distal pulses
2.
Which one of the following statements is NOT correct regarding Necrotising Soft Tissue
infections?

(a) Crepitus, skin blistering and focal skin gangrene are typical presenting features
(b) They are monomicrobial in nature
(c) Treatment consists of wide local excision and appropriate antibiotics
(d) Tissue biopsy is required for culture and diagnosis
3.
Which type of surgery is laparoscopic cholecystectomy classified as?

(a) Clean
(b) Clean contaminated
(c) Contaminated
(d) Dirty
4.
A 22-year female has presented with a history of malaise, cough, alternating constipation and
diarrhoea with intermittent abdominal pain for last 6 months. She also complains of
abdominal distension for last 2 days. On examination her abdomen has a doughy feel along
with an ill defined mass over the right lower quadrant. She is most likely suffering from:

(a) Appendicular lump
(b) Ileocaecal tuberculosis
(c) Carcinoma caecum
(d) Ovarian mass
5.
Consider the following statements regarding needle stick injuries:

1. Injured part should be washed under running water
2. Dominant index finger is the commonest site for needle stick injury
3. All needle stick injuries should be reported
4. Hepatitis/HIV testing should be done after needle stick injury

Which of the statements given above are correct?

(a) 1, 2 and 4
(b) 1, 2 and 3
(c) 1, 3 and 4
(d) 2, 3 and 4




6.
Consider the following statements regarding claudication:

1. It is a marker for silent coronary disease
2. Structured exercise program ( 2 hours per week for 3 months) leads to
improvement in symptoms
3. Diabetes mellitus increases the risk and severity of claudication
4. Beta blockers may exacerbate claudication

Which of the above statements are correct?

(a) 1 and 2 only
(b) 1, 3 and 4 only
(c) 2, 3 and 4 only
(d) 1, 2, 3 and 4
7.
A 50-year old lady underwent uneventful bariatric surgery for morbid obesity. On the third
post operative day, she develops breathless and pulmonary embolism is suspected. The next
investigation to confirm the diagnosis will be:

(a) Echocardiography
(b) Duplex venography
(c) CT pulmonary angiography
(d) MR angiography
8.
Medical management of thyrotoxic crisis includes all of the following EXCEPT:

(a) IV fluids
(b) IV propanolol
(c) IV hydrocortisone
(d) IV antibiotics
9.
A patient operated for a parotid gland tumour developed symptoms of sweating and
erytherma (flushing) over the region of surgical excision while eating. The probable
diagnosis is:

(a) Parotid gland fistula
(b) Sialadenitis
(c) Chronic wound infection
(d) Frey?s syndrome
10.
First line hormone therapy for post-menopausal woman with metastatic carcinoma breast is:

(a) Tamoxifen
(b) Ovarian suppression by surgery
(c) Antiprogestins
(d) Anastrazole



11.
All of the following are major subtypes of breast cancer based on Gene array analysis
EXCEPT:

(a) Luminal A and Luminal B
(b) Triple negative
(c) Her-2 receptor positive
(d) Oestrogen receptor positive
12.
All of the following are sequelae of peptic ulcer surgery EXCEPT:

(a) Bilious vomiting
(b) Dumping syndrome
(c) Diarrhoea
(d) Increased appetite
13.
The Child-Turcotte-Pugh (CTP) score for quantifying the severity of chronic liver disease
includes all variables EXCEPT:

(a) Serum bilirubin
(b) Serum albumin
(c) Serum creatinine
(d) INR (International Normalised Ratio)
14.
?Chain of Lakes? appearance due to sacculation with intervening short strictures of pancreatic
duct is seen on:

(a) ERCP
(b) CECT abdomen
(c) Plain X-ray abdomen
(d) Ultrasonography
15.
Which one of the following statements is NOT correct regarding Pyogenic Liver Abscess?

(a) Anorexia, fever, malaise and right upper quadrant abdominal discomfort are the
most common presenting features
(b) It is more common in elderly, diabetics and immunocompromised patients
(c) Treatment is with oral antibiotics alone
(d) Streptococcus milleri and escherichia coli are the most common causative
organisms
16.
?Swiss cheese defects? are seen during laparoscopic repair of:

(a) Ventral hernia
(b) Inguinal hernia
(c) Obturator hernia
(d) Femoral hernia



17.
Which of the following are correct regarding splenic artery aneurysm?

1. Main arterial trunk is the common site
2. Palpable thrill can be felt
3. It is symptomless unless it ruptures

Select the correct answer using the code given below:

(a) 1 and 2 only
(b) 2 and 3 only
(c) 1 and 3 only
(d) 1, 2 and 3
18.
Valentino?s syndrome is:

(a) Pain on per-vaginal examination in pelvic abscess
(b) Pain over left shoulder in left hypochondriac collection
(c) Pain over left groin in perirenal collection
(d) Pain in right iliac fossa in perforated peptic ulcer
19.
Spontaneous bacterial peritonitis occurs due to:

(a) duodenal stump blowout
(b) peptic ulcer perforation
(c) acute bacterial infection of ascites
(d) infection via fallopian tubes
20.
Structure not forming boundaries of the ?Triangle of doom? seen during laparoscopic
inguinal hernia surgery dissection is:

(a) Vas deferens
(b) Inferior epigastric artery
(c) Spermatic cord vessels
(d) Peritoneum
21.
The term mid-line shift is associated with:

(a) Head injury
(b) Chest injury
(c) Abdominal injury
(d) Limb injury
22.
Which one of the following cranial nerves does NOT supply to the external ear?

(a) Cranial nerve V
(b) Cranial nerve VI
(c) Cranial nerve VII
(d) Cranial nerve IX

23.
Left Internal Mammary Artery (LIMA) has become the conduit of choice for Left Anterior
Descending (LAD) artery during coronary artery bypass grafting because:

(a) Long term patency rates are more than 98%
(b) It is close to LAD
(c) Atherosclerosis is never seen in this vessel
(d) It is very easy to harvest
24.
A 50-year old male with significant smoking history presented in the surgical emergency
with sudden severe breathlessness. Chest X-ray shows right sided Pneumothorax. The
appropriate management requires:

(a) Aspiration of air with 16-18 G cannula
(b) Right chest drain of size 8-14 Fr
(c) Oxygen by face mask
(d) Mechanical ventilation
25.
Which one of the following statements regarding Felon is NOT correct?

(a) There is infection of the finger tip between specialised fibrous septa
(b) It is a painless condition
(c) Incision and drainage is the treatment of choice
(d) It is common in diabetics
26.
The most common site for osteosarcoma is:

(a) Proximal femur
(b) Distal femur
(c) Proximal humerus
(d) Distal humerus
27.
Rapid Sequence Induction is indicated in:

(a) Emergency surgery for intestinal obstruction
(b) Elective open hernia surgery
(c) Cardiopulmonary bypass surgery
(d) Elective laparoscopic surgery

28.
Which one of the following statements about Compartment Syndrome is NOT correct?

(a) It is commonest in a closed fracture
(b) Pain is on active movement but not on passive movement of muscles
(c) Fasciotomy is the treatment of choice
(d) Volkmann?s Ischaemic contractive is a late complication




29.
Which one of the following is NOT the strength of ultrasound as a diagnostic modality?

(a) No radiation
(b) Short learning curve
(c) Inexpensive
(d) Allows dynamic studies to be done
30.
Which of the following statements regarding lymphoedema following breast cancer treatment
are correct?

1. Incidence has decreased due to rarely combined therapy of axillary LN dissection and
radiotherapy
2. Precipitating cause like LN metastasis is a major determinant
3. The condition is often painful
4. Oedematous limb is susceptible to bacterial infection

Select the correct answer using the code given below:

(a) 1, 2 and 3
(b) 2, 3 and 4
(c) 1, 3 and 4
(d) 1, 2 and 4
31.
Which one of the following is NOT a risk factor for development of venous thrombosis in
surgical patients?

(a) Age > 60 years
(b) Pregnancy
(c) Obesity (BMI > 30 kg/m
2
)
(d) Diabetes ( HbA1c > 7.5%)
32.
Which one of the following is NOT a complication of massive blood transfusion?

(a) Coagulopathy
(b) Hypocalcaemia
(c) Hyperthermia
(d) Hyperkalemia
33.
Stage III ?Pressure sore? is full thickness skin loss extending:

(a) into subcutaneous tissue but not through fascia
(b) through subcutaneous tissue into fascia
(c) through subcutaneous tissue into fascia and muscles
(d) through subcutaneous tissue into fascia, muscles and bone



34.
During subclavian vein puncture in a surgical ward suddenly a patient developed severe
breathlessness. On auscultation breath sound was absent and the ipsilateral chest was
tympanitic on percussion. The probable diagnosis is:

(a) Iatrogenic pneumothorax
(b) Introgenic hemothorax
(c) Spontaneous pneumothorax
(d) Tension pneumothorax
35.
Kohler?s disease is avascular necrosis of :

(a) Lunate
(b) Capitellum of humerus
(c) First metatarsal head
(d) Navicular
36.
Which one of the following is NOT true of Pyoderma gangrenosum?

(a) It is characterized by cutaneous ulceration with purple undermined edges
(b) It is often secondary to heightened immunological reactivity from another disease
process
(c) Cultures often show Gram positive Staphylococci
(d) Lesions generally respond to steroids
37.
Which of the following is NOT a tissue repair surgery for inguinal hernia repair?

(a) Bassini?s repair
(b) Shouldice repair
(c) Stoppa?s repair
(d) Desarda repair
38.
Which one of the following type of meshes is recommended for intraperitoneal use in
abdominal wall hernia?
(a) Light weight, porous meshes
(b) Heavy weight, porous meshes
(c) Absorbable meshes
(d) Tissue separating meshes
39.
Which one of the following is NOT correct regarding MEN-1 syndrome?

(a) It involves parathyroid glands
(b) It involves pancreas
(c) It involves pituitary gland
(d) It involves pineal gland

40.
Which one of the following is NOT an electronic information site in surgery?

(a) Pubmed
(b) Embase
(c) Cochrane library
(d) National medical library
41.
Which one of the following statements regarding pre-conceptional counseling is NOT
correct?

(a) It is needed only in selected complicated pregnancies
(b) It helps in early detection of risk factors
(c) It helps in reducing maternal morbidity and mortality
(d) It is a part of preventive medicine
42.
Consider the following statements regarding Non Stress Test (NST):

1. Reactive NST indicates a healthy fetus
2. NST is an observed association of fetal breathing with fetal movements
3. NST has a low false negative rate (< 1%) but high false positive rate (>50%)
4. Testing should be started at 20 weeks

Which of the statement(s) given above is/are correct?

(a) 1 and 3
(b) 2 only
(c) 3 only
(d) 1 and 4
43.
Which one of the following is a protective factor for endometrial hyperplasia?

(a) Diabetes
(b) Tamoxifen therapy
(c) Multiparity
(d) Delayed menopause
44.
A woman who is not breast feeding her newborn child is advised to use a contraceptive
method by:

(a) 3
rd
postpartum week
(b) 6
th
postpartum week
(c) 3
rd
postpartum month
(d) 6
th
postpartum month





45.
Pearl index for contraceptive effectiveness is calculated in terms of which of the following?

1. Pregnancy rate
2. Abortion rate
3. Hundred woman years
4. Thousand woman years

Select the correct answer using the code given below:

(a) 1 only
(b) 2 and 3
(c) 1, 2 and 4
(d) 1 and 3
46.
Indications for removal of IUDs are all EXCEPT:

(a) Perforation of uterus
(b) Cyclical menstrual bleeding
(c) Flaring up of salpingitis
(d) Pregnancy with IUD
47.
Contraindications for insertion of IUDs are all EXCEPT:

(a) Suspected pregnancy
(b) Trophoblastic disease
(c) Severe dysmenorrhea
(d) During cesarean section
48.
Which one of the following is NOT a contraindication for use of Mini pill?

(a) Pregnancy
(b) Breast feeding
(c) Thromboembolic disease
(d) History of breast cancer
49.
Which one of the following is the most commonly used surgical method/technique of female
sterilization as recommended by Government of India?

(a) Uchida technique
(b) Irving method
(c) Pomeroy?s method
(d) Madlener technique







50.
Which of the following is/are required for a registered medical practitioner to qualify for
performing Medical Termination of Pregnancy (MTP), as per revised rules of MTP Act?

1. Certified for assisting at least 15 MTP in an authorized centre
2. Diploma or degree in Obstetrics and Gynaecology
3. House surgeon training for 3 months in Obstetrics and Gynaecology
4. Certified training for 6 months in laparoscopic surgeries

Select the correct answer using the code given below:
(a) 1 only
(b) 2 only
(c) 1, 2 and 3
(d) 1, 2 and 4
51.
Which one of the following is NOT a support of uterus, preventing its descent?

(a) Endopelvic fascia
(b) Mackenrodt?s ligament
(c) Inguinal ligament
(d) Pubocervical ligament
52.
As per ICMR guidelines, which one of the following statements is true regarding effects of
COVID-19 on fetus according to current evidence?

(a) There is increased risk of early pregnancy loss
(b) COVID-19 virus is not teratogenic
(c) COVID-19 virus infection is an indication of MTP
(d) There is increased risk of fetal growth restriction
53.
As per ICMR guidelines, which one of the following statements is true regarding COVID-19
infection in pregnancy?

(a) Covid-19 pneumonia in pregnancy is more severe with poor recovery
(b) Pregnant women with heart disease are at higher risk
(c) Vaginal secretions always test positive for COVID-19 in pregnancy
(d) COVID-19 virus is secreted in breast milk
54.
Which one of the following is NOT a method of management of Deep Transverse Arrest with
the living fetus?

(a) Caesarean section
(b) Delivery by ventouse
(c) Delivery by application of forceps to the unrotated head
(d) Manual rotation and application of forceps





55.
Successful version of breech presentation is likely in case all of the following EXCEPT:

(a) Breech with extended legs
(b) Complete breech with sacroanterior position
(c) Non engaged breech
(d) Adequate amniotic fluid
56.
Implantation of a fertilised ovum occurs on which day following fertilisation?

(a) Day 6
(b) Day 10
(c) Day 14
(d) Day 20
57.
During total abdominal hysterectomy the ureter is likely to undergo injury or ligation during
the following steps EXCEPT:

(a) During division and ligation of the round ligaments
(b) During division and ligation of infundibulopelvic ligaments
(c) During division and ligation of mackenrodt?s and uterosacral ligaments
(d) At the vaginal angles while incising the vagina to remove the cervix with the
uterus
58.
The net effect of antenatal care has been the following EXCEPT:

(a) Reduction in maternal mortality
(b) Reduction in perinatal mortality
(c) Reduction in the incidence of institutional delivery
(d) Reduction in maternal morbidity
59.
Which one of the following is NOT a component of active phase in the partograph?

(a) Acceleration phase
(b) Phase of maximum slope
(c) Phase of deceleration
(d) Phase of expulsion
60.
From medicolegal point of view which one of the following is NOT a sign of previous child
birth?

(a) Perineum is lax and there is evidence of scarring
(b) Introitus is gaping and there is presence of carunculae myrtiformis
(c) Abdomen is lax and loose with striae and linea alba
(d) Conical cervix with round external os





61.
The components of partograph are all EXCEPT:

(a) Time
(b) Fetal heart rate
(c) Maternal respiratory rate
(d) Maternal urine analysis
62.
Which of the following information are provided by partograph?

1. Colour of liquor
2. Uterine contractions with duration and frequency
3. Dilatation of cervix

Select the correct answer using the code given below:

(a) 1 and 2 only
(b) 2 and 3 only
(c) 1 and 3 only
(d) 1, 2 and 3
63.
Intraoperative recognition of ureter is by which of the following features?

1. Transparent tubular appearance
2. Pale glistening appearance
3. Longitudinal vessels on surface
4. Circumferential vessels on surface

Select the correct answer using the code given below:

(a) 1 and 3
(b) 2 and 4
(c) 2 and 3
(d) 1 and 4
64.
Hysterosalpingography (HSG) is least helpful in detecting which of the following?

(a) Tubal patency
(b) Pelvic adhesions
(c) Asherman syndrome
(d) Congenital uterine anomaly









65.
Which of the following are characteristics of Trichomonas vaginitis?

1. Presence of greenish frothy discharge
2. Vaginal pH > 4.5
3. Presence of clue cells in microscopic examination
4. Strawberry spots on the vaginal mucosa

Select the correct answer using the code given below:

(a) 1, 2 and 3
(b) 1, 2 and 4
(c) 2, 3 and 4
(d) 1, 3 and 4
66.
Tumor marker of epithelial ovarian carcinoma is:

(a) Ca.125
(b) Alpha feto protein
(c) Beta HCG
(d) LDH
67.
The most common site of cervical cancer is:

(a) Endocervix
(b) Ectocervix
(c) Transformation zone
(d) Isthmus
68.
The placenta synthesizes all EXCEPT:

(a) Oestriol
(b) Corticotrophin releasing hormone
(c) PAPP-A(Pregnancy Associated Plasma Protein A)
(d) Dehydroepiandrosterone
69.
Withdrawal bleeding following administration of progesterone in a case of secondary
amenorrhea indicates all EXCEPT:

(a) Absence of pregnancy
(b) Production endogenous estrogen
(c) Endometrium is responsive to estrogen
(d) Defect in pituitary gland
70.
Monilial vaginitis is commonly associated with all EXCEPT:

(a) Prolonged antibiotic therapy
(b) Diabetes Mellitus
(c) Treatment of malaria with chloroquine
(d) Pregnancy
71.
Which one of the following is NOT a risk factor for the development of placenta previa?

(a) Maternal age
(b) Smoking
(c) Previous caesarean section
(d) Maternal anaemia
72.
Common clinical presentations of moderate to severe abruption are all EXCEPT:

(a) Uterine tenderness
(b) Fetal distress
(c) Unexplained pre term labour
(d) Prolonged labour
73.
Common trisomies resulting in spontaneous abortion are all EXCEPT:

(a) Trisomy 21
(b) Trisomy 18
(c) Trisomy 16
(d) Trisomy 1
74.
The initial prevention strategy for antiphospholipid syndrome will be:

1. Steroids
2. Heparin
3. Low dose aspirin
4. Progesterone support

Which of the above is/are correct?

(a) 2 and 3
(b) 3 and 4
(c) 3 only
(d) 1 and 4
75.
Diagnostic criteria for PCOD are:

1. Oligo/amenorrohoea
2. Hyperandrogenism
3. Polycystic ovaries on ultrasound

Which of the above are correct?

(a) 1 and 2 only
(b) 2 and 3 only
(c) 1 and 3 only
(d) 1, 2 and 3


76.
Which of the following symptoms can be associated with pelvic organ prolapse?

1. Difficulty in passing urine
2. Incomplete evacuation of urine
3. Urgency and frequency

Select the correct answer using the code given below:

(a) 1 and 2 only
(b) 2 and 3 only
(c) 1 and 3 only
(d) 1, 2 and 3
77.
A 30 year old lady, P
2
L
2
presents with painful unilateral swelling in vulva for 3 days. Which
of the following statements are true regarding the above case?

1. Bartholin?s abscess may be the likely diagnosis
2. It is to be managed by marsupialisation
3. Gonococcus is the most common pathogenic organism

Select the correct answer using the code given below:
(a) 1 and 3 only
(b) 3 only
(c) 1 and 2 only
(d) 1, 2 and 3
78.
Which one of the following is NOT a sign of separation of placenta?

(a) Uterus becomes globular, firm and ballotable
(b) The fundal height reduces further
(c) Slight bulging in the suprapubic region
(d) Apparent lengthenic of the cord with slight gush of vaginal bleeding
79.
Consider the following regarding examination of a rape victim:

1. Emergency pill is provided
2. Internal examination must be performed
3. HIV testing is done

Which of the above statements is/are correct?

(a) 1 and 3 only
(b) 2 only
(c) 1, 2 and 3
(d) 3 only




80.
Consider the following cardinal movements of mechanism of normal labor:
1. Engagement
2. Internal rotation
3. Flexion
4. Restitution
5. Crowning
6. External rotation

What is the correct sequence of movements in labor in occipito-lateral position?

(a) 1, 2, 3, 4, 5 and 6
(b) 1, 3, 2, 5, 4 and 6
(c) 2, 1, 3, 4, 5 and 6
(d) 3, 1, 2, 4, 6 and 5
81.
Which of the following represent the properties of an ideal disinfectant?

1. It is broad spectrum
2. It is fast acting
3. It is non-toxic

Select the correct answer using the code given below:
(a) 1 and 2 only
(b) 2 and 3 only
(c) 1 and 3 only
(d) 1, 2 and 3
82.
Which of the following measures can help reduce the risk of systemic hypertension?

1. Reduction in dietary intake of common salt
2. Controlling weight for age
3. Increasing potassium rich foods in the diet

Select the correct answer using the code given below:

(a) 1 and 2 only
(b) 2 and 3 only
(c) 1 and 3 only
(d) 1, 2 and 3








83.
Keeping biological determinants in perspective, consider the following statements:

1. Presence of a normal karyotype is the first requisite for human health
2. Genetic screening can play an important role in prevention of wide spectrum of
diseases
3. If an individual is allowed to live in healthy relationship with the environment, the
person?s genetic potentialities can transform into phenotypic realities

Which of the above statements is/are correct?

(a) 1 and 2 only
(b) 1 and 3 only
(c) 1, 2 and 3
(d) 2 and 3 only
84.
NITI Aayog has the following roles EXCEPT:

(a) It provides critical directional and strategic input in the development process
(b) It provides relevant technical advice with focus on technology upgradation
(c) It focuses on capacity building
(d) It focuses on the development of ?Referral Service Complex?
85.
Poor hand hygiene of a mess worker in a university college mess led to Hepatitis A cases in
the hostel inmates. What type of epidemic will this exposure present with?

1. Propagated
2. Common source-continuous exposure
3. Common source-point exposure

Select the correct answer using the code given below:

(a) 1 and 2
(b) 1 and 3
(c) 1 only
(d) 2 only












86.
What is the specificity of sputum microscopy in detection of Pulmonary Tuberculosis (PTB)
as per the information given below?

PTB Total
Sputum microscopy Present Absent
Positive 270 20 290
Negative 30 180 210
Total 300 200 500

(a) 10 %
(b) 36 %
(c) 90 %
(d) 94 %
87.
In a cohort of 500 women attending antenatal clinic, 70 % had ultrasonography (USG). This
cohort was followed up at delivery. Of the women who had USG, 70 delivered low birth
weight (LBW) babies; whereas of the women, who did not undergo USG, 50 delivered LBW
babies. The incidence of LBW babies among women who had USG is:

(a) 10 %
(b) 15 %
(c) 20 %
(d) 25 %
88.
Major source of vitamin K
1
is:

(a) Fresh dark green vegetables
(b) Exposure of body to sunlight
(c) Citrus fruits
(d) Foods rich in polyunsaturated fatty acids
89.
Which one of the following is NOT a function of Epidemiology?

(a) To study historically the rise and fall of disease in the population
(b) Searching for the causes and risk factor
(c) Identifying syndromes
(d) Making clinical diagnosis
90.
Which one of the following is NOT a contagious disease?

(a) Malaria
(b) Scabies
(c) Trachoma
(d) Leprosy


91.
In primary immune response, how much more antigenic dose is required to induce IgG
antibodies as against the induction of IgM antibodies?

(a) 10 times more
(b) 25 times more
(c) 50 times more
(d) 100 times more
92.
Pentavalent vaccine provides protection against which of the following diseases?

(a) Diphtheria, Pertussis, Tuberculosis, Measles and Hepatitis B
(b) Diphtheria, Pertussis, Measles, Hepatitis B and Hib
(c) Diphtheria, Pertussis, Tetanus, Hepatitis B and Rubella
(d) Diphtheria, Pertussis, Tetanus, Hepatitis B and Hib
93.
Consider the following data for a country:
Population in 0-14 years of age ? 391,558,367
Population between 15?64 years of age ? 856,076,200
Population above 65 years of age ? 71,943,390
What shall be the dependency ratio of this country?

(a) 42.4 %
(b) 54.1 %
(c) 66.2 %
(d) 78.6 %
94.
Which one of the following statements regarding Rabies Immunoglobulin is NOT true?

(a) It should be administered only once as soon as possible after the initiation of post
exposure prophylaxis
(b) It should be administered all into or around the wound sites
(c) There is no scientific ground for performing a skin test prior to administering equine
immunoglobulin
(d) It can be administered till 15 days after the first dose of anti-rabies vaccine
95.
All of the following are true about Bedaquiline (BDQ) EXCEPT:

(a) It specifically targets mycobacterial ATP synthase
(b) It is a bacteriostatic drug
(c) It has extended half life
(d) It has high volume of tissue distribution
96.
Visual inspection based screening test with 5 % acetic acid is used for the screening of which
one of the following cancers?

(a) Lung cancer
(b) Cervix cancer
(c) Oral cancer
(d) Breast cancer
97.
Predictive accuracy of a screening test depends on the following EXCEPT:

(a) Disease prevalence
(b) Disease incidence
(c) Sensitivity of screening test
(d) Specificity of screening test
98.
How much of Zinc supplement is recommended by WHO and UNICEF for infants less than 6
months of age after an episode of acute diarrhoea?

(a) 20 mg per day for 10?14 days
(b) 10 mg per day for 10?14 days
(c) 5 mg per day for 7 days
(d) 6 mg per day for 7 days
99.
Which one of the following statements regarding sequential administration of Inactivated
Polio Vaccine (IPV) and Oral Polio Vaccine (OPV) is NOT correct?

(a) It will be cost effective in developing countries for Polio prevention
(b) The combined schedules of IPV and OPV appear to reduce or prevent Vaccine
Associated Paralytic Polio (VAPP)
(c) Intestinal mucosal immunity is lost due to IPV administration
(d) IPV and OPV together may optimize both the humoral and mucosal immunogenicity
of Polio vaccine
100.
What is the recommended dose regimen of Vitamin A for the treatment of early stages of
Xerophthalmia?

(a) Single massive dose of 2 lac International Units (IU)
(b) 2 lac IU on two successive days
(c) 2 doses of 1 lac IU in two successive days
(d) 2 doses of 1 lac IU at a gap of one week
101.
Which is the most specific causative agent of Rabies?

(a) Lyssavirus serotype 1
(b) Lyssavirus serotype 2
(c) Lyssavirus serotype 3
(d) Lyssavirus serotype 4
102.
Which one of the following is the antibiotic of choice for the prevention of Rheumatic heart
disease?

(a) Benzathine Benzyl Penicillin
(b) Procaine Penicillin
(c) Doxycycline
(d) Ciprofloxacin


103.
All of the following are global targets for WHO Global Action Plan (2013?2020) for
Prevention and Control of NCDs, EXCEPT:

(a) A 10 % relative reduction in risk of premature mortality from cardiovascular diseases,
cancer, diabetes and chronic respiratory diseases
(b) A 10 % relative reduction in prevalence of insufficient physical activity
(c) A 10 % relative reduction in mean population intake of salt/sodium
(d) At least 10 % relative reduction in the harmful use of alcohol
104.
Under the Employees State Insurance (ESI) Scheme, extended sickness benefit is provided in
which of the following infectious diseases?

1. Tuberculosis
2. Leprosy
3. Chronic empyema

Select the correct answer using the code given below:

(a) 1 and 2 only
(b) 2 and 3 only
(c) 1 and 3 only
(d) 1, 2 and 3

105.
In the context of NCD prevention and control in India, the extent of relative reduction in
household use of solid fuels as a primary source of energy for cooking by 2025 is targeted at:

(a) 30 %
(b) 40 %
(c) 50 %
(d) 60 %
106.
Consider the following criteria which may indicate elimination of lymphatic filariasis in a
community:

1. When lymphatic filariasis ceases to be a public health problem in the community
2. When the number of microfilaria carriers declines to 1.5 % within the community
3. When children born in the community after the initiation of elimination programme
are free from circulating antigenaemia

Which of the criteria stated above hold true?

(a) 1 and 2 only
(b) 2 and 3 only
(c) 1 and 3 only
(d) 1, 2 and 3



FirstRanker.com - FirstRanker's Choice
Combined Medical Services Examination-2020
Paper-II

1.
Indications for fasciotomy in compartment syndrome include all EXCEPT:

(a) Distal sensory disturbance
(b) Compartment pressure > 30 mm Hg
(c) Pain on passive movement of affected muscles
(d) Palpable distal pulses
2.
Which one of the following statements is NOT correct regarding Necrotising Soft Tissue
infections?

(a) Crepitus, skin blistering and focal skin gangrene are typical presenting features
(b) They are monomicrobial in nature
(c) Treatment consists of wide local excision and appropriate antibiotics
(d) Tissue biopsy is required for culture and diagnosis
3.
Which type of surgery is laparoscopic cholecystectomy classified as?

(a) Clean
(b) Clean contaminated
(c) Contaminated
(d) Dirty
4.
A 22-year female has presented with a history of malaise, cough, alternating constipation and
diarrhoea with intermittent abdominal pain for last 6 months. She also complains of
abdominal distension for last 2 days. On examination her abdomen has a doughy feel along
with an ill defined mass over the right lower quadrant. She is most likely suffering from:

(a) Appendicular lump
(b) Ileocaecal tuberculosis
(c) Carcinoma caecum
(d) Ovarian mass
5.
Consider the following statements regarding needle stick injuries:

1. Injured part should be washed under running water
2. Dominant index finger is the commonest site for needle stick injury
3. All needle stick injuries should be reported
4. Hepatitis/HIV testing should be done after needle stick injury

Which of the statements given above are correct?

(a) 1, 2 and 4
(b) 1, 2 and 3
(c) 1, 3 and 4
(d) 2, 3 and 4




6.
Consider the following statements regarding claudication:

1. It is a marker for silent coronary disease
2. Structured exercise program ( 2 hours per week for 3 months) leads to
improvement in symptoms
3. Diabetes mellitus increases the risk and severity of claudication
4. Beta blockers may exacerbate claudication

Which of the above statements are correct?

(a) 1 and 2 only
(b) 1, 3 and 4 only
(c) 2, 3 and 4 only
(d) 1, 2, 3 and 4
7.
A 50-year old lady underwent uneventful bariatric surgery for morbid obesity. On the third
post operative day, she develops breathless and pulmonary embolism is suspected. The next
investigation to confirm the diagnosis will be:

(a) Echocardiography
(b) Duplex venography
(c) CT pulmonary angiography
(d) MR angiography
8.
Medical management of thyrotoxic crisis includes all of the following EXCEPT:

(a) IV fluids
(b) IV propanolol
(c) IV hydrocortisone
(d) IV antibiotics
9.
A patient operated for a parotid gland tumour developed symptoms of sweating and
erytherma (flushing) over the region of surgical excision while eating. The probable
diagnosis is:

(a) Parotid gland fistula
(b) Sialadenitis
(c) Chronic wound infection
(d) Frey?s syndrome
10.
First line hormone therapy for post-menopausal woman with metastatic carcinoma breast is:

(a) Tamoxifen
(b) Ovarian suppression by surgery
(c) Antiprogestins
(d) Anastrazole



11.
All of the following are major subtypes of breast cancer based on Gene array analysis
EXCEPT:

(a) Luminal A and Luminal B
(b) Triple negative
(c) Her-2 receptor positive
(d) Oestrogen receptor positive
12.
All of the following are sequelae of peptic ulcer surgery EXCEPT:

(a) Bilious vomiting
(b) Dumping syndrome
(c) Diarrhoea
(d) Increased appetite
13.
The Child-Turcotte-Pugh (CTP) score for quantifying the severity of chronic liver disease
includes all variables EXCEPT:

(a) Serum bilirubin
(b) Serum albumin
(c) Serum creatinine
(d) INR (International Normalised Ratio)
14.
?Chain of Lakes? appearance due to sacculation with intervening short strictures of pancreatic
duct is seen on:

(a) ERCP
(b) CECT abdomen
(c) Plain X-ray abdomen
(d) Ultrasonography
15.
Which one of the following statements is NOT correct regarding Pyogenic Liver Abscess?

(a) Anorexia, fever, malaise and right upper quadrant abdominal discomfort are the
most common presenting features
(b) It is more common in elderly, diabetics and immunocompromised patients
(c) Treatment is with oral antibiotics alone
(d) Streptococcus milleri and escherichia coli are the most common causative
organisms
16.
?Swiss cheese defects? are seen during laparoscopic repair of:

(a) Ventral hernia
(b) Inguinal hernia
(c) Obturator hernia
(d) Femoral hernia



17.
Which of the following are correct regarding splenic artery aneurysm?

1. Main arterial trunk is the common site
2. Palpable thrill can be felt
3. It is symptomless unless it ruptures

Select the correct answer using the code given below:

(a) 1 and 2 only
(b) 2 and 3 only
(c) 1 and 3 only
(d) 1, 2 and 3
18.
Valentino?s syndrome is:

(a) Pain on per-vaginal examination in pelvic abscess
(b) Pain over left shoulder in left hypochondriac collection
(c) Pain over left groin in perirenal collection
(d) Pain in right iliac fossa in perforated peptic ulcer
19.
Spontaneous bacterial peritonitis occurs due to:

(a) duodenal stump blowout
(b) peptic ulcer perforation
(c) acute bacterial infection of ascites
(d) infection via fallopian tubes
20.
Structure not forming boundaries of the ?Triangle of doom? seen during laparoscopic
inguinal hernia surgery dissection is:

(a) Vas deferens
(b) Inferior epigastric artery
(c) Spermatic cord vessels
(d) Peritoneum
21.
The term mid-line shift is associated with:

(a) Head injury
(b) Chest injury
(c) Abdominal injury
(d) Limb injury
22.
Which one of the following cranial nerves does NOT supply to the external ear?

(a) Cranial nerve V
(b) Cranial nerve VI
(c) Cranial nerve VII
(d) Cranial nerve IX

23.
Left Internal Mammary Artery (LIMA) has become the conduit of choice for Left Anterior
Descending (LAD) artery during coronary artery bypass grafting because:

(a) Long term patency rates are more than 98%
(b) It is close to LAD
(c) Atherosclerosis is never seen in this vessel
(d) It is very easy to harvest
24.
A 50-year old male with significant smoking history presented in the surgical emergency
with sudden severe breathlessness. Chest X-ray shows right sided Pneumothorax. The
appropriate management requires:

(a) Aspiration of air with 16-18 G cannula
(b) Right chest drain of size 8-14 Fr
(c) Oxygen by face mask
(d) Mechanical ventilation
25.
Which one of the following statements regarding Felon is NOT correct?

(a) There is infection of the finger tip between specialised fibrous septa
(b) It is a painless condition
(c) Incision and drainage is the treatment of choice
(d) It is common in diabetics
26.
The most common site for osteosarcoma is:

(a) Proximal femur
(b) Distal femur
(c) Proximal humerus
(d) Distal humerus
27.
Rapid Sequence Induction is indicated in:

(a) Emergency surgery for intestinal obstruction
(b) Elective open hernia surgery
(c) Cardiopulmonary bypass surgery
(d) Elective laparoscopic surgery

28.
Which one of the following statements about Compartment Syndrome is NOT correct?

(a) It is commonest in a closed fracture
(b) Pain is on active movement but not on passive movement of muscles
(c) Fasciotomy is the treatment of choice
(d) Volkmann?s Ischaemic contractive is a late complication




29.
Which one of the following is NOT the strength of ultrasound as a diagnostic modality?

(a) No radiation
(b) Short learning curve
(c) Inexpensive
(d) Allows dynamic studies to be done
30.
Which of the following statements regarding lymphoedema following breast cancer treatment
are correct?

1. Incidence has decreased due to rarely combined therapy of axillary LN dissection and
radiotherapy
2. Precipitating cause like LN metastasis is a major determinant
3. The condition is often painful
4. Oedematous limb is susceptible to bacterial infection

Select the correct answer using the code given below:

(a) 1, 2 and 3
(b) 2, 3 and 4
(c) 1, 3 and 4
(d) 1, 2 and 4
31.
Which one of the following is NOT a risk factor for development of venous thrombosis in
surgical patients?

(a) Age > 60 years
(b) Pregnancy
(c) Obesity (BMI > 30 kg/m
2
)
(d) Diabetes ( HbA1c > 7.5%)
32.
Which one of the following is NOT a complication of massive blood transfusion?

(a) Coagulopathy
(b) Hypocalcaemia
(c) Hyperthermia
(d) Hyperkalemia
33.
Stage III ?Pressure sore? is full thickness skin loss extending:

(a) into subcutaneous tissue but not through fascia
(b) through subcutaneous tissue into fascia
(c) through subcutaneous tissue into fascia and muscles
(d) through subcutaneous tissue into fascia, muscles and bone



34.
During subclavian vein puncture in a surgical ward suddenly a patient developed severe
breathlessness. On auscultation breath sound was absent and the ipsilateral chest was
tympanitic on percussion. The probable diagnosis is:

(a) Iatrogenic pneumothorax
(b) Introgenic hemothorax
(c) Spontaneous pneumothorax
(d) Tension pneumothorax
35.
Kohler?s disease is avascular necrosis of :

(a) Lunate
(b) Capitellum of humerus
(c) First metatarsal head
(d) Navicular
36.
Which one of the following is NOT true of Pyoderma gangrenosum?

(a) It is characterized by cutaneous ulceration with purple undermined edges
(b) It is often secondary to heightened immunological reactivity from another disease
process
(c) Cultures often show Gram positive Staphylococci
(d) Lesions generally respond to steroids
37.
Which of the following is NOT a tissue repair surgery for inguinal hernia repair?

(a) Bassini?s repair
(b) Shouldice repair
(c) Stoppa?s repair
(d) Desarda repair
38.
Which one of the following type of meshes is recommended for intraperitoneal use in
abdominal wall hernia?
(a) Light weight, porous meshes
(b) Heavy weight, porous meshes
(c) Absorbable meshes
(d) Tissue separating meshes
39.
Which one of the following is NOT correct regarding MEN-1 syndrome?

(a) It involves parathyroid glands
(b) It involves pancreas
(c) It involves pituitary gland
(d) It involves pineal gland

40.
Which one of the following is NOT an electronic information site in surgery?

(a) Pubmed
(b) Embase
(c) Cochrane library
(d) National medical library
41.
Which one of the following statements regarding pre-conceptional counseling is NOT
correct?

(a) It is needed only in selected complicated pregnancies
(b) It helps in early detection of risk factors
(c) It helps in reducing maternal morbidity and mortality
(d) It is a part of preventive medicine
42.
Consider the following statements regarding Non Stress Test (NST):

1. Reactive NST indicates a healthy fetus
2. NST is an observed association of fetal breathing with fetal movements
3. NST has a low false negative rate (< 1%) but high false positive rate (>50%)
4. Testing should be started at 20 weeks

Which of the statement(s) given above is/are correct?

(a) 1 and 3
(b) 2 only
(c) 3 only
(d) 1 and 4
43.
Which one of the following is a protective factor for endometrial hyperplasia?

(a) Diabetes
(b) Tamoxifen therapy
(c) Multiparity
(d) Delayed menopause
44.
A woman who is not breast feeding her newborn child is advised to use a contraceptive
method by:

(a) 3
rd
postpartum week
(b) 6
th
postpartum week
(c) 3
rd
postpartum month
(d) 6
th
postpartum month





45.
Pearl index for contraceptive effectiveness is calculated in terms of which of the following?

1. Pregnancy rate
2. Abortion rate
3. Hundred woman years
4. Thousand woman years

Select the correct answer using the code given below:

(a) 1 only
(b) 2 and 3
(c) 1, 2 and 4
(d) 1 and 3
46.
Indications for removal of IUDs are all EXCEPT:

(a) Perforation of uterus
(b) Cyclical menstrual bleeding
(c) Flaring up of salpingitis
(d) Pregnancy with IUD
47.
Contraindications for insertion of IUDs are all EXCEPT:

(a) Suspected pregnancy
(b) Trophoblastic disease
(c) Severe dysmenorrhea
(d) During cesarean section
48.
Which one of the following is NOT a contraindication for use of Mini pill?

(a) Pregnancy
(b) Breast feeding
(c) Thromboembolic disease
(d) History of breast cancer
49.
Which one of the following is the most commonly used surgical method/technique of female
sterilization as recommended by Government of India?

(a) Uchida technique
(b) Irving method
(c) Pomeroy?s method
(d) Madlener technique







50.
Which of the following is/are required for a registered medical practitioner to qualify for
performing Medical Termination of Pregnancy (MTP), as per revised rules of MTP Act?

1. Certified for assisting at least 15 MTP in an authorized centre
2. Diploma or degree in Obstetrics and Gynaecology
3. House surgeon training for 3 months in Obstetrics and Gynaecology
4. Certified training for 6 months in laparoscopic surgeries

Select the correct answer using the code given below:
(a) 1 only
(b) 2 only
(c) 1, 2 and 3
(d) 1, 2 and 4
51.
Which one of the following is NOT a support of uterus, preventing its descent?

(a) Endopelvic fascia
(b) Mackenrodt?s ligament
(c) Inguinal ligament
(d) Pubocervical ligament
52.
As per ICMR guidelines, which one of the following statements is true regarding effects of
COVID-19 on fetus according to current evidence?

(a) There is increased risk of early pregnancy loss
(b) COVID-19 virus is not teratogenic
(c) COVID-19 virus infection is an indication of MTP
(d) There is increased risk of fetal growth restriction
53.
As per ICMR guidelines, which one of the following statements is true regarding COVID-19
infection in pregnancy?

(a) Covid-19 pneumonia in pregnancy is more severe with poor recovery
(b) Pregnant women with heart disease are at higher risk
(c) Vaginal secretions always test positive for COVID-19 in pregnancy
(d) COVID-19 virus is secreted in breast milk
54.
Which one of the following is NOT a method of management of Deep Transverse Arrest with
the living fetus?

(a) Caesarean section
(b) Delivery by ventouse
(c) Delivery by application of forceps to the unrotated head
(d) Manual rotation and application of forceps





55.
Successful version of breech presentation is likely in case all of the following EXCEPT:

(a) Breech with extended legs
(b) Complete breech with sacroanterior position
(c) Non engaged breech
(d) Adequate amniotic fluid
56.
Implantation of a fertilised ovum occurs on which day following fertilisation?

(a) Day 6
(b) Day 10
(c) Day 14
(d) Day 20
57.
During total abdominal hysterectomy the ureter is likely to undergo injury or ligation during
the following steps EXCEPT:

(a) During division and ligation of the round ligaments
(b) During division and ligation of infundibulopelvic ligaments
(c) During division and ligation of mackenrodt?s and uterosacral ligaments
(d) At the vaginal angles while incising the vagina to remove the cervix with the
uterus
58.
The net effect of antenatal care has been the following EXCEPT:

(a) Reduction in maternal mortality
(b) Reduction in perinatal mortality
(c) Reduction in the incidence of institutional delivery
(d) Reduction in maternal morbidity
59.
Which one of the following is NOT a component of active phase in the partograph?

(a) Acceleration phase
(b) Phase of maximum slope
(c) Phase of deceleration
(d) Phase of expulsion
60.
From medicolegal point of view which one of the following is NOT a sign of previous child
birth?

(a) Perineum is lax and there is evidence of scarring
(b) Introitus is gaping and there is presence of carunculae myrtiformis
(c) Abdomen is lax and loose with striae and linea alba
(d) Conical cervix with round external os





61.
The components of partograph are all EXCEPT:

(a) Time
(b) Fetal heart rate
(c) Maternal respiratory rate
(d) Maternal urine analysis
62.
Which of the following information are provided by partograph?

1. Colour of liquor
2. Uterine contractions with duration and frequency
3. Dilatation of cervix

Select the correct answer using the code given below:

(a) 1 and 2 only
(b) 2 and 3 only
(c) 1 and 3 only
(d) 1, 2 and 3
63.
Intraoperative recognition of ureter is by which of the following features?

1. Transparent tubular appearance
2. Pale glistening appearance
3. Longitudinal vessels on surface
4. Circumferential vessels on surface

Select the correct answer using the code given below:

(a) 1 and 3
(b) 2 and 4
(c) 2 and 3
(d) 1 and 4
64.
Hysterosalpingography (HSG) is least helpful in detecting which of the following?

(a) Tubal patency
(b) Pelvic adhesions
(c) Asherman syndrome
(d) Congenital uterine anomaly









65.
Which of the following are characteristics of Trichomonas vaginitis?

1. Presence of greenish frothy discharge
2. Vaginal pH > 4.5
3. Presence of clue cells in microscopic examination
4. Strawberry spots on the vaginal mucosa

Select the correct answer using the code given below:

(a) 1, 2 and 3
(b) 1, 2 and 4
(c) 2, 3 and 4
(d) 1, 3 and 4
66.
Tumor marker of epithelial ovarian carcinoma is:

(a) Ca.125
(b) Alpha feto protein
(c) Beta HCG
(d) LDH
67.
The most common site of cervical cancer is:

(a) Endocervix
(b) Ectocervix
(c) Transformation zone
(d) Isthmus
68.
The placenta synthesizes all EXCEPT:

(a) Oestriol
(b) Corticotrophin releasing hormone
(c) PAPP-A(Pregnancy Associated Plasma Protein A)
(d) Dehydroepiandrosterone
69.
Withdrawal bleeding following administration of progesterone in a case of secondary
amenorrhea indicates all EXCEPT:

(a) Absence of pregnancy
(b) Production endogenous estrogen
(c) Endometrium is responsive to estrogen
(d) Defect in pituitary gland
70.
Monilial vaginitis is commonly associated with all EXCEPT:

(a) Prolonged antibiotic therapy
(b) Diabetes Mellitus
(c) Treatment of malaria with chloroquine
(d) Pregnancy
71.
Which one of the following is NOT a risk factor for the development of placenta previa?

(a) Maternal age
(b) Smoking
(c) Previous caesarean section
(d) Maternal anaemia
72.
Common clinical presentations of moderate to severe abruption are all EXCEPT:

(a) Uterine tenderness
(b) Fetal distress
(c) Unexplained pre term labour
(d) Prolonged labour
73.
Common trisomies resulting in spontaneous abortion are all EXCEPT:

(a) Trisomy 21
(b) Trisomy 18
(c) Trisomy 16
(d) Trisomy 1
74.
The initial prevention strategy for antiphospholipid syndrome will be:

1. Steroids
2. Heparin
3. Low dose aspirin
4. Progesterone support

Which of the above is/are correct?

(a) 2 and 3
(b) 3 and 4
(c) 3 only
(d) 1 and 4
75.
Diagnostic criteria for PCOD are:

1. Oligo/amenorrohoea
2. Hyperandrogenism
3. Polycystic ovaries on ultrasound

Which of the above are correct?

(a) 1 and 2 only
(b) 2 and 3 only
(c) 1 and 3 only
(d) 1, 2 and 3


76.
Which of the following symptoms can be associated with pelvic organ prolapse?

1. Difficulty in passing urine
2. Incomplete evacuation of urine
3. Urgency and frequency

Select the correct answer using the code given below:

(a) 1 and 2 only
(b) 2 and 3 only
(c) 1 and 3 only
(d) 1, 2 and 3
77.
A 30 year old lady, P
2
L
2
presents with painful unilateral swelling in vulva for 3 days. Which
of the following statements are true regarding the above case?

1. Bartholin?s abscess may be the likely diagnosis
2. It is to be managed by marsupialisation
3. Gonococcus is the most common pathogenic organism

Select the correct answer using the code given below:
(a) 1 and 3 only
(b) 3 only
(c) 1 and 2 only
(d) 1, 2 and 3
78.
Which one of the following is NOT a sign of separation of placenta?

(a) Uterus becomes globular, firm and ballotable
(b) The fundal height reduces further
(c) Slight bulging in the suprapubic region
(d) Apparent lengthenic of the cord with slight gush of vaginal bleeding
79.
Consider the following regarding examination of a rape victim:

1. Emergency pill is provided
2. Internal examination must be performed
3. HIV testing is done

Which of the above statements is/are correct?

(a) 1 and 3 only
(b) 2 only
(c) 1, 2 and 3
(d) 3 only




80.
Consider the following cardinal movements of mechanism of normal labor:
1. Engagement
2. Internal rotation
3. Flexion
4. Restitution
5. Crowning
6. External rotation

What is the correct sequence of movements in labor in occipito-lateral position?

(a) 1, 2, 3, 4, 5 and 6
(b) 1, 3, 2, 5, 4 and 6
(c) 2, 1, 3, 4, 5 and 6
(d) 3, 1, 2, 4, 6 and 5
81.
Which of the following represent the properties of an ideal disinfectant?

1. It is broad spectrum
2. It is fast acting
3. It is non-toxic

Select the correct answer using the code given below:
(a) 1 and 2 only
(b) 2 and 3 only
(c) 1 and 3 only
(d) 1, 2 and 3
82.
Which of the following measures can help reduce the risk of systemic hypertension?

1. Reduction in dietary intake of common salt
2. Controlling weight for age
3. Increasing potassium rich foods in the diet

Select the correct answer using the code given below:

(a) 1 and 2 only
(b) 2 and 3 only
(c) 1 and 3 only
(d) 1, 2 and 3








83.
Keeping biological determinants in perspective, consider the following statements:

1. Presence of a normal karyotype is the first requisite for human health
2. Genetic screening can play an important role in prevention of wide spectrum of
diseases
3. If an individual is allowed to live in healthy relationship with the environment, the
person?s genetic potentialities can transform into phenotypic realities

Which of the above statements is/are correct?

(a) 1 and 2 only
(b) 1 and 3 only
(c) 1, 2 and 3
(d) 2 and 3 only
84.
NITI Aayog has the following roles EXCEPT:

(a) It provides critical directional and strategic input in the development process
(b) It provides relevant technical advice with focus on technology upgradation
(c) It focuses on capacity building
(d) It focuses on the development of ?Referral Service Complex?
85.
Poor hand hygiene of a mess worker in a university college mess led to Hepatitis A cases in
the hostel inmates. What type of epidemic will this exposure present with?

1. Propagated
2. Common source-continuous exposure
3. Common source-point exposure

Select the correct answer using the code given below:

(a) 1 and 2
(b) 1 and 3
(c) 1 only
(d) 2 only












86.
What is the specificity of sputum microscopy in detection of Pulmonary Tuberculosis (PTB)
as per the information given below?

PTB Total
Sputum microscopy Present Absent
Positive 270 20 290
Negative 30 180 210
Total 300 200 500

(a) 10 %
(b) 36 %
(c) 90 %
(d) 94 %
87.
In a cohort of 500 women attending antenatal clinic, 70 % had ultrasonography (USG). This
cohort was followed up at delivery. Of the women who had USG, 70 delivered low birth
weight (LBW) babies; whereas of the women, who did not undergo USG, 50 delivered LBW
babies. The incidence of LBW babies among women who had USG is:

(a) 10 %
(b) 15 %
(c) 20 %
(d) 25 %
88.
Major source of vitamin K
1
is:

(a) Fresh dark green vegetables
(b) Exposure of body to sunlight
(c) Citrus fruits
(d) Foods rich in polyunsaturated fatty acids
89.
Which one of the following is NOT a function of Epidemiology?

(a) To study historically the rise and fall of disease in the population
(b) Searching for the causes and risk factor
(c) Identifying syndromes
(d) Making clinical diagnosis
90.
Which one of the following is NOT a contagious disease?

(a) Malaria
(b) Scabies
(c) Trachoma
(d) Leprosy


91.
In primary immune response, how much more antigenic dose is required to induce IgG
antibodies as against the induction of IgM antibodies?

(a) 10 times more
(b) 25 times more
(c) 50 times more
(d) 100 times more
92.
Pentavalent vaccine provides protection against which of the following diseases?

(a) Diphtheria, Pertussis, Tuberculosis, Measles and Hepatitis B
(b) Diphtheria, Pertussis, Measles, Hepatitis B and Hib
(c) Diphtheria, Pertussis, Tetanus, Hepatitis B and Rubella
(d) Diphtheria, Pertussis, Tetanus, Hepatitis B and Hib
93.
Consider the following data for a country:
Population in 0-14 years of age ? 391,558,367
Population between 15?64 years of age ? 856,076,200
Population above 65 years of age ? 71,943,390
What shall be the dependency ratio of this country?

(a) 42.4 %
(b) 54.1 %
(c) 66.2 %
(d) 78.6 %
94.
Which one of the following statements regarding Rabies Immunoglobulin is NOT true?

(a) It should be administered only once as soon as possible after the initiation of post
exposure prophylaxis
(b) It should be administered all into or around the wound sites
(c) There is no scientific ground for performing a skin test prior to administering equine
immunoglobulin
(d) It can be administered till 15 days after the first dose of anti-rabies vaccine
95.
All of the following are true about Bedaquiline (BDQ) EXCEPT:

(a) It specifically targets mycobacterial ATP synthase
(b) It is a bacteriostatic drug
(c) It has extended half life
(d) It has high volume of tissue distribution
96.
Visual inspection based screening test with 5 % acetic acid is used for the screening of which
one of the following cancers?

(a) Lung cancer
(b) Cervix cancer
(c) Oral cancer
(d) Breast cancer
97.
Predictive accuracy of a screening test depends on the following EXCEPT:

(a) Disease prevalence
(b) Disease incidence
(c) Sensitivity of screening test
(d) Specificity of screening test
98.
How much of Zinc supplement is recommended by WHO and UNICEF for infants less than 6
months of age after an episode of acute diarrhoea?

(a) 20 mg per day for 10?14 days
(b) 10 mg per day for 10?14 days
(c) 5 mg per day for 7 days
(d) 6 mg per day for 7 days
99.
Which one of the following statements regarding sequential administration of Inactivated
Polio Vaccine (IPV) and Oral Polio Vaccine (OPV) is NOT correct?

(a) It will be cost effective in developing countries for Polio prevention
(b) The combined schedules of IPV and OPV appear to reduce or prevent Vaccine
Associated Paralytic Polio (VAPP)
(c) Intestinal mucosal immunity is lost due to IPV administration
(d) IPV and OPV together may optimize both the humoral and mucosal immunogenicity
of Polio vaccine
100.
What is the recommended dose regimen of Vitamin A for the treatment of early stages of
Xerophthalmia?

(a) Single massive dose of 2 lac International Units (IU)
(b) 2 lac IU on two successive days
(c) 2 doses of 1 lac IU in two successive days
(d) 2 doses of 1 lac IU at a gap of one week
101.
Which is the most specific causative agent of Rabies?

(a) Lyssavirus serotype 1
(b) Lyssavirus serotype 2
(c) Lyssavirus serotype 3
(d) Lyssavirus serotype 4
102.
Which one of the following is the antibiotic of choice for the prevention of Rheumatic heart
disease?

(a) Benzathine Benzyl Penicillin
(b) Procaine Penicillin
(c) Doxycycline
(d) Ciprofloxacin


103.
All of the following are global targets for WHO Global Action Plan (2013?2020) for
Prevention and Control of NCDs, EXCEPT:

(a) A 10 % relative reduction in risk of premature mortality from cardiovascular diseases,
cancer, diabetes and chronic respiratory diseases
(b) A 10 % relative reduction in prevalence of insufficient physical activity
(c) A 10 % relative reduction in mean population intake of salt/sodium
(d) At least 10 % relative reduction in the harmful use of alcohol
104.
Under the Employees State Insurance (ESI) Scheme, extended sickness benefit is provided in
which of the following infectious diseases?

1. Tuberculosis
2. Leprosy
3. Chronic empyema

Select the correct answer using the code given below:

(a) 1 and 2 only
(b) 2 and 3 only
(c) 1 and 3 only
(d) 1, 2 and 3

105.
In the context of NCD prevention and control in India, the extent of relative reduction in
household use of solid fuels as a primary source of energy for cooking by 2025 is targeted at:

(a) 30 %
(b) 40 %
(c) 50 %
(d) 60 %
106.
Consider the following criteria which may indicate elimination of lymphatic filariasis in a
community:

1. When lymphatic filariasis ceases to be a public health problem in the community
2. When the number of microfilaria carriers declines to 1.5 % within the community
3. When children born in the community after the initiation of elimination programme
are free from circulating antigenaemia

Which of the criteria stated above hold true?

(a) 1 and 2 only
(b) 2 and 3 only
(c) 1 and 3 only
(d) 1, 2 and 3



107.
Hold over time of cold chain equipment depends on all of the following factors EXCEPT:

(a) Ambient temperature
(b) Quantity of vaccines kept
(c) Types of vaccines kept
(d) Condition of icepack lining
108.
Which of the following statements regarding Physical Quality of Life Index (PQLI) are
correct?

1. It consolidates infant mortality, life expectancy at age one, and literacy
2. It does not measure economic growth but measures the result of economic policies
3. For each component, the performance of individual countries is placed on a scale of
0-100

Select the correct answer using the code given below:

(a) 1 and 2 only
(b) 2 and 3 only
(c) 1 and 3 only
(d) 1, 2 and 3
109.
Disability-adjusted life years (DALYs) include:

(a) Years of lost life (YLL)
(b) Years lost to disability (YLD)
(c) Both YLL and YLD
(d) Neither YLL nor YLD
110.
Which of the following are health care delivery indicators?

1. Population per trained birth attendant
2. Population per health/sub centre
3. Doctor-nurse ratio

Select the correct answer using the code given below:

(a) 1 and 2 only
(b) 2 and 3 only
(c) 1 and 3 only
(d) 1, 2 and 3








FirstRanker.com - FirstRanker's Choice
Combined Medical Services Examination-2020
Paper-II

1.
Indications for fasciotomy in compartment syndrome include all EXCEPT:

(a) Distal sensory disturbance
(b) Compartment pressure > 30 mm Hg
(c) Pain on passive movement of affected muscles
(d) Palpable distal pulses
2.
Which one of the following statements is NOT correct regarding Necrotising Soft Tissue
infections?

(a) Crepitus, skin blistering and focal skin gangrene are typical presenting features
(b) They are monomicrobial in nature
(c) Treatment consists of wide local excision and appropriate antibiotics
(d) Tissue biopsy is required for culture and diagnosis
3.
Which type of surgery is laparoscopic cholecystectomy classified as?

(a) Clean
(b) Clean contaminated
(c) Contaminated
(d) Dirty
4.
A 22-year female has presented with a history of malaise, cough, alternating constipation and
diarrhoea with intermittent abdominal pain for last 6 months. She also complains of
abdominal distension for last 2 days. On examination her abdomen has a doughy feel along
with an ill defined mass over the right lower quadrant. She is most likely suffering from:

(a) Appendicular lump
(b) Ileocaecal tuberculosis
(c) Carcinoma caecum
(d) Ovarian mass
5.
Consider the following statements regarding needle stick injuries:

1. Injured part should be washed under running water
2. Dominant index finger is the commonest site for needle stick injury
3. All needle stick injuries should be reported
4. Hepatitis/HIV testing should be done after needle stick injury

Which of the statements given above are correct?

(a) 1, 2 and 4
(b) 1, 2 and 3
(c) 1, 3 and 4
(d) 2, 3 and 4




6.
Consider the following statements regarding claudication:

1. It is a marker for silent coronary disease
2. Structured exercise program ( 2 hours per week for 3 months) leads to
improvement in symptoms
3. Diabetes mellitus increases the risk and severity of claudication
4. Beta blockers may exacerbate claudication

Which of the above statements are correct?

(a) 1 and 2 only
(b) 1, 3 and 4 only
(c) 2, 3 and 4 only
(d) 1, 2, 3 and 4
7.
A 50-year old lady underwent uneventful bariatric surgery for morbid obesity. On the third
post operative day, she develops breathless and pulmonary embolism is suspected. The next
investigation to confirm the diagnosis will be:

(a) Echocardiography
(b) Duplex venography
(c) CT pulmonary angiography
(d) MR angiography
8.
Medical management of thyrotoxic crisis includes all of the following EXCEPT:

(a) IV fluids
(b) IV propanolol
(c) IV hydrocortisone
(d) IV antibiotics
9.
A patient operated for a parotid gland tumour developed symptoms of sweating and
erytherma (flushing) over the region of surgical excision while eating. The probable
diagnosis is:

(a) Parotid gland fistula
(b) Sialadenitis
(c) Chronic wound infection
(d) Frey?s syndrome
10.
First line hormone therapy for post-menopausal woman with metastatic carcinoma breast is:

(a) Tamoxifen
(b) Ovarian suppression by surgery
(c) Antiprogestins
(d) Anastrazole



11.
All of the following are major subtypes of breast cancer based on Gene array analysis
EXCEPT:

(a) Luminal A and Luminal B
(b) Triple negative
(c) Her-2 receptor positive
(d) Oestrogen receptor positive
12.
All of the following are sequelae of peptic ulcer surgery EXCEPT:

(a) Bilious vomiting
(b) Dumping syndrome
(c) Diarrhoea
(d) Increased appetite
13.
The Child-Turcotte-Pugh (CTP) score for quantifying the severity of chronic liver disease
includes all variables EXCEPT:

(a) Serum bilirubin
(b) Serum albumin
(c) Serum creatinine
(d) INR (International Normalised Ratio)
14.
?Chain of Lakes? appearance due to sacculation with intervening short strictures of pancreatic
duct is seen on:

(a) ERCP
(b) CECT abdomen
(c) Plain X-ray abdomen
(d) Ultrasonography
15.
Which one of the following statements is NOT correct regarding Pyogenic Liver Abscess?

(a) Anorexia, fever, malaise and right upper quadrant abdominal discomfort are the
most common presenting features
(b) It is more common in elderly, diabetics and immunocompromised patients
(c) Treatment is with oral antibiotics alone
(d) Streptococcus milleri and escherichia coli are the most common causative
organisms
16.
?Swiss cheese defects? are seen during laparoscopic repair of:

(a) Ventral hernia
(b) Inguinal hernia
(c) Obturator hernia
(d) Femoral hernia



17.
Which of the following are correct regarding splenic artery aneurysm?

1. Main arterial trunk is the common site
2. Palpable thrill can be felt
3. It is symptomless unless it ruptures

Select the correct answer using the code given below:

(a) 1 and 2 only
(b) 2 and 3 only
(c) 1 and 3 only
(d) 1, 2 and 3
18.
Valentino?s syndrome is:

(a) Pain on per-vaginal examination in pelvic abscess
(b) Pain over left shoulder in left hypochondriac collection
(c) Pain over left groin in perirenal collection
(d) Pain in right iliac fossa in perforated peptic ulcer
19.
Spontaneous bacterial peritonitis occurs due to:

(a) duodenal stump blowout
(b) peptic ulcer perforation
(c) acute bacterial infection of ascites
(d) infection via fallopian tubes
20.
Structure not forming boundaries of the ?Triangle of doom? seen during laparoscopic
inguinal hernia surgery dissection is:

(a) Vas deferens
(b) Inferior epigastric artery
(c) Spermatic cord vessels
(d) Peritoneum
21.
The term mid-line shift is associated with:

(a) Head injury
(b) Chest injury
(c) Abdominal injury
(d) Limb injury
22.
Which one of the following cranial nerves does NOT supply to the external ear?

(a) Cranial nerve V
(b) Cranial nerve VI
(c) Cranial nerve VII
(d) Cranial nerve IX

23.
Left Internal Mammary Artery (LIMA) has become the conduit of choice for Left Anterior
Descending (LAD) artery during coronary artery bypass grafting because:

(a) Long term patency rates are more than 98%
(b) It is close to LAD
(c) Atherosclerosis is never seen in this vessel
(d) It is very easy to harvest
24.
A 50-year old male with significant smoking history presented in the surgical emergency
with sudden severe breathlessness. Chest X-ray shows right sided Pneumothorax. The
appropriate management requires:

(a) Aspiration of air with 16-18 G cannula
(b) Right chest drain of size 8-14 Fr
(c) Oxygen by face mask
(d) Mechanical ventilation
25.
Which one of the following statements regarding Felon is NOT correct?

(a) There is infection of the finger tip between specialised fibrous septa
(b) It is a painless condition
(c) Incision and drainage is the treatment of choice
(d) It is common in diabetics
26.
The most common site for osteosarcoma is:

(a) Proximal femur
(b) Distal femur
(c) Proximal humerus
(d) Distal humerus
27.
Rapid Sequence Induction is indicated in:

(a) Emergency surgery for intestinal obstruction
(b) Elective open hernia surgery
(c) Cardiopulmonary bypass surgery
(d) Elective laparoscopic surgery

28.
Which one of the following statements about Compartment Syndrome is NOT correct?

(a) It is commonest in a closed fracture
(b) Pain is on active movement but not on passive movement of muscles
(c) Fasciotomy is the treatment of choice
(d) Volkmann?s Ischaemic contractive is a late complication




29.
Which one of the following is NOT the strength of ultrasound as a diagnostic modality?

(a) No radiation
(b) Short learning curve
(c) Inexpensive
(d) Allows dynamic studies to be done
30.
Which of the following statements regarding lymphoedema following breast cancer treatment
are correct?

1. Incidence has decreased due to rarely combined therapy of axillary LN dissection and
radiotherapy
2. Precipitating cause like LN metastasis is a major determinant
3. The condition is often painful
4. Oedematous limb is susceptible to bacterial infection

Select the correct answer using the code given below:

(a) 1, 2 and 3
(b) 2, 3 and 4
(c) 1, 3 and 4
(d) 1, 2 and 4
31.
Which one of the following is NOT a risk factor for development of venous thrombosis in
surgical patients?

(a) Age > 60 years
(b) Pregnancy
(c) Obesity (BMI > 30 kg/m
2
)
(d) Diabetes ( HbA1c > 7.5%)
32.
Which one of the following is NOT a complication of massive blood transfusion?

(a) Coagulopathy
(b) Hypocalcaemia
(c) Hyperthermia
(d) Hyperkalemia
33.
Stage III ?Pressure sore? is full thickness skin loss extending:

(a) into subcutaneous tissue but not through fascia
(b) through subcutaneous tissue into fascia
(c) through subcutaneous tissue into fascia and muscles
(d) through subcutaneous tissue into fascia, muscles and bone



34.
During subclavian vein puncture in a surgical ward suddenly a patient developed severe
breathlessness. On auscultation breath sound was absent and the ipsilateral chest was
tympanitic on percussion. The probable diagnosis is:

(a) Iatrogenic pneumothorax
(b) Introgenic hemothorax
(c) Spontaneous pneumothorax
(d) Tension pneumothorax
35.
Kohler?s disease is avascular necrosis of :

(a) Lunate
(b) Capitellum of humerus
(c) First metatarsal head
(d) Navicular
36.
Which one of the following is NOT true of Pyoderma gangrenosum?

(a) It is characterized by cutaneous ulceration with purple undermined edges
(b) It is often secondary to heightened immunological reactivity from another disease
process
(c) Cultures often show Gram positive Staphylococci
(d) Lesions generally respond to steroids
37.
Which of the following is NOT a tissue repair surgery for inguinal hernia repair?

(a) Bassini?s repair
(b) Shouldice repair
(c) Stoppa?s repair
(d) Desarda repair
38.
Which one of the following type of meshes is recommended for intraperitoneal use in
abdominal wall hernia?
(a) Light weight, porous meshes
(b) Heavy weight, porous meshes
(c) Absorbable meshes
(d) Tissue separating meshes
39.
Which one of the following is NOT correct regarding MEN-1 syndrome?

(a) It involves parathyroid glands
(b) It involves pancreas
(c) It involves pituitary gland
(d) It involves pineal gland

40.
Which one of the following is NOT an electronic information site in surgery?

(a) Pubmed
(b) Embase
(c) Cochrane library
(d) National medical library
41.
Which one of the following statements regarding pre-conceptional counseling is NOT
correct?

(a) It is needed only in selected complicated pregnancies
(b) It helps in early detection of risk factors
(c) It helps in reducing maternal morbidity and mortality
(d) It is a part of preventive medicine
42.
Consider the following statements regarding Non Stress Test (NST):

1. Reactive NST indicates a healthy fetus
2. NST is an observed association of fetal breathing with fetal movements
3. NST has a low false negative rate (< 1%) but high false positive rate (>50%)
4. Testing should be started at 20 weeks

Which of the statement(s) given above is/are correct?

(a) 1 and 3
(b) 2 only
(c) 3 only
(d) 1 and 4
43.
Which one of the following is a protective factor for endometrial hyperplasia?

(a) Diabetes
(b) Tamoxifen therapy
(c) Multiparity
(d) Delayed menopause
44.
A woman who is not breast feeding her newborn child is advised to use a contraceptive
method by:

(a) 3
rd
postpartum week
(b) 6
th
postpartum week
(c) 3
rd
postpartum month
(d) 6
th
postpartum month





45.
Pearl index for contraceptive effectiveness is calculated in terms of which of the following?

1. Pregnancy rate
2. Abortion rate
3. Hundred woman years
4. Thousand woman years

Select the correct answer using the code given below:

(a) 1 only
(b) 2 and 3
(c) 1, 2 and 4
(d) 1 and 3
46.
Indications for removal of IUDs are all EXCEPT:

(a) Perforation of uterus
(b) Cyclical menstrual bleeding
(c) Flaring up of salpingitis
(d) Pregnancy with IUD
47.
Contraindications for insertion of IUDs are all EXCEPT:

(a) Suspected pregnancy
(b) Trophoblastic disease
(c) Severe dysmenorrhea
(d) During cesarean section
48.
Which one of the following is NOT a contraindication for use of Mini pill?

(a) Pregnancy
(b) Breast feeding
(c) Thromboembolic disease
(d) History of breast cancer
49.
Which one of the following is the most commonly used surgical method/technique of female
sterilization as recommended by Government of India?

(a) Uchida technique
(b) Irving method
(c) Pomeroy?s method
(d) Madlener technique







50.
Which of the following is/are required for a registered medical practitioner to qualify for
performing Medical Termination of Pregnancy (MTP), as per revised rules of MTP Act?

1. Certified for assisting at least 15 MTP in an authorized centre
2. Diploma or degree in Obstetrics and Gynaecology
3. House surgeon training for 3 months in Obstetrics and Gynaecology
4. Certified training for 6 months in laparoscopic surgeries

Select the correct answer using the code given below:
(a) 1 only
(b) 2 only
(c) 1, 2 and 3
(d) 1, 2 and 4
51.
Which one of the following is NOT a support of uterus, preventing its descent?

(a) Endopelvic fascia
(b) Mackenrodt?s ligament
(c) Inguinal ligament
(d) Pubocervical ligament
52.
As per ICMR guidelines, which one of the following statements is true regarding effects of
COVID-19 on fetus according to current evidence?

(a) There is increased risk of early pregnancy loss
(b) COVID-19 virus is not teratogenic
(c) COVID-19 virus infection is an indication of MTP
(d) There is increased risk of fetal growth restriction
53.
As per ICMR guidelines, which one of the following statements is true regarding COVID-19
infection in pregnancy?

(a) Covid-19 pneumonia in pregnancy is more severe with poor recovery
(b) Pregnant women with heart disease are at higher risk
(c) Vaginal secretions always test positive for COVID-19 in pregnancy
(d) COVID-19 virus is secreted in breast milk
54.
Which one of the following is NOT a method of management of Deep Transverse Arrest with
the living fetus?

(a) Caesarean section
(b) Delivery by ventouse
(c) Delivery by application of forceps to the unrotated head
(d) Manual rotation and application of forceps





55.
Successful version of breech presentation is likely in case all of the following EXCEPT:

(a) Breech with extended legs
(b) Complete breech with sacroanterior position
(c) Non engaged breech
(d) Adequate amniotic fluid
56.
Implantation of a fertilised ovum occurs on which day following fertilisation?

(a) Day 6
(b) Day 10
(c) Day 14
(d) Day 20
57.
During total abdominal hysterectomy the ureter is likely to undergo injury or ligation during
the following steps EXCEPT:

(a) During division and ligation of the round ligaments
(b) During division and ligation of infundibulopelvic ligaments
(c) During division and ligation of mackenrodt?s and uterosacral ligaments
(d) At the vaginal angles while incising the vagina to remove the cervix with the
uterus
58.
The net effect of antenatal care has been the following EXCEPT:

(a) Reduction in maternal mortality
(b) Reduction in perinatal mortality
(c) Reduction in the incidence of institutional delivery
(d) Reduction in maternal morbidity
59.
Which one of the following is NOT a component of active phase in the partograph?

(a) Acceleration phase
(b) Phase of maximum slope
(c) Phase of deceleration
(d) Phase of expulsion
60.
From medicolegal point of view which one of the following is NOT a sign of previous child
birth?

(a) Perineum is lax and there is evidence of scarring
(b) Introitus is gaping and there is presence of carunculae myrtiformis
(c) Abdomen is lax and loose with striae and linea alba
(d) Conical cervix with round external os





61.
The components of partograph are all EXCEPT:

(a) Time
(b) Fetal heart rate
(c) Maternal respiratory rate
(d) Maternal urine analysis
62.
Which of the following information are provided by partograph?

1. Colour of liquor
2. Uterine contractions with duration and frequency
3. Dilatation of cervix

Select the correct answer using the code given below:

(a) 1 and 2 only
(b) 2 and 3 only
(c) 1 and 3 only
(d) 1, 2 and 3
63.
Intraoperative recognition of ureter is by which of the following features?

1. Transparent tubular appearance
2. Pale glistening appearance
3. Longitudinal vessels on surface
4. Circumferential vessels on surface

Select the correct answer using the code given below:

(a) 1 and 3
(b) 2 and 4
(c) 2 and 3
(d) 1 and 4
64.
Hysterosalpingography (HSG) is least helpful in detecting which of the following?

(a) Tubal patency
(b) Pelvic adhesions
(c) Asherman syndrome
(d) Congenital uterine anomaly









65.
Which of the following are characteristics of Trichomonas vaginitis?

1. Presence of greenish frothy discharge
2. Vaginal pH > 4.5
3. Presence of clue cells in microscopic examination
4. Strawberry spots on the vaginal mucosa

Select the correct answer using the code given below:

(a) 1, 2 and 3
(b) 1, 2 and 4
(c) 2, 3 and 4
(d) 1, 3 and 4
66.
Tumor marker of epithelial ovarian carcinoma is:

(a) Ca.125
(b) Alpha feto protein
(c) Beta HCG
(d) LDH
67.
The most common site of cervical cancer is:

(a) Endocervix
(b) Ectocervix
(c) Transformation zone
(d) Isthmus
68.
The placenta synthesizes all EXCEPT:

(a) Oestriol
(b) Corticotrophin releasing hormone
(c) PAPP-A(Pregnancy Associated Plasma Protein A)
(d) Dehydroepiandrosterone
69.
Withdrawal bleeding following administration of progesterone in a case of secondary
amenorrhea indicates all EXCEPT:

(a) Absence of pregnancy
(b) Production endogenous estrogen
(c) Endometrium is responsive to estrogen
(d) Defect in pituitary gland
70.
Monilial vaginitis is commonly associated with all EXCEPT:

(a) Prolonged antibiotic therapy
(b) Diabetes Mellitus
(c) Treatment of malaria with chloroquine
(d) Pregnancy
71.
Which one of the following is NOT a risk factor for the development of placenta previa?

(a) Maternal age
(b) Smoking
(c) Previous caesarean section
(d) Maternal anaemia
72.
Common clinical presentations of moderate to severe abruption are all EXCEPT:

(a) Uterine tenderness
(b) Fetal distress
(c) Unexplained pre term labour
(d) Prolonged labour
73.
Common trisomies resulting in spontaneous abortion are all EXCEPT:

(a) Trisomy 21
(b) Trisomy 18
(c) Trisomy 16
(d) Trisomy 1
74.
The initial prevention strategy for antiphospholipid syndrome will be:

1. Steroids
2. Heparin
3. Low dose aspirin
4. Progesterone support

Which of the above is/are correct?

(a) 2 and 3
(b) 3 and 4
(c) 3 only
(d) 1 and 4
75.
Diagnostic criteria for PCOD are:

1. Oligo/amenorrohoea
2. Hyperandrogenism
3. Polycystic ovaries on ultrasound

Which of the above are correct?

(a) 1 and 2 only
(b) 2 and 3 only
(c) 1 and 3 only
(d) 1, 2 and 3


76.
Which of the following symptoms can be associated with pelvic organ prolapse?

1. Difficulty in passing urine
2. Incomplete evacuation of urine
3. Urgency and frequency

Select the correct answer using the code given below:

(a) 1 and 2 only
(b) 2 and 3 only
(c) 1 and 3 only
(d) 1, 2 and 3
77.
A 30 year old lady, P
2
L
2
presents with painful unilateral swelling in vulva for 3 days. Which
of the following statements are true regarding the above case?

1. Bartholin?s abscess may be the likely diagnosis
2. It is to be managed by marsupialisation
3. Gonococcus is the most common pathogenic organism

Select the correct answer using the code given below:
(a) 1 and 3 only
(b) 3 only
(c) 1 and 2 only
(d) 1, 2 and 3
78.
Which one of the following is NOT a sign of separation of placenta?

(a) Uterus becomes globular, firm and ballotable
(b) The fundal height reduces further
(c) Slight bulging in the suprapubic region
(d) Apparent lengthenic of the cord with slight gush of vaginal bleeding
79.
Consider the following regarding examination of a rape victim:

1. Emergency pill is provided
2. Internal examination must be performed
3. HIV testing is done

Which of the above statements is/are correct?

(a) 1 and 3 only
(b) 2 only
(c) 1, 2 and 3
(d) 3 only




80.
Consider the following cardinal movements of mechanism of normal labor:
1. Engagement
2. Internal rotation
3. Flexion
4. Restitution
5. Crowning
6. External rotation

What is the correct sequence of movements in labor in occipito-lateral position?

(a) 1, 2, 3, 4, 5 and 6
(b) 1, 3, 2, 5, 4 and 6
(c) 2, 1, 3, 4, 5 and 6
(d) 3, 1, 2, 4, 6 and 5
81.
Which of the following represent the properties of an ideal disinfectant?

1. It is broad spectrum
2. It is fast acting
3. It is non-toxic

Select the correct answer using the code given below:
(a) 1 and 2 only
(b) 2 and 3 only
(c) 1 and 3 only
(d) 1, 2 and 3
82.
Which of the following measures can help reduce the risk of systemic hypertension?

1. Reduction in dietary intake of common salt
2. Controlling weight for age
3. Increasing potassium rich foods in the diet

Select the correct answer using the code given below:

(a) 1 and 2 only
(b) 2 and 3 only
(c) 1 and 3 only
(d) 1, 2 and 3








83.
Keeping biological determinants in perspective, consider the following statements:

1. Presence of a normal karyotype is the first requisite for human health
2. Genetic screening can play an important role in prevention of wide spectrum of
diseases
3. If an individual is allowed to live in healthy relationship with the environment, the
person?s genetic potentialities can transform into phenotypic realities

Which of the above statements is/are correct?

(a) 1 and 2 only
(b) 1 and 3 only
(c) 1, 2 and 3
(d) 2 and 3 only
84.
NITI Aayog has the following roles EXCEPT:

(a) It provides critical directional and strategic input in the development process
(b) It provides relevant technical advice with focus on technology upgradation
(c) It focuses on capacity building
(d) It focuses on the development of ?Referral Service Complex?
85.
Poor hand hygiene of a mess worker in a university college mess led to Hepatitis A cases in
the hostel inmates. What type of epidemic will this exposure present with?

1. Propagated
2. Common source-continuous exposure
3. Common source-point exposure

Select the correct answer using the code given below:

(a) 1 and 2
(b) 1 and 3
(c) 1 only
(d) 2 only












86.
What is the specificity of sputum microscopy in detection of Pulmonary Tuberculosis (PTB)
as per the information given below?

PTB Total
Sputum microscopy Present Absent
Positive 270 20 290
Negative 30 180 210
Total 300 200 500

(a) 10 %
(b) 36 %
(c) 90 %
(d) 94 %
87.
In a cohort of 500 women attending antenatal clinic, 70 % had ultrasonography (USG). This
cohort was followed up at delivery. Of the women who had USG, 70 delivered low birth
weight (LBW) babies; whereas of the women, who did not undergo USG, 50 delivered LBW
babies. The incidence of LBW babies among women who had USG is:

(a) 10 %
(b) 15 %
(c) 20 %
(d) 25 %
88.
Major source of vitamin K
1
is:

(a) Fresh dark green vegetables
(b) Exposure of body to sunlight
(c) Citrus fruits
(d) Foods rich in polyunsaturated fatty acids
89.
Which one of the following is NOT a function of Epidemiology?

(a) To study historically the rise and fall of disease in the population
(b) Searching for the causes and risk factor
(c) Identifying syndromes
(d) Making clinical diagnosis
90.
Which one of the following is NOT a contagious disease?

(a) Malaria
(b) Scabies
(c) Trachoma
(d) Leprosy


91.
In primary immune response, how much more antigenic dose is required to induce IgG
antibodies as against the induction of IgM antibodies?

(a) 10 times more
(b) 25 times more
(c) 50 times more
(d) 100 times more
92.
Pentavalent vaccine provides protection against which of the following diseases?

(a) Diphtheria, Pertussis, Tuberculosis, Measles and Hepatitis B
(b) Diphtheria, Pertussis, Measles, Hepatitis B and Hib
(c) Diphtheria, Pertussis, Tetanus, Hepatitis B and Rubella
(d) Diphtheria, Pertussis, Tetanus, Hepatitis B and Hib
93.
Consider the following data for a country:
Population in 0-14 years of age ? 391,558,367
Population between 15?64 years of age ? 856,076,200
Population above 65 years of age ? 71,943,390
What shall be the dependency ratio of this country?

(a) 42.4 %
(b) 54.1 %
(c) 66.2 %
(d) 78.6 %
94.
Which one of the following statements regarding Rabies Immunoglobulin is NOT true?

(a) It should be administered only once as soon as possible after the initiation of post
exposure prophylaxis
(b) It should be administered all into or around the wound sites
(c) There is no scientific ground for performing a skin test prior to administering equine
immunoglobulin
(d) It can be administered till 15 days after the first dose of anti-rabies vaccine
95.
All of the following are true about Bedaquiline (BDQ) EXCEPT:

(a) It specifically targets mycobacterial ATP synthase
(b) It is a bacteriostatic drug
(c) It has extended half life
(d) It has high volume of tissue distribution
96.
Visual inspection based screening test with 5 % acetic acid is used for the screening of which
one of the following cancers?

(a) Lung cancer
(b) Cervix cancer
(c) Oral cancer
(d) Breast cancer
97.
Predictive accuracy of a screening test depends on the following EXCEPT:

(a) Disease prevalence
(b) Disease incidence
(c) Sensitivity of screening test
(d) Specificity of screening test
98.
How much of Zinc supplement is recommended by WHO and UNICEF for infants less than 6
months of age after an episode of acute diarrhoea?

(a) 20 mg per day for 10?14 days
(b) 10 mg per day for 10?14 days
(c) 5 mg per day for 7 days
(d) 6 mg per day for 7 days
99.
Which one of the following statements regarding sequential administration of Inactivated
Polio Vaccine (IPV) and Oral Polio Vaccine (OPV) is NOT correct?

(a) It will be cost effective in developing countries for Polio prevention
(b) The combined schedules of IPV and OPV appear to reduce or prevent Vaccine
Associated Paralytic Polio (VAPP)
(c) Intestinal mucosal immunity is lost due to IPV administration
(d) IPV and OPV together may optimize both the humoral and mucosal immunogenicity
of Polio vaccine
100.
What is the recommended dose regimen of Vitamin A for the treatment of early stages of
Xerophthalmia?

(a) Single massive dose of 2 lac International Units (IU)
(b) 2 lac IU on two successive days
(c) 2 doses of 1 lac IU in two successive days
(d) 2 doses of 1 lac IU at a gap of one week
101.
Which is the most specific causative agent of Rabies?

(a) Lyssavirus serotype 1
(b) Lyssavirus serotype 2
(c) Lyssavirus serotype 3
(d) Lyssavirus serotype 4
102.
Which one of the following is the antibiotic of choice for the prevention of Rheumatic heart
disease?

(a) Benzathine Benzyl Penicillin
(b) Procaine Penicillin
(c) Doxycycline
(d) Ciprofloxacin


103.
All of the following are global targets for WHO Global Action Plan (2013?2020) for
Prevention and Control of NCDs, EXCEPT:

(a) A 10 % relative reduction in risk of premature mortality from cardiovascular diseases,
cancer, diabetes and chronic respiratory diseases
(b) A 10 % relative reduction in prevalence of insufficient physical activity
(c) A 10 % relative reduction in mean population intake of salt/sodium
(d) At least 10 % relative reduction in the harmful use of alcohol
104.
Under the Employees State Insurance (ESI) Scheme, extended sickness benefit is provided in
which of the following infectious diseases?

1. Tuberculosis
2. Leprosy
3. Chronic empyema

Select the correct answer using the code given below:

(a) 1 and 2 only
(b) 2 and 3 only
(c) 1 and 3 only
(d) 1, 2 and 3

105.
In the context of NCD prevention and control in India, the extent of relative reduction in
household use of solid fuels as a primary source of energy for cooking by 2025 is targeted at:

(a) 30 %
(b) 40 %
(c) 50 %
(d) 60 %
106.
Consider the following criteria which may indicate elimination of lymphatic filariasis in a
community:

1. When lymphatic filariasis ceases to be a public health problem in the community
2. When the number of microfilaria carriers declines to 1.5 % within the community
3. When children born in the community after the initiation of elimination programme
are free from circulating antigenaemia

Which of the criteria stated above hold true?

(a) 1 and 2 only
(b) 2 and 3 only
(c) 1 and 3 only
(d) 1, 2 and 3



107.
Hold over time of cold chain equipment depends on all of the following factors EXCEPT:

(a) Ambient temperature
(b) Quantity of vaccines kept
(c) Types of vaccines kept
(d) Condition of icepack lining
108.
Which of the following statements regarding Physical Quality of Life Index (PQLI) are
correct?

1. It consolidates infant mortality, life expectancy at age one, and literacy
2. It does not measure economic growth but measures the result of economic policies
3. For each component, the performance of individual countries is placed on a scale of
0-100

Select the correct answer using the code given below:

(a) 1 and 2 only
(b) 2 and 3 only
(c) 1 and 3 only
(d) 1, 2 and 3
109.
Disability-adjusted life years (DALYs) include:

(a) Years of lost life (YLL)
(b) Years lost to disability (YLD)
(c) Both YLL and YLD
(d) Neither YLL nor YLD
110.
Which of the following are health care delivery indicators?

1. Population per trained birth attendant
2. Population per health/sub centre
3. Doctor-nurse ratio

Select the correct answer using the code given below:

(a) 1 and 2 only
(b) 2 and 3 only
(c) 1 and 3 only
(d) 1, 2 and 3








111.
Which of the following key action areas are incorporated in the Ottawa Charter?

1. Building healthy public policy
2. Strengthening community action for health
3. Reorienting health services

Select the correct answer using the code given below:

(a) 1 and 2 only
(b) 2 and 3 only
(c) 1 and 3 only
(d) 1, 2 and 3
112.
Under Chapter XXI of the 10
th
revision of the International Statistical Classification of
Diseases and Related Health Problems (ICD-10), lifestyle-related problems fall under which
of the following code range?

(a) U 50.0 ? U 50.5
(b) U 10.0 ? U 10.5
(c) Z 72.0 ? Z 72.5
(d) Z 10.0 ? Z 10.5
113.
Which of the following diseases are spread by airborne transmission?

1. Influenza
2. Chicken pox
3. Q-fever
4. Psittacosis

Select the correct answer using the code given below:

(a) 1 and 3 only
(b) 1 and 4 only
(c) 3 and 4 only
(d) 1, 2, 3 and 4
114.
DASH diet plan is advocated in the control of which one of the following diseases?

(a) Cancer
(b) Chronic Obstructive Pulmonary Disease
(c) Hypertension
(d) Rheumatic heart disease
115.
Endemic ascitis occurs due to:

(a) Naturally occurring toxin
(b) Bacterial toxin
(c) Fungal toxin
(d) Viral toxin
FirstRanker.com - FirstRanker's Choice
Combined Medical Services Examination-2020
Paper-II

1.
Indications for fasciotomy in compartment syndrome include all EXCEPT:

(a) Distal sensory disturbance
(b) Compartment pressure > 30 mm Hg
(c) Pain on passive movement of affected muscles
(d) Palpable distal pulses
2.
Which one of the following statements is NOT correct regarding Necrotising Soft Tissue
infections?

(a) Crepitus, skin blistering and focal skin gangrene are typical presenting features
(b) They are monomicrobial in nature
(c) Treatment consists of wide local excision and appropriate antibiotics
(d) Tissue biopsy is required for culture and diagnosis
3.
Which type of surgery is laparoscopic cholecystectomy classified as?

(a) Clean
(b) Clean contaminated
(c) Contaminated
(d) Dirty
4.
A 22-year female has presented with a history of malaise, cough, alternating constipation and
diarrhoea with intermittent abdominal pain for last 6 months. She also complains of
abdominal distension for last 2 days. On examination her abdomen has a doughy feel along
with an ill defined mass over the right lower quadrant. She is most likely suffering from:

(a) Appendicular lump
(b) Ileocaecal tuberculosis
(c) Carcinoma caecum
(d) Ovarian mass
5.
Consider the following statements regarding needle stick injuries:

1. Injured part should be washed under running water
2. Dominant index finger is the commonest site for needle stick injury
3. All needle stick injuries should be reported
4. Hepatitis/HIV testing should be done after needle stick injury

Which of the statements given above are correct?

(a) 1, 2 and 4
(b) 1, 2 and 3
(c) 1, 3 and 4
(d) 2, 3 and 4




6.
Consider the following statements regarding claudication:

1. It is a marker for silent coronary disease
2. Structured exercise program ( 2 hours per week for 3 months) leads to
improvement in symptoms
3. Diabetes mellitus increases the risk and severity of claudication
4. Beta blockers may exacerbate claudication

Which of the above statements are correct?

(a) 1 and 2 only
(b) 1, 3 and 4 only
(c) 2, 3 and 4 only
(d) 1, 2, 3 and 4
7.
A 50-year old lady underwent uneventful bariatric surgery for morbid obesity. On the third
post operative day, she develops breathless and pulmonary embolism is suspected. The next
investigation to confirm the diagnosis will be:

(a) Echocardiography
(b) Duplex venography
(c) CT pulmonary angiography
(d) MR angiography
8.
Medical management of thyrotoxic crisis includes all of the following EXCEPT:

(a) IV fluids
(b) IV propanolol
(c) IV hydrocortisone
(d) IV antibiotics
9.
A patient operated for a parotid gland tumour developed symptoms of sweating and
erytherma (flushing) over the region of surgical excision while eating. The probable
diagnosis is:

(a) Parotid gland fistula
(b) Sialadenitis
(c) Chronic wound infection
(d) Frey?s syndrome
10.
First line hormone therapy for post-menopausal woman with metastatic carcinoma breast is:

(a) Tamoxifen
(b) Ovarian suppression by surgery
(c) Antiprogestins
(d) Anastrazole



11.
All of the following are major subtypes of breast cancer based on Gene array analysis
EXCEPT:

(a) Luminal A and Luminal B
(b) Triple negative
(c) Her-2 receptor positive
(d) Oestrogen receptor positive
12.
All of the following are sequelae of peptic ulcer surgery EXCEPT:

(a) Bilious vomiting
(b) Dumping syndrome
(c) Diarrhoea
(d) Increased appetite
13.
The Child-Turcotte-Pugh (CTP) score for quantifying the severity of chronic liver disease
includes all variables EXCEPT:

(a) Serum bilirubin
(b) Serum albumin
(c) Serum creatinine
(d) INR (International Normalised Ratio)
14.
?Chain of Lakes? appearance due to sacculation with intervening short strictures of pancreatic
duct is seen on:

(a) ERCP
(b) CECT abdomen
(c) Plain X-ray abdomen
(d) Ultrasonography
15.
Which one of the following statements is NOT correct regarding Pyogenic Liver Abscess?

(a) Anorexia, fever, malaise and right upper quadrant abdominal discomfort are the
most common presenting features
(b) It is more common in elderly, diabetics and immunocompromised patients
(c) Treatment is with oral antibiotics alone
(d) Streptococcus milleri and escherichia coli are the most common causative
organisms
16.
?Swiss cheese defects? are seen during laparoscopic repair of:

(a) Ventral hernia
(b) Inguinal hernia
(c) Obturator hernia
(d) Femoral hernia



17.
Which of the following are correct regarding splenic artery aneurysm?

1. Main arterial trunk is the common site
2. Palpable thrill can be felt
3. It is symptomless unless it ruptures

Select the correct answer using the code given below:

(a) 1 and 2 only
(b) 2 and 3 only
(c) 1 and 3 only
(d) 1, 2 and 3
18.
Valentino?s syndrome is:

(a) Pain on per-vaginal examination in pelvic abscess
(b) Pain over left shoulder in left hypochondriac collection
(c) Pain over left groin in perirenal collection
(d) Pain in right iliac fossa in perforated peptic ulcer
19.
Spontaneous bacterial peritonitis occurs due to:

(a) duodenal stump blowout
(b) peptic ulcer perforation
(c) acute bacterial infection of ascites
(d) infection via fallopian tubes
20.
Structure not forming boundaries of the ?Triangle of doom? seen during laparoscopic
inguinal hernia surgery dissection is:

(a) Vas deferens
(b) Inferior epigastric artery
(c) Spermatic cord vessels
(d) Peritoneum
21.
The term mid-line shift is associated with:

(a) Head injury
(b) Chest injury
(c) Abdominal injury
(d) Limb injury
22.
Which one of the following cranial nerves does NOT supply to the external ear?

(a) Cranial nerve V
(b) Cranial nerve VI
(c) Cranial nerve VII
(d) Cranial nerve IX

23.
Left Internal Mammary Artery (LIMA) has become the conduit of choice for Left Anterior
Descending (LAD) artery during coronary artery bypass grafting because:

(a) Long term patency rates are more than 98%
(b) It is close to LAD
(c) Atherosclerosis is never seen in this vessel
(d) It is very easy to harvest
24.
A 50-year old male with significant smoking history presented in the surgical emergency
with sudden severe breathlessness. Chest X-ray shows right sided Pneumothorax. The
appropriate management requires:

(a) Aspiration of air with 16-18 G cannula
(b) Right chest drain of size 8-14 Fr
(c) Oxygen by face mask
(d) Mechanical ventilation
25.
Which one of the following statements regarding Felon is NOT correct?

(a) There is infection of the finger tip between specialised fibrous septa
(b) It is a painless condition
(c) Incision and drainage is the treatment of choice
(d) It is common in diabetics
26.
The most common site for osteosarcoma is:

(a) Proximal femur
(b) Distal femur
(c) Proximal humerus
(d) Distal humerus
27.
Rapid Sequence Induction is indicated in:

(a) Emergency surgery for intestinal obstruction
(b) Elective open hernia surgery
(c) Cardiopulmonary bypass surgery
(d) Elective laparoscopic surgery

28.
Which one of the following statements about Compartment Syndrome is NOT correct?

(a) It is commonest in a closed fracture
(b) Pain is on active movement but not on passive movement of muscles
(c) Fasciotomy is the treatment of choice
(d) Volkmann?s Ischaemic contractive is a late complication




29.
Which one of the following is NOT the strength of ultrasound as a diagnostic modality?

(a) No radiation
(b) Short learning curve
(c) Inexpensive
(d) Allows dynamic studies to be done
30.
Which of the following statements regarding lymphoedema following breast cancer treatment
are correct?

1. Incidence has decreased due to rarely combined therapy of axillary LN dissection and
radiotherapy
2. Precipitating cause like LN metastasis is a major determinant
3. The condition is often painful
4. Oedematous limb is susceptible to bacterial infection

Select the correct answer using the code given below:

(a) 1, 2 and 3
(b) 2, 3 and 4
(c) 1, 3 and 4
(d) 1, 2 and 4
31.
Which one of the following is NOT a risk factor for development of venous thrombosis in
surgical patients?

(a) Age > 60 years
(b) Pregnancy
(c) Obesity (BMI > 30 kg/m
2
)
(d) Diabetes ( HbA1c > 7.5%)
32.
Which one of the following is NOT a complication of massive blood transfusion?

(a) Coagulopathy
(b) Hypocalcaemia
(c) Hyperthermia
(d) Hyperkalemia
33.
Stage III ?Pressure sore? is full thickness skin loss extending:

(a) into subcutaneous tissue but not through fascia
(b) through subcutaneous tissue into fascia
(c) through subcutaneous tissue into fascia and muscles
(d) through subcutaneous tissue into fascia, muscles and bone



34.
During subclavian vein puncture in a surgical ward suddenly a patient developed severe
breathlessness. On auscultation breath sound was absent and the ipsilateral chest was
tympanitic on percussion. The probable diagnosis is:

(a) Iatrogenic pneumothorax
(b) Introgenic hemothorax
(c) Spontaneous pneumothorax
(d) Tension pneumothorax
35.
Kohler?s disease is avascular necrosis of :

(a) Lunate
(b) Capitellum of humerus
(c) First metatarsal head
(d) Navicular
36.
Which one of the following is NOT true of Pyoderma gangrenosum?

(a) It is characterized by cutaneous ulceration with purple undermined edges
(b) It is often secondary to heightened immunological reactivity from another disease
process
(c) Cultures often show Gram positive Staphylococci
(d) Lesions generally respond to steroids
37.
Which of the following is NOT a tissue repair surgery for inguinal hernia repair?

(a) Bassini?s repair
(b) Shouldice repair
(c) Stoppa?s repair
(d) Desarda repair
38.
Which one of the following type of meshes is recommended for intraperitoneal use in
abdominal wall hernia?
(a) Light weight, porous meshes
(b) Heavy weight, porous meshes
(c) Absorbable meshes
(d) Tissue separating meshes
39.
Which one of the following is NOT correct regarding MEN-1 syndrome?

(a) It involves parathyroid glands
(b) It involves pancreas
(c) It involves pituitary gland
(d) It involves pineal gland

40.
Which one of the following is NOT an electronic information site in surgery?

(a) Pubmed
(b) Embase
(c) Cochrane library
(d) National medical library
41.
Which one of the following statements regarding pre-conceptional counseling is NOT
correct?

(a) It is needed only in selected complicated pregnancies
(b) It helps in early detection of risk factors
(c) It helps in reducing maternal morbidity and mortality
(d) It is a part of preventive medicine
42.
Consider the following statements regarding Non Stress Test (NST):

1. Reactive NST indicates a healthy fetus
2. NST is an observed association of fetal breathing with fetal movements
3. NST has a low false negative rate (< 1%) but high false positive rate (>50%)
4. Testing should be started at 20 weeks

Which of the statement(s) given above is/are correct?

(a) 1 and 3
(b) 2 only
(c) 3 only
(d) 1 and 4
43.
Which one of the following is a protective factor for endometrial hyperplasia?

(a) Diabetes
(b) Tamoxifen therapy
(c) Multiparity
(d) Delayed menopause
44.
A woman who is not breast feeding her newborn child is advised to use a contraceptive
method by:

(a) 3
rd
postpartum week
(b) 6
th
postpartum week
(c) 3
rd
postpartum month
(d) 6
th
postpartum month





45.
Pearl index for contraceptive effectiveness is calculated in terms of which of the following?

1. Pregnancy rate
2. Abortion rate
3. Hundred woman years
4. Thousand woman years

Select the correct answer using the code given below:

(a) 1 only
(b) 2 and 3
(c) 1, 2 and 4
(d) 1 and 3
46.
Indications for removal of IUDs are all EXCEPT:

(a) Perforation of uterus
(b) Cyclical menstrual bleeding
(c) Flaring up of salpingitis
(d) Pregnancy with IUD
47.
Contraindications for insertion of IUDs are all EXCEPT:

(a) Suspected pregnancy
(b) Trophoblastic disease
(c) Severe dysmenorrhea
(d) During cesarean section
48.
Which one of the following is NOT a contraindication for use of Mini pill?

(a) Pregnancy
(b) Breast feeding
(c) Thromboembolic disease
(d) History of breast cancer
49.
Which one of the following is the most commonly used surgical method/technique of female
sterilization as recommended by Government of India?

(a) Uchida technique
(b) Irving method
(c) Pomeroy?s method
(d) Madlener technique







50.
Which of the following is/are required for a registered medical practitioner to qualify for
performing Medical Termination of Pregnancy (MTP), as per revised rules of MTP Act?

1. Certified for assisting at least 15 MTP in an authorized centre
2. Diploma or degree in Obstetrics and Gynaecology
3. House surgeon training for 3 months in Obstetrics and Gynaecology
4. Certified training for 6 months in laparoscopic surgeries

Select the correct answer using the code given below:
(a) 1 only
(b) 2 only
(c) 1, 2 and 3
(d) 1, 2 and 4
51.
Which one of the following is NOT a support of uterus, preventing its descent?

(a) Endopelvic fascia
(b) Mackenrodt?s ligament
(c) Inguinal ligament
(d) Pubocervical ligament
52.
As per ICMR guidelines, which one of the following statements is true regarding effects of
COVID-19 on fetus according to current evidence?

(a) There is increased risk of early pregnancy loss
(b) COVID-19 virus is not teratogenic
(c) COVID-19 virus infection is an indication of MTP
(d) There is increased risk of fetal growth restriction
53.
As per ICMR guidelines, which one of the following statements is true regarding COVID-19
infection in pregnancy?

(a) Covid-19 pneumonia in pregnancy is more severe with poor recovery
(b) Pregnant women with heart disease are at higher risk
(c) Vaginal secretions always test positive for COVID-19 in pregnancy
(d) COVID-19 virus is secreted in breast milk
54.
Which one of the following is NOT a method of management of Deep Transverse Arrest with
the living fetus?

(a) Caesarean section
(b) Delivery by ventouse
(c) Delivery by application of forceps to the unrotated head
(d) Manual rotation and application of forceps





55.
Successful version of breech presentation is likely in case all of the following EXCEPT:

(a) Breech with extended legs
(b) Complete breech with sacroanterior position
(c) Non engaged breech
(d) Adequate amniotic fluid
56.
Implantation of a fertilised ovum occurs on which day following fertilisation?

(a) Day 6
(b) Day 10
(c) Day 14
(d) Day 20
57.
During total abdominal hysterectomy the ureter is likely to undergo injury or ligation during
the following steps EXCEPT:

(a) During division and ligation of the round ligaments
(b) During division and ligation of infundibulopelvic ligaments
(c) During division and ligation of mackenrodt?s and uterosacral ligaments
(d) At the vaginal angles while incising the vagina to remove the cervix with the
uterus
58.
The net effect of antenatal care has been the following EXCEPT:

(a) Reduction in maternal mortality
(b) Reduction in perinatal mortality
(c) Reduction in the incidence of institutional delivery
(d) Reduction in maternal morbidity
59.
Which one of the following is NOT a component of active phase in the partograph?

(a) Acceleration phase
(b) Phase of maximum slope
(c) Phase of deceleration
(d) Phase of expulsion
60.
From medicolegal point of view which one of the following is NOT a sign of previous child
birth?

(a) Perineum is lax and there is evidence of scarring
(b) Introitus is gaping and there is presence of carunculae myrtiformis
(c) Abdomen is lax and loose with striae and linea alba
(d) Conical cervix with round external os





61.
The components of partograph are all EXCEPT:

(a) Time
(b) Fetal heart rate
(c) Maternal respiratory rate
(d) Maternal urine analysis
62.
Which of the following information are provided by partograph?

1. Colour of liquor
2. Uterine contractions with duration and frequency
3. Dilatation of cervix

Select the correct answer using the code given below:

(a) 1 and 2 only
(b) 2 and 3 only
(c) 1 and 3 only
(d) 1, 2 and 3
63.
Intraoperative recognition of ureter is by which of the following features?

1. Transparent tubular appearance
2. Pale glistening appearance
3. Longitudinal vessels on surface
4. Circumferential vessels on surface

Select the correct answer using the code given below:

(a) 1 and 3
(b) 2 and 4
(c) 2 and 3
(d) 1 and 4
64.
Hysterosalpingography (HSG) is least helpful in detecting which of the following?

(a) Tubal patency
(b) Pelvic adhesions
(c) Asherman syndrome
(d) Congenital uterine anomaly









65.
Which of the following are characteristics of Trichomonas vaginitis?

1. Presence of greenish frothy discharge
2. Vaginal pH > 4.5
3. Presence of clue cells in microscopic examination
4. Strawberry spots on the vaginal mucosa

Select the correct answer using the code given below:

(a) 1, 2 and 3
(b) 1, 2 and 4
(c) 2, 3 and 4
(d) 1, 3 and 4
66.
Tumor marker of epithelial ovarian carcinoma is:

(a) Ca.125
(b) Alpha feto protein
(c) Beta HCG
(d) LDH
67.
The most common site of cervical cancer is:

(a) Endocervix
(b) Ectocervix
(c) Transformation zone
(d) Isthmus
68.
The placenta synthesizes all EXCEPT:

(a) Oestriol
(b) Corticotrophin releasing hormone
(c) PAPP-A(Pregnancy Associated Plasma Protein A)
(d) Dehydroepiandrosterone
69.
Withdrawal bleeding following administration of progesterone in a case of secondary
amenorrhea indicates all EXCEPT:

(a) Absence of pregnancy
(b) Production endogenous estrogen
(c) Endometrium is responsive to estrogen
(d) Defect in pituitary gland
70.
Monilial vaginitis is commonly associated with all EXCEPT:

(a) Prolonged antibiotic therapy
(b) Diabetes Mellitus
(c) Treatment of malaria with chloroquine
(d) Pregnancy
71.
Which one of the following is NOT a risk factor for the development of placenta previa?

(a) Maternal age
(b) Smoking
(c) Previous caesarean section
(d) Maternal anaemia
72.
Common clinical presentations of moderate to severe abruption are all EXCEPT:

(a) Uterine tenderness
(b) Fetal distress
(c) Unexplained pre term labour
(d) Prolonged labour
73.
Common trisomies resulting in spontaneous abortion are all EXCEPT:

(a) Trisomy 21
(b) Trisomy 18
(c) Trisomy 16
(d) Trisomy 1
74.
The initial prevention strategy for antiphospholipid syndrome will be:

1. Steroids
2. Heparin
3. Low dose aspirin
4. Progesterone support

Which of the above is/are correct?

(a) 2 and 3
(b) 3 and 4
(c) 3 only
(d) 1 and 4
75.
Diagnostic criteria for PCOD are:

1. Oligo/amenorrohoea
2. Hyperandrogenism
3. Polycystic ovaries on ultrasound

Which of the above are correct?

(a) 1 and 2 only
(b) 2 and 3 only
(c) 1 and 3 only
(d) 1, 2 and 3


76.
Which of the following symptoms can be associated with pelvic organ prolapse?

1. Difficulty in passing urine
2. Incomplete evacuation of urine
3. Urgency and frequency

Select the correct answer using the code given below:

(a) 1 and 2 only
(b) 2 and 3 only
(c) 1 and 3 only
(d) 1, 2 and 3
77.
A 30 year old lady, P
2
L
2
presents with painful unilateral swelling in vulva for 3 days. Which
of the following statements are true regarding the above case?

1. Bartholin?s abscess may be the likely diagnosis
2. It is to be managed by marsupialisation
3. Gonococcus is the most common pathogenic organism

Select the correct answer using the code given below:
(a) 1 and 3 only
(b) 3 only
(c) 1 and 2 only
(d) 1, 2 and 3
78.
Which one of the following is NOT a sign of separation of placenta?

(a) Uterus becomes globular, firm and ballotable
(b) The fundal height reduces further
(c) Slight bulging in the suprapubic region
(d) Apparent lengthenic of the cord with slight gush of vaginal bleeding
79.
Consider the following regarding examination of a rape victim:

1. Emergency pill is provided
2. Internal examination must be performed
3. HIV testing is done

Which of the above statements is/are correct?

(a) 1 and 3 only
(b) 2 only
(c) 1, 2 and 3
(d) 3 only




80.
Consider the following cardinal movements of mechanism of normal labor:
1. Engagement
2. Internal rotation
3. Flexion
4. Restitution
5. Crowning
6. External rotation

What is the correct sequence of movements in labor in occipito-lateral position?

(a) 1, 2, 3, 4, 5 and 6
(b) 1, 3, 2, 5, 4 and 6
(c) 2, 1, 3, 4, 5 and 6
(d) 3, 1, 2, 4, 6 and 5
81.
Which of the following represent the properties of an ideal disinfectant?

1. It is broad spectrum
2. It is fast acting
3. It is non-toxic

Select the correct answer using the code given below:
(a) 1 and 2 only
(b) 2 and 3 only
(c) 1 and 3 only
(d) 1, 2 and 3
82.
Which of the following measures can help reduce the risk of systemic hypertension?

1. Reduction in dietary intake of common salt
2. Controlling weight for age
3. Increasing potassium rich foods in the diet

Select the correct answer using the code given below:

(a) 1 and 2 only
(b) 2 and 3 only
(c) 1 and 3 only
(d) 1, 2 and 3








83.
Keeping biological determinants in perspective, consider the following statements:

1. Presence of a normal karyotype is the first requisite for human health
2. Genetic screening can play an important role in prevention of wide spectrum of
diseases
3. If an individual is allowed to live in healthy relationship with the environment, the
person?s genetic potentialities can transform into phenotypic realities

Which of the above statements is/are correct?

(a) 1 and 2 only
(b) 1 and 3 only
(c) 1, 2 and 3
(d) 2 and 3 only
84.
NITI Aayog has the following roles EXCEPT:

(a) It provides critical directional and strategic input in the development process
(b) It provides relevant technical advice with focus on technology upgradation
(c) It focuses on capacity building
(d) It focuses on the development of ?Referral Service Complex?
85.
Poor hand hygiene of a mess worker in a university college mess led to Hepatitis A cases in
the hostel inmates. What type of epidemic will this exposure present with?

1. Propagated
2. Common source-continuous exposure
3. Common source-point exposure

Select the correct answer using the code given below:

(a) 1 and 2
(b) 1 and 3
(c) 1 only
(d) 2 only












86.
What is the specificity of sputum microscopy in detection of Pulmonary Tuberculosis (PTB)
as per the information given below?

PTB Total
Sputum microscopy Present Absent
Positive 270 20 290
Negative 30 180 210
Total 300 200 500

(a) 10 %
(b) 36 %
(c) 90 %
(d) 94 %
87.
In a cohort of 500 women attending antenatal clinic, 70 % had ultrasonography (USG). This
cohort was followed up at delivery. Of the women who had USG, 70 delivered low birth
weight (LBW) babies; whereas of the women, who did not undergo USG, 50 delivered LBW
babies. The incidence of LBW babies among women who had USG is:

(a) 10 %
(b) 15 %
(c) 20 %
(d) 25 %
88.
Major source of vitamin K
1
is:

(a) Fresh dark green vegetables
(b) Exposure of body to sunlight
(c) Citrus fruits
(d) Foods rich in polyunsaturated fatty acids
89.
Which one of the following is NOT a function of Epidemiology?

(a) To study historically the rise and fall of disease in the population
(b) Searching for the causes and risk factor
(c) Identifying syndromes
(d) Making clinical diagnosis
90.
Which one of the following is NOT a contagious disease?

(a) Malaria
(b) Scabies
(c) Trachoma
(d) Leprosy


91.
In primary immune response, how much more antigenic dose is required to induce IgG
antibodies as against the induction of IgM antibodies?

(a) 10 times more
(b) 25 times more
(c) 50 times more
(d) 100 times more
92.
Pentavalent vaccine provides protection against which of the following diseases?

(a) Diphtheria, Pertussis, Tuberculosis, Measles and Hepatitis B
(b) Diphtheria, Pertussis, Measles, Hepatitis B and Hib
(c) Diphtheria, Pertussis, Tetanus, Hepatitis B and Rubella
(d) Diphtheria, Pertussis, Tetanus, Hepatitis B and Hib
93.
Consider the following data for a country:
Population in 0-14 years of age ? 391,558,367
Population between 15?64 years of age ? 856,076,200
Population above 65 years of age ? 71,943,390
What shall be the dependency ratio of this country?

(a) 42.4 %
(b) 54.1 %
(c) 66.2 %
(d) 78.6 %
94.
Which one of the following statements regarding Rabies Immunoglobulin is NOT true?

(a) It should be administered only once as soon as possible after the initiation of post
exposure prophylaxis
(b) It should be administered all into or around the wound sites
(c) There is no scientific ground for performing a skin test prior to administering equine
immunoglobulin
(d) It can be administered till 15 days after the first dose of anti-rabies vaccine
95.
All of the following are true about Bedaquiline (BDQ) EXCEPT:

(a) It specifically targets mycobacterial ATP synthase
(b) It is a bacteriostatic drug
(c) It has extended half life
(d) It has high volume of tissue distribution
96.
Visual inspection based screening test with 5 % acetic acid is used for the screening of which
one of the following cancers?

(a) Lung cancer
(b) Cervix cancer
(c) Oral cancer
(d) Breast cancer
97.
Predictive accuracy of a screening test depends on the following EXCEPT:

(a) Disease prevalence
(b) Disease incidence
(c) Sensitivity of screening test
(d) Specificity of screening test
98.
How much of Zinc supplement is recommended by WHO and UNICEF for infants less than 6
months of age after an episode of acute diarrhoea?

(a) 20 mg per day for 10?14 days
(b) 10 mg per day for 10?14 days
(c) 5 mg per day for 7 days
(d) 6 mg per day for 7 days
99.
Which one of the following statements regarding sequential administration of Inactivated
Polio Vaccine (IPV) and Oral Polio Vaccine (OPV) is NOT correct?

(a) It will be cost effective in developing countries for Polio prevention
(b) The combined schedules of IPV and OPV appear to reduce or prevent Vaccine
Associated Paralytic Polio (VAPP)
(c) Intestinal mucosal immunity is lost due to IPV administration
(d) IPV and OPV together may optimize both the humoral and mucosal immunogenicity
of Polio vaccine
100.
What is the recommended dose regimen of Vitamin A for the treatment of early stages of
Xerophthalmia?

(a) Single massive dose of 2 lac International Units (IU)
(b) 2 lac IU on two successive days
(c) 2 doses of 1 lac IU in two successive days
(d) 2 doses of 1 lac IU at a gap of one week
101.
Which is the most specific causative agent of Rabies?

(a) Lyssavirus serotype 1
(b) Lyssavirus serotype 2
(c) Lyssavirus serotype 3
(d) Lyssavirus serotype 4
102.
Which one of the following is the antibiotic of choice for the prevention of Rheumatic heart
disease?

(a) Benzathine Benzyl Penicillin
(b) Procaine Penicillin
(c) Doxycycline
(d) Ciprofloxacin


103.
All of the following are global targets for WHO Global Action Plan (2013?2020) for
Prevention and Control of NCDs, EXCEPT:

(a) A 10 % relative reduction in risk of premature mortality from cardiovascular diseases,
cancer, diabetes and chronic respiratory diseases
(b) A 10 % relative reduction in prevalence of insufficient physical activity
(c) A 10 % relative reduction in mean population intake of salt/sodium
(d) At least 10 % relative reduction in the harmful use of alcohol
104.
Under the Employees State Insurance (ESI) Scheme, extended sickness benefit is provided in
which of the following infectious diseases?

1. Tuberculosis
2. Leprosy
3. Chronic empyema

Select the correct answer using the code given below:

(a) 1 and 2 only
(b) 2 and 3 only
(c) 1 and 3 only
(d) 1, 2 and 3

105.
In the context of NCD prevention and control in India, the extent of relative reduction in
household use of solid fuels as a primary source of energy for cooking by 2025 is targeted at:

(a) 30 %
(b) 40 %
(c) 50 %
(d) 60 %
106.
Consider the following criteria which may indicate elimination of lymphatic filariasis in a
community:

1. When lymphatic filariasis ceases to be a public health problem in the community
2. When the number of microfilaria carriers declines to 1.5 % within the community
3. When children born in the community after the initiation of elimination programme
are free from circulating antigenaemia

Which of the criteria stated above hold true?

(a) 1 and 2 only
(b) 2 and 3 only
(c) 1 and 3 only
(d) 1, 2 and 3



107.
Hold over time of cold chain equipment depends on all of the following factors EXCEPT:

(a) Ambient temperature
(b) Quantity of vaccines kept
(c) Types of vaccines kept
(d) Condition of icepack lining
108.
Which of the following statements regarding Physical Quality of Life Index (PQLI) are
correct?

1. It consolidates infant mortality, life expectancy at age one, and literacy
2. It does not measure economic growth but measures the result of economic policies
3. For each component, the performance of individual countries is placed on a scale of
0-100

Select the correct answer using the code given below:

(a) 1 and 2 only
(b) 2 and 3 only
(c) 1 and 3 only
(d) 1, 2 and 3
109.
Disability-adjusted life years (DALYs) include:

(a) Years of lost life (YLL)
(b) Years lost to disability (YLD)
(c) Both YLL and YLD
(d) Neither YLL nor YLD
110.
Which of the following are health care delivery indicators?

1. Population per trained birth attendant
2. Population per health/sub centre
3. Doctor-nurse ratio

Select the correct answer using the code given below:

(a) 1 and 2 only
(b) 2 and 3 only
(c) 1 and 3 only
(d) 1, 2 and 3








111.
Which of the following key action areas are incorporated in the Ottawa Charter?

1. Building healthy public policy
2. Strengthening community action for health
3. Reorienting health services

Select the correct answer using the code given below:

(a) 1 and 2 only
(b) 2 and 3 only
(c) 1 and 3 only
(d) 1, 2 and 3
112.
Under Chapter XXI of the 10
th
revision of the International Statistical Classification of
Diseases and Related Health Problems (ICD-10), lifestyle-related problems fall under which
of the following code range?

(a) U 50.0 ? U 50.5
(b) U 10.0 ? U 10.5
(c) Z 72.0 ? Z 72.5
(d) Z 10.0 ? Z 10.5
113.
Which of the following diseases are spread by airborne transmission?

1. Influenza
2. Chicken pox
3. Q-fever
4. Psittacosis

Select the correct answer using the code given below:

(a) 1 and 3 only
(b) 1 and 4 only
(c) 3 and 4 only
(d) 1, 2, 3 and 4
114.
DASH diet plan is advocated in the control of which one of the following diseases?

(a) Cancer
(b) Chronic Obstructive Pulmonary Disease
(c) Hypertension
(d) Rheumatic heart disease
115.
Endemic ascitis occurs due to:

(a) Naturally occurring toxin
(b) Bacterial toxin
(c) Fungal toxin
(d) Viral toxin
116.
The presence of which of the following bacteria in drinking water is an important
confirmatory evidence of recent faecal pollution of water?

(a) E. coli
(b) Streptococci
(c) Clostridium
(d) Klebsiella
117.
With regard to the Goals, Milestones and Targets for the Global Technical Strategy for
Malaria (2016-2030), consider the following statements:

1. Compared to 2015, the target is to reduce the malaria mortality rates globally by at
least 90% by year 2030
2. Compared to 2015, the target is to reduce malaria case incidence by 90% by year
2030
3. Eliminate malaria from at least 35 countries, where malaria was transmitted in
2015, by year 2030

Which of the above statements hold true?

(a) 1 and 2 only
(b) 2 and 3 only
(c) 1 and 3 only
(d) 1, 2 and 3
118.
Which one of the following IUDs is associated with a low pregnancy rate (0.2 per 100), less
number of ectopic pregnancies and lower menstrual blood loss?

(a) Cu T-380A
(b) ML-Cu 375
(c) Progestasert
(d) LNG-20(Mirena)
119.
The incidence of diarrhoea is highest among infants in the age group of 6?11 months due to
all of the following reasons EXCEPT:

(a) Declining level of maternal antibodies
(b) Introduction of foods which may be contaminated
(c) Eruption of teeth
(d) Direct contact with human or animal faeces









FirstRanker.com - FirstRanker's Choice
Combined Medical Services Examination-2020
Paper-II

1.
Indications for fasciotomy in compartment syndrome include all EXCEPT:

(a) Distal sensory disturbance
(b) Compartment pressure > 30 mm Hg
(c) Pain on passive movement of affected muscles
(d) Palpable distal pulses
2.
Which one of the following statements is NOT correct regarding Necrotising Soft Tissue
infections?

(a) Crepitus, skin blistering and focal skin gangrene are typical presenting features
(b) They are monomicrobial in nature
(c) Treatment consists of wide local excision and appropriate antibiotics
(d) Tissue biopsy is required for culture and diagnosis
3.
Which type of surgery is laparoscopic cholecystectomy classified as?

(a) Clean
(b) Clean contaminated
(c) Contaminated
(d) Dirty
4.
A 22-year female has presented with a history of malaise, cough, alternating constipation and
diarrhoea with intermittent abdominal pain for last 6 months. She also complains of
abdominal distension for last 2 days. On examination her abdomen has a doughy feel along
with an ill defined mass over the right lower quadrant. She is most likely suffering from:

(a) Appendicular lump
(b) Ileocaecal tuberculosis
(c) Carcinoma caecum
(d) Ovarian mass
5.
Consider the following statements regarding needle stick injuries:

1. Injured part should be washed under running water
2. Dominant index finger is the commonest site for needle stick injury
3. All needle stick injuries should be reported
4. Hepatitis/HIV testing should be done after needle stick injury

Which of the statements given above are correct?

(a) 1, 2 and 4
(b) 1, 2 and 3
(c) 1, 3 and 4
(d) 2, 3 and 4




6.
Consider the following statements regarding claudication:

1. It is a marker for silent coronary disease
2. Structured exercise program ( 2 hours per week for 3 months) leads to
improvement in symptoms
3. Diabetes mellitus increases the risk and severity of claudication
4. Beta blockers may exacerbate claudication

Which of the above statements are correct?

(a) 1 and 2 only
(b) 1, 3 and 4 only
(c) 2, 3 and 4 only
(d) 1, 2, 3 and 4
7.
A 50-year old lady underwent uneventful bariatric surgery for morbid obesity. On the third
post operative day, she develops breathless and pulmonary embolism is suspected. The next
investigation to confirm the diagnosis will be:

(a) Echocardiography
(b) Duplex venography
(c) CT pulmonary angiography
(d) MR angiography
8.
Medical management of thyrotoxic crisis includes all of the following EXCEPT:

(a) IV fluids
(b) IV propanolol
(c) IV hydrocortisone
(d) IV antibiotics
9.
A patient operated for a parotid gland tumour developed symptoms of sweating and
erytherma (flushing) over the region of surgical excision while eating. The probable
diagnosis is:

(a) Parotid gland fistula
(b) Sialadenitis
(c) Chronic wound infection
(d) Frey?s syndrome
10.
First line hormone therapy for post-menopausal woman with metastatic carcinoma breast is:

(a) Tamoxifen
(b) Ovarian suppression by surgery
(c) Antiprogestins
(d) Anastrazole



11.
All of the following are major subtypes of breast cancer based on Gene array analysis
EXCEPT:

(a) Luminal A and Luminal B
(b) Triple negative
(c) Her-2 receptor positive
(d) Oestrogen receptor positive
12.
All of the following are sequelae of peptic ulcer surgery EXCEPT:

(a) Bilious vomiting
(b) Dumping syndrome
(c) Diarrhoea
(d) Increased appetite
13.
The Child-Turcotte-Pugh (CTP) score for quantifying the severity of chronic liver disease
includes all variables EXCEPT:

(a) Serum bilirubin
(b) Serum albumin
(c) Serum creatinine
(d) INR (International Normalised Ratio)
14.
?Chain of Lakes? appearance due to sacculation with intervening short strictures of pancreatic
duct is seen on:

(a) ERCP
(b) CECT abdomen
(c) Plain X-ray abdomen
(d) Ultrasonography
15.
Which one of the following statements is NOT correct regarding Pyogenic Liver Abscess?

(a) Anorexia, fever, malaise and right upper quadrant abdominal discomfort are the
most common presenting features
(b) It is more common in elderly, diabetics and immunocompromised patients
(c) Treatment is with oral antibiotics alone
(d) Streptococcus milleri and escherichia coli are the most common causative
organisms
16.
?Swiss cheese defects? are seen during laparoscopic repair of:

(a) Ventral hernia
(b) Inguinal hernia
(c) Obturator hernia
(d) Femoral hernia



17.
Which of the following are correct regarding splenic artery aneurysm?

1. Main arterial trunk is the common site
2. Palpable thrill can be felt
3. It is symptomless unless it ruptures

Select the correct answer using the code given below:

(a) 1 and 2 only
(b) 2 and 3 only
(c) 1 and 3 only
(d) 1, 2 and 3
18.
Valentino?s syndrome is:

(a) Pain on per-vaginal examination in pelvic abscess
(b) Pain over left shoulder in left hypochondriac collection
(c) Pain over left groin in perirenal collection
(d) Pain in right iliac fossa in perforated peptic ulcer
19.
Spontaneous bacterial peritonitis occurs due to:

(a) duodenal stump blowout
(b) peptic ulcer perforation
(c) acute bacterial infection of ascites
(d) infection via fallopian tubes
20.
Structure not forming boundaries of the ?Triangle of doom? seen during laparoscopic
inguinal hernia surgery dissection is:

(a) Vas deferens
(b) Inferior epigastric artery
(c) Spermatic cord vessels
(d) Peritoneum
21.
The term mid-line shift is associated with:

(a) Head injury
(b) Chest injury
(c) Abdominal injury
(d) Limb injury
22.
Which one of the following cranial nerves does NOT supply to the external ear?

(a) Cranial nerve V
(b) Cranial nerve VI
(c) Cranial nerve VII
(d) Cranial nerve IX

23.
Left Internal Mammary Artery (LIMA) has become the conduit of choice for Left Anterior
Descending (LAD) artery during coronary artery bypass grafting because:

(a) Long term patency rates are more than 98%
(b) It is close to LAD
(c) Atherosclerosis is never seen in this vessel
(d) It is very easy to harvest
24.
A 50-year old male with significant smoking history presented in the surgical emergency
with sudden severe breathlessness. Chest X-ray shows right sided Pneumothorax. The
appropriate management requires:

(a) Aspiration of air with 16-18 G cannula
(b) Right chest drain of size 8-14 Fr
(c) Oxygen by face mask
(d) Mechanical ventilation
25.
Which one of the following statements regarding Felon is NOT correct?

(a) There is infection of the finger tip between specialised fibrous septa
(b) It is a painless condition
(c) Incision and drainage is the treatment of choice
(d) It is common in diabetics
26.
The most common site for osteosarcoma is:

(a) Proximal femur
(b) Distal femur
(c) Proximal humerus
(d) Distal humerus
27.
Rapid Sequence Induction is indicated in:

(a) Emergency surgery for intestinal obstruction
(b) Elective open hernia surgery
(c) Cardiopulmonary bypass surgery
(d) Elective laparoscopic surgery

28.
Which one of the following statements about Compartment Syndrome is NOT correct?

(a) It is commonest in a closed fracture
(b) Pain is on active movement but not on passive movement of muscles
(c) Fasciotomy is the treatment of choice
(d) Volkmann?s Ischaemic contractive is a late complication




29.
Which one of the following is NOT the strength of ultrasound as a diagnostic modality?

(a) No radiation
(b) Short learning curve
(c) Inexpensive
(d) Allows dynamic studies to be done
30.
Which of the following statements regarding lymphoedema following breast cancer treatment
are correct?

1. Incidence has decreased due to rarely combined therapy of axillary LN dissection and
radiotherapy
2. Precipitating cause like LN metastasis is a major determinant
3. The condition is often painful
4. Oedematous limb is susceptible to bacterial infection

Select the correct answer using the code given below:

(a) 1, 2 and 3
(b) 2, 3 and 4
(c) 1, 3 and 4
(d) 1, 2 and 4
31.
Which one of the following is NOT a risk factor for development of venous thrombosis in
surgical patients?

(a) Age > 60 years
(b) Pregnancy
(c) Obesity (BMI > 30 kg/m
2
)
(d) Diabetes ( HbA1c > 7.5%)
32.
Which one of the following is NOT a complication of massive blood transfusion?

(a) Coagulopathy
(b) Hypocalcaemia
(c) Hyperthermia
(d) Hyperkalemia
33.
Stage III ?Pressure sore? is full thickness skin loss extending:

(a) into subcutaneous tissue but not through fascia
(b) through subcutaneous tissue into fascia
(c) through subcutaneous tissue into fascia and muscles
(d) through subcutaneous tissue into fascia, muscles and bone



34.
During subclavian vein puncture in a surgical ward suddenly a patient developed severe
breathlessness. On auscultation breath sound was absent and the ipsilateral chest was
tympanitic on percussion. The probable diagnosis is:

(a) Iatrogenic pneumothorax
(b) Introgenic hemothorax
(c) Spontaneous pneumothorax
(d) Tension pneumothorax
35.
Kohler?s disease is avascular necrosis of :

(a) Lunate
(b) Capitellum of humerus
(c) First metatarsal head
(d) Navicular
36.
Which one of the following is NOT true of Pyoderma gangrenosum?

(a) It is characterized by cutaneous ulceration with purple undermined edges
(b) It is often secondary to heightened immunological reactivity from another disease
process
(c) Cultures often show Gram positive Staphylococci
(d) Lesions generally respond to steroids
37.
Which of the following is NOT a tissue repair surgery for inguinal hernia repair?

(a) Bassini?s repair
(b) Shouldice repair
(c) Stoppa?s repair
(d) Desarda repair
38.
Which one of the following type of meshes is recommended for intraperitoneal use in
abdominal wall hernia?
(a) Light weight, porous meshes
(b) Heavy weight, porous meshes
(c) Absorbable meshes
(d) Tissue separating meshes
39.
Which one of the following is NOT correct regarding MEN-1 syndrome?

(a) It involves parathyroid glands
(b) It involves pancreas
(c) It involves pituitary gland
(d) It involves pineal gland

40.
Which one of the following is NOT an electronic information site in surgery?

(a) Pubmed
(b) Embase
(c) Cochrane library
(d) National medical library
41.
Which one of the following statements regarding pre-conceptional counseling is NOT
correct?

(a) It is needed only in selected complicated pregnancies
(b) It helps in early detection of risk factors
(c) It helps in reducing maternal morbidity and mortality
(d) It is a part of preventive medicine
42.
Consider the following statements regarding Non Stress Test (NST):

1. Reactive NST indicates a healthy fetus
2. NST is an observed association of fetal breathing with fetal movements
3. NST has a low false negative rate (< 1%) but high false positive rate (>50%)
4. Testing should be started at 20 weeks

Which of the statement(s) given above is/are correct?

(a) 1 and 3
(b) 2 only
(c) 3 only
(d) 1 and 4
43.
Which one of the following is a protective factor for endometrial hyperplasia?

(a) Diabetes
(b) Tamoxifen therapy
(c) Multiparity
(d) Delayed menopause
44.
A woman who is not breast feeding her newborn child is advised to use a contraceptive
method by:

(a) 3
rd
postpartum week
(b) 6
th
postpartum week
(c) 3
rd
postpartum month
(d) 6
th
postpartum month





45.
Pearl index for contraceptive effectiveness is calculated in terms of which of the following?

1. Pregnancy rate
2. Abortion rate
3. Hundred woman years
4. Thousand woman years

Select the correct answer using the code given below:

(a) 1 only
(b) 2 and 3
(c) 1, 2 and 4
(d) 1 and 3
46.
Indications for removal of IUDs are all EXCEPT:

(a) Perforation of uterus
(b) Cyclical menstrual bleeding
(c) Flaring up of salpingitis
(d) Pregnancy with IUD
47.
Contraindications for insertion of IUDs are all EXCEPT:

(a) Suspected pregnancy
(b) Trophoblastic disease
(c) Severe dysmenorrhea
(d) During cesarean section
48.
Which one of the following is NOT a contraindication for use of Mini pill?

(a) Pregnancy
(b) Breast feeding
(c) Thromboembolic disease
(d) History of breast cancer
49.
Which one of the following is the most commonly used surgical method/technique of female
sterilization as recommended by Government of India?

(a) Uchida technique
(b) Irving method
(c) Pomeroy?s method
(d) Madlener technique







50.
Which of the following is/are required for a registered medical practitioner to qualify for
performing Medical Termination of Pregnancy (MTP), as per revised rules of MTP Act?

1. Certified for assisting at least 15 MTP in an authorized centre
2. Diploma or degree in Obstetrics and Gynaecology
3. House surgeon training for 3 months in Obstetrics and Gynaecology
4. Certified training for 6 months in laparoscopic surgeries

Select the correct answer using the code given below:
(a) 1 only
(b) 2 only
(c) 1, 2 and 3
(d) 1, 2 and 4
51.
Which one of the following is NOT a support of uterus, preventing its descent?

(a) Endopelvic fascia
(b) Mackenrodt?s ligament
(c) Inguinal ligament
(d) Pubocervical ligament
52.
As per ICMR guidelines, which one of the following statements is true regarding effects of
COVID-19 on fetus according to current evidence?

(a) There is increased risk of early pregnancy loss
(b) COVID-19 virus is not teratogenic
(c) COVID-19 virus infection is an indication of MTP
(d) There is increased risk of fetal growth restriction
53.
As per ICMR guidelines, which one of the following statements is true regarding COVID-19
infection in pregnancy?

(a) Covid-19 pneumonia in pregnancy is more severe with poor recovery
(b) Pregnant women with heart disease are at higher risk
(c) Vaginal secretions always test positive for COVID-19 in pregnancy
(d) COVID-19 virus is secreted in breast milk
54.
Which one of the following is NOT a method of management of Deep Transverse Arrest with
the living fetus?

(a) Caesarean section
(b) Delivery by ventouse
(c) Delivery by application of forceps to the unrotated head
(d) Manual rotation and application of forceps





55.
Successful version of breech presentation is likely in case all of the following EXCEPT:

(a) Breech with extended legs
(b) Complete breech with sacroanterior position
(c) Non engaged breech
(d) Adequate amniotic fluid
56.
Implantation of a fertilised ovum occurs on which day following fertilisation?

(a) Day 6
(b) Day 10
(c) Day 14
(d) Day 20
57.
During total abdominal hysterectomy the ureter is likely to undergo injury or ligation during
the following steps EXCEPT:

(a) During division and ligation of the round ligaments
(b) During division and ligation of infundibulopelvic ligaments
(c) During division and ligation of mackenrodt?s and uterosacral ligaments
(d) At the vaginal angles while incising the vagina to remove the cervix with the
uterus
58.
The net effect of antenatal care has been the following EXCEPT:

(a) Reduction in maternal mortality
(b) Reduction in perinatal mortality
(c) Reduction in the incidence of institutional delivery
(d) Reduction in maternal morbidity
59.
Which one of the following is NOT a component of active phase in the partograph?

(a) Acceleration phase
(b) Phase of maximum slope
(c) Phase of deceleration
(d) Phase of expulsion
60.
From medicolegal point of view which one of the following is NOT a sign of previous child
birth?

(a) Perineum is lax and there is evidence of scarring
(b) Introitus is gaping and there is presence of carunculae myrtiformis
(c) Abdomen is lax and loose with striae and linea alba
(d) Conical cervix with round external os





61.
The components of partograph are all EXCEPT:

(a) Time
(b) Fetal heart rate
(c) Maternal respiratory rate
(d) Maternal urine analysis
62.
Which of the following information are provided by partograph?

1. Colour of liquor
2. Uterine contractions with duration and frequency
3. Dilatation of cervix

Select the correct answer using the code given below:

(a) 1 and 2 only
(b) 2 and 3 only
(c) 1 and 3 only
(d) 1, 2 and 3
63.
Intraoperative recognition of ureter is by which of the following features?

1. Transparent tubular appearance
2. Pale glistening appearance
3. Longitudinal vessels on surface
4. Circumferential vessels on surface

Select the correct answer using the code given below:

(a) 1 and 3
(b) 2 and 4
(c) 2 and 3
(d) 1 and 4
64.
Hysterosalpingography (HSG) is least helpful in detecting which of the following?

(a) Tubal patency
(b) Pelvic adhesions
(c) Asherman syndrome
(d) Congenital uterine anomaly









65.
Which of the following are characteristics of Trichomonas vaginitis?

1. Presence of greenish frothy discharge
2. Vaginal pH > 4.5
3. Presence of clue cells in microscopic examination
4. Strawberry spots on the vaginal mucosa

Select the correct answer using the code given below:

(a) 1, 2 and 3
(b) 1, 2 and 4
(c) 2, 3 and 4
(d) 1, 3 and 4
66.
Tumor marker of epithelial ovarian carcinoma is:

(a) Ca.125
(b) Alpha feto protein
(c) Beta HCG
(d) LDH
67.
The most common site of cervical cancer is:

(a) Endocervix
(b) Ectocervix
(c) Transformation zone
(d) Isthmus
68.
The placenta synthesizes all EXCEPT:

(a) Oestriol
(b) Corticotrophin releasing hormone
(c) PAPP-A(Pregnancy Associated Plasma Protein A)
(d) Dehydroepiandrosterone
69.
Withdrawal bleeding following administration of progesterone in a case of secondary
amenorrhea indicates all EXCEPT:

(a) Absence of pregnancy
(b) Production endogenous estrogen
(c) Endometrium is responsive to estrogen
(d) Defect in pituitary gland
70.
Monilial vaginitis is commonly associated with all EXCEPT:

(a) Prolonged antibiotic therapy
(b) Diabetes Mellitus
(c) Treatment of malaria with chloroquine
(d) Pregnancy
71.
Which one of the following is NOT a risk factor for the development of placenta previa?

(a) Maternal age
(b) Smoking
(c) Previous caesarean section
(d) Maternal anaemia
72.
Common clinical presentations of moderate to severe abruption are all EXCEPT:

(a) Uterine tenderness
(b) Fetal distress
(c) Unexplained pre term labour
(d) Prolonged labour
73.
Common trisomies resulting in spontaneous abortion are all EXCEPT:

(a) Trisomy 21
(b) Trisomy 18
(c) Trisomy 16
(d) Trisomy 1
74.
The initial prevention strategy for antiphospholipid syndrome will be:

1. Steroids
2. Heparin
3. Low dose aspirin
4. Progesterone support

Which of the above is/are correct?

(a) 2 and 3
(b) 3 and 4
(c) 3 only
(d) 1 and 4
75.
Diagnostic criteria for PCOD are:

1. Oligo/amenorrohoea
2. Hyperandrogenism
3. Polycystic ovaries on ultrasound

Which of the above are correct?

(a) 1 and 2 only
(b) 2 and 3 only
(c) 1 and 3 only
(d) 1, 2 and 3


76.
Which of the following symptoms can be associated with pelvic organ prolapse?

1. Difficulty in passing urine
2. Incomplete evacuation of urine
3. Urgency and frequency

Select the correct answer using the code given below:

(a) 1 and 2 only
(b) 2 and 3 only
(c) 1 and 3 only
(d) 1, 2 and 3
77.
A 30 year old lady, P
2
L
2
presents with painful unilateral swelling in vulva for 3 days. Which
of the following statements are true regarding the above case?

1. Bartholin?s abscess may be the likely diagnosis
2. It is to be managed by marsupialisation
3. Gonococcus is the most common pathogenic organism

Select the correct answer using the code given below:
(a) 1 and 3 only
(b) 3 only
(c) 1 and 2 only
(d) 1, 2 and 3
78.
Which one of the following is NOT a sign of separation of placenta?

(a) Uterus becomes globular, firm and ballotable
(b) The fundal height reduces further
(c) Slight bulging in the suprapubic region
(d) Apparent lengthenic of the cord with slight gush of vaginal bleeding
79.
Consider the following regarding examination of a rape victim:

1. Emergency pill is provided
2. Internal examination must be performed
3. HIV testing is done

Which of the above statements is/are correct?

(a) 1 and 3 only
(b) 2 only
(c) 1, 2 and 3
(d) 3 only




80.
Consider the following cardinal movements of mechanism of normal labor:
1. Engagement
2. Internal rotation
3. Flexion
4. Restitution
5. Crowning
6. External rotation

What is the correct sequence of movements in labor in occipito-lateral position?

(a) 1, 2, 3, 4, 5 and 6
(b) 1, 3, 2, 5, 4 and 6
(c) 2, 1, 3, 4, 5 and 6
(d) 3, 1, 2, 4, 6 and 5
81.
Which of the following represent the properties of an ideal disinfectant?

1. It is broad spectrum
2. It is fast acting
3. It is non-toxic

Select the correct answer using the code given below:
(a) 1 and 2 only
(b) 2 and 3 only
(c) 1 and 3 only
(d) 1, 2 and 3
82.
Which of the following measures can help reduce the risk of systemic hypertension?

1. Reduction in dietary intake of common salt
2. Controlling weight for age
3. Increasing potassium rich foods in the diet

Select the correct answer using the code given below:

(a) 1 and 2 only
(b) 2 and 3 only
(c) 1 and 3 only
(d) 1, 2 and 3








83.
Keeping biological determinants in perspective, consider the following statements:

1. Presence of a normal karyotype is the first requisite for human health
2. Genetic screening can play an important role in prevention of wide spectrum of
diseases
3. If an individual is allowed to live in healthy relationship with the environment, the
person?s genetic potentialities can transform into phenotypic realities

Which of the above statements is/are correct?

(a) 1 and 2 only
(b) 1 and 3 only
(c) 1, 2 and 3
(d) 2 and 3 only
84.
NITI Aayog has the following roles EXCEPT:

(a) It provides critical directional and strategic input in the development process
(b) It provides relevant technical advice with focus on technology upgradation
(c) It focuses on capacity building
(d) It focuses on the development of ?Referral Service Complex?
85.
Poor hand hygiene of a mess worker in a university college mess led to Hepatitis A cases in
the hostel inmates. What type of epidemic will this exposure present with?

1. Propagated
2. Common source-continuous exposure
3. Common source-point exposure

Select the correct answer using the code given below:

(a) 1 and 2
(b) 1 and 3
(c) 1 only
(d) 2 only












86.
What is the specificity of sputum microscopy in detection of Pulmonary Tuberculosis (PTB)
as per the information given below?

PTB Total
Sputum microscopy Present Absent
Positive 270 20 290
Negative 30 180 210
Total 300 200 500

(a) 10 %
(b) 36 %
(c) 90 %
(d) 94 %
87.
In a cohort of 500 women attending antenatal clinic, 70 % had ultrasonography (USG). This
cohort was followed up at delivery. Of the women who had USG, 70 delivered low birth
weight (LBW) babies; whereas of the women, who did not undergo USG, 50 delivered LBW
babies. The incidence of LBW babies among women who had USG is:

(a) 10 %
(b) 15 %
(c) 20 %
(d) 25 %
88.
Major source of vitamin K
1
is:

(a) Fresh dark green vegetables
(b) Exposure of body to sunlight
(c) Citrus fruits
(d) Foods rich in polyunsaturated fatty acids
89.
Which one of the following is NOT a function of Epidemiology?

(a) To study historically the rise and fall of disease in the population
(b) Searching for the causes and risk factor
(c) Identifying syndromes
(d) Making clinical diagnosis
90.
Which one of the following is NOT a contagious disease?

(a) Malaria
(b) Scabies
(c) Trachoma
(d) Leprosy


91.
In primary immune response, how much more antigenic dose is required to induce IgG
antibodies as against the induction of IgM antibodies?

(a) 10 times more
(b) 25 times more
(c) 50 times more
(d) 100 times more
92.
Pentavalent vaccine provides protection against which of the following diseases?

(a) Diphtheria, Pertussis, Tuberculosis, Measles and Hepatitis B
(b) Diphtheria, Pertussis, Measles, Hepatitis B and Hib
(c) Diphtheria, Pertussis, Tetanus, Hepatitis B and Rubella
(d) Diphtheria, Pertussis, Tetanus, Hepatitis B and Hib
93.
Consider the following data for a country:
Population in 0-14 years of age ? 391,558,367
Population between 15?64 years of age ? 856,076,200
Population above 65 years of age ? 71,943,390
What shall be the dependency ratio of this country?

(a) 42.4 %
(b) 54.1 %
(c) 66.2 %
(d) 78.6 %
94.
Which one of the following statements regarding Rabies Immunoglobulin is NOT true?

(a) It should be administered only once as soon as possible after the initiation of post
exposure prophylaxis
(b) It should be administered all into or around the wound sites
(c) There is no scientific ground for performing a skin test prior to administering equine
immunoglobulin
(d) It can be administered till 15 days after the first dose of anti-rabies vaccine
95.
All of the following are true about Bedaquiline (BDQ) EXCEPT:

(a) It specifically targets mycobacterial ATP synthase
(b) It is a bacteriostatic drug
(c) It has extended half life
(d) It has high volume of tissue distribution
96.
Visual inspection based screening test with 5 % acetic acid is used for the screening of which
one of the following cancers?

(a) Lung cancer
(b) Cervix cancer
(c) Oral cancer
(d) Breast cancer
97.
Predictive accuracy of a screening test depends on the following EXCEPT:

(a) Disease prevalence
(b) Disease incidence
(c) Sensitivity of screening test
(d) Specificity of screening test
98.
How much of Zinc supplement is recommended by WHO and UNICEF for infants less than 6
months of age after an episode of acute diarrhoea?

(a) 20 mg per day for 10?14 days
(b) 10 mg per day for 10?14 days
(c) 5 mg per day for 7 days
(d) 6 mg per day for 7 days
99.
Which one of the following statements regarding sequential administration of Inactivated
Polio Vaccine (IPV) and Oral Polio Vaccine (OPV) is NOT correct?

(a) It will be cost effective in developing countries for Polio prevention
(b) The combined schedules of IPV and OPV appear to reduce or prevent Vaccine
Associated Paralytic Polio (VAPP)
(c) Intestinal mucosal immunity is lost due to IPV administration
(d) IPV and OPV together may optimize both the humoral and mucosal immunogenicity
of Polio vaccine
100.
What is the recommended dose regimen of Vitamin A for the treatment of early stages of
Xerophthalmia?

(a) Single massive dose of 2 lac International Units (IU)
(b) 2 lac IU on two successive days
(c) 2 doses of 1 lac IU in two successive days
(d) 2 doses of 1 lac IU at a gap of one week
101.
Which is the most specific causative agent of Rabies?

(a) Lyssavirus serotype 1
(b) Lyssavirus serotype 2
(c) Lyssavirus serotype 3
(d) Lyssavirus serotype 4
102.
Which one of the following is the antibiotic of choice for the prevention of Rheumatic heart
disease?

(a) Benzathine Benzyl Penicillin
(b) Procaine Penicillin
(c) Doxycycline
(d) Ciprofloxacin


103.
All of the following are global targets for WHO Global Action Plan (2013?2020) for
Prevention and Control of NCDs, EXCEPT:

(a) A 10 % relative reduction in risk of premature mortality from cardiovascular diseases,
cancer, diabetes and chronic respiratory diseases
(b) A 10 % relative reduction in prevalence of insufficient physical activity
(c) A 10 % relative reduction in mean population intake of salt/sodium
(d) At least 10 % relative reduction in the harmful use of alcohol
104.
Under the Employees State Insurance (ESI) Scheme, extended sickness benefit is provided in
which of the following infectious diseases?

1. Tuberculosis
2. Leprosy
3. Chronic empyema

Select the correct answer using the code given below:

(a) 1 and 2 only
(b) 2 and 3 only
(c) 1 and 3 only
(d) 1, 2 and 3

105.
In the context of NCD prevention and control in India, the extent of relative reduction in
household use of solid fuels as a primary source of energy for cooking by 2025 is targeted at:

(a) 30 %
(b) 40 %
(c) 50 %
(d) 60 %
106.
Consider the following criteria which may indicate elimination of lymphatic filariasis in a
community:

1. When lymphatic filariasis ceases to be a public health problem in the community
2. When the number of microfilaria carriers declines to 1.5 % within the community
3. When children born in the community after the initiation of elimination programme
are free from circulating antigenaemia

Which of the criteria stated above hold true?

(a) 1 and 2 only
(b) 2 and 3 only
(c) 1 and 3 only
(d) 1, 2 and 3



107.
Hold over time of cold chain equipment depends on all of the following factors EXCEPT:

(a) Ambient temperature
(b) Quantity of vaccines kept
(c) Types of vaccines kept
(d) Condition of icepack lining
108.
Which of the following statements regarding Physical Quality of Life Index (PQLI) are
correct?

1. It consolidates infant mortality, life expectancy at age one, and literacy
2. It does not measure economic growth but measures the result of economic policies
3. For each component, the performance of individual countries is placed on a scale of
0-100

Select the correct answer using the code given below:

(a) 1 and 2 only
(b) 2 and 3 only
(c) 1 and 3 only
(d) 1, 2 and 3
109.
Disability-adjusted life years (DALYs) include:

(a) Years of lost life (YLL)
(b) Years lost to disability (YLD)
(c) Both YLL and YLD
(d) Neither YLL nor YLD
110.
Which of the following are health care delivery indicators?

1. Population per trained birth attendant
2. Population per health/sub centre
3. Doctor-nurse ratio

Select the correct answer using the code given below:

(a) 1 and 2 only
(b) 2 and 3 only
(c) 1 and 3 only
(d) 1, 2 and 3








111.
Which of the following key action areas are incorporated in the Ottawa Charter?

1. Building healthy public policy
2. Strengthening community action for health
3. Reorienting health services

Select the correct answer using the code given below:

(a) 1 and 2 only
(b) 2 and 3 only
(c) 1 and 3 only
(d) 1, 2 and 3
112.
Under Chapter XXI of the 10
th
revision of the International Statistical Classification of
Diseases and Related Health Problems (ICD-10), lifestyle-related problems fall under which
of the following code range?

(a) U 50.0 ? U 50.5
(b) U 10.0 ? U 10.5
(c) Z 72.0 ? Z 72.5
(d) Z 10.0 ? Z 10.5
113.
Which of the following diseases are spread by airborne transmission?

1. Influenza
2. Chicken pox
3. Q-fever
4. Psittacosis

Select the correct answer using the code given below:

(a) 1 and 3 only
(b) 1 and 4 only
(c) 3 and 4 only
(d) 1, 2, 3 and 4
114.
DASH diet plan is advocated in the control of which one of the following diseases?

(a) Cancer
(b) Chronic Obstructive Pulmonary Disease
(c) Hypertension
(d) Rheumatic heart disease
115.
Endemic ascitis occurs due to:

(a) Naturally occurring toxin
(b) Bacterial toxin
(c) Fungal toxin
(d) Viral toxin
116.
The presence of which of the following bacteria in drinking water is an important
confirmatory evidence of recent faecal pollution of water?

(a) E. coli
(b) Streptococci
(c) Clostridium
(d) Klebsiella
117.
With regard to the Goals, Milestones and Targets for the Global Technical Strategy for
Malaria (2016-2030), consider the following statements:

1. Compared to 2015, the target is to reduce the malaria mortality rates globally by at
least 90% by year 2030
2. Compared to 2015, the target is to reduce malaria case incidence by 90% by year
2030
3. Eliminate malaria from at least 35 countries, where malaria was transmitted in
2015, by year 2030

Which of the above statements hold true?

(a) 1 and 2 only
(b) 2 and 3 only
(c) 1 and 3 only
(d) 1, 2 and 3
118.
Which one of the following IUDs is associated with a low pregnancy rate (0.2 per 100), less
number of ectopic pregnancies and lower menstrual blood loss?

(a) Cu T-380A
(b) ML-Cu 375
(c) Progestasert
(d) LNG-20(Mirena)
119.
The incidence of diarrhoea is highest among infants in the age group of 6?11 months due to
all of the following reasons EXCEPT:

(a) Declining level of maternal antibodies
(b) Introduction of foods which may be contaminated
(c) Eruption of teeth
(d) Direct contact with human or animal faeces









120.
In the context of ?Anti-Malaria Month Campaign?, consider the following statements:

1. It is observed every year in the month of June
2. It is planned during the Monsoon season
3. It aims to enhance the level of community awareness and participation

Select the correct answer using the code given below:

(a) 1 and 2
(b) 2 and 3
(c) 3 only
(d) 1 and 3




















FirstRanker.com - FirstRanker's Choice

This post was last modified on 02 August 2021